美华管理传播网,中国经济管理大学,工商管理MBA专业资源库(28年)

 找回密码
 注册

QQ登录

只需一步,快速开始

查看: 86436|回复: 10

【人力资源管理】《HRM》11讲--中国经济管理大学

[复制链接]

该用户从未签到

发表于 2010-10-3 20:08:56 | 显示全部楼层 |阅读模式
As competitors strive to win the war for talent, effective human resource management is necessary to gain true competitive advantage in the marketplace. Three challenges companies face are sustainability, technology, and globalization. Human Resource Management Fifth Edition brings these challenges to life by highlighting real-world examples pertaining to these issues and relating it to the concepts within the chapter.
Chapter Summary


This chapter discusses the role of the Human Resource Management (HRM) function in the corporate effort to gain a competitive advantage. The chapter first discusses the roles and skills that a human resource management department and/or managers need for any company to be competitive.
The second section of the chapter identifies the competitive challenges that U.S. companies currently face, which influence their ability to meet the needs of shareholders, customers, employees, and other stakeholders.
We discuss how these competitive challenges are influencing HRM.
The chapter concludes by highlighting the HRM practices covered in this book and the ways they help companies compete.


Learning Objectives

After studying this chapter, the student should be able to:

1.
Discuss the roles and activities of a company’s human resource management function.

2.
Discuss the implications of the economy, the makeup of the labor force, and ethics for company sustainability.

3.
Discuss how human resource management affects a company’s balanced scorecard.

4.
Discuss what companies should do to compete in the global marketplace.

5.
Identify the characteristics of the workforce and how they influence human resource management.

6.
Discuss human resource management practices that support high performance work systems.

7.
Provide a brief description of human resource management practices.


Extended Chapter Outline

Note: Key terms are boldface and are listed in the "Chapter Vocabulary" section.

Opening Vignette: At Xerox,
Human Resource Management Excellence Helps Company Rebound


Since 1993 Xerox has been one of the innovators in using technology for HR functions.
The HR function is a shared services organization in which pay, bonuses, staffing, recruiting, benefits, diversity, learning, and HR systems are all part of corporate HR.
HR has provided support for the business strategy at Xerox as it evolved to help the company survive.
Today, HR continues to ensure that talented employees get the right experiences, job assignments, visibility, and learning opportunities.

I.

Introduction



A.
Competitiveness refers to a company's ability to maintain and  gain market share in its industry.



1.
Competitiveness is related to effectiveness, which is determined by whether the company satisfies the needs of stakeholders (groups affected by business practices).



B.
Human Resource management (HRM)
refers to policies, practices and systems that influence employees’ behaviors, attitudes and performance. (See Figure 1.1)



1.
Effective HRM practices have been shown to relate to company performance by contributing to employee and customer satisfaction, innovation, productivity, and development of a favorable reputation in the community in which the firm is located.



2.
The HRM contribution has only recently been recognized.



II.
What responsibilities and roles do HR departments perform?



A.
Responsibilities that the HR department is solely responsible for include outplacement, Labor Law Compliance, recruiting, compensation, and health and safety.



B.
Many different roles and responsibilities can be performed by the HR department depending on the size of the company, the characteristics of the workforce, the industry, and the value system of the company’s management.



C.
The HR function can play roles in the management of strategic human resources (strategic partner), the management of company infrastructure (administrative expert), the management of transformation and change (change expert), and the management of employee contribution (employee advocate). (See Figure 1.2and Table 1.1)




D.
Roles of HRM include the following:




1. Strategic partner – aligning HRM strategies to business strategies is important to help the company execute its business strategy.
Administrative expert – includes designing systems for selection, developing, appraising, and rewarding employees.

Employee advocate – entails managing the commitment and contributions of employees.

Change agent – requires that HRM help transform organizations to meet the new competitive conditions.





III.
What skills do HR professionals need? Figure 1.3 shows the competencies that are needed by HR professionals. (See Figure 1.3)



1.
Ability to consider current and future business goals and how HRM can contribute.



2.
Ability to analyze turnover, retention, productivity, and customer service problems to recommend potential HRM solutions (strategic partner).



3.
Ability to overcome resistance to new HRM policies and procedures, technology, and work designs (change agent).



4.
Ability to coach and counsel employees and represent their views to management (employee advocate).



5.
Ability to design and deliver effect HRM systems and understand how technology can make HRM systems more efficient and less costly (administrative expert).




IV.
How is the HRM function changing?



A.
The amount of time that the HR function is devoting to administrative tasks is decreasing and its role as a strategic business partner, change agent, and employee advocate are increasing.



B.Advancement in technology is enhancing self-service.
Self-service is the process of giving employees control of HR transactions. This allows employees to take greater responsibility for their own careers.



C.
Outsourcing
refers to the practice of having another company provide services.



V.
The HRM Profession



1.
There are many different types of jobs in the HRM profession.
(See Table 1.3)



2.
The primary professional organization for HRM is the Society for Human Resource Management (SHRM).



VI.
Competitive Challenges Influencing Human Resource Management
(See Figure 1.4)



A.
The Sustainability Challenge




1.
Sustainability refers to the ability of a company to survive and succeed in a dynamic competitive environment.




2.
Company success is based on how well the company meets the needs of its stakeholders.
Stakeholders refer to shareholders, the community, customers, and all of the other parties that have an interest in seeing that the company succeeds.




3.
Several changes in the economy have important implications for human resource management.
Some key statistics about the economy and the workforce are shown in Table 1.4.





4.
In 2000 the economic picture was positive for the United States.
Then came September 11, 2001.
The events of 9/11 combined with an economic recession put companies into a more uncertain economic period.


5.
The competition for labor is affected by the growth and decline of industries, jobs, and occupations.
Competition for labor is also influenced by the number and skills of persons available for full-time work.
(See Figure 1.5)



a.
Most of the new job growth in professional specialties is expects among teachers, librarians, and counselors: computer, mathematical, and operations research occupations; and health assessment and treatment occupations.



b.
The largest number of job openings will be in occupations requiring a bachelor’s degree and on-the-job training.



c.
Retail will continue to provide jobs for unskilled workers.



6.
Increased Value Placed On Knowledge



a.
Intellectual capital refers to the creativity, productivity, and service provided by employees.
Effective management of people is key to boosting the value of intellectual capital.



b.
Knowledge workers are employees who own the means of producing a product or service.



c.
Empowerment means giving employees responsibility and authority to make decisions regarding all aspects of product development or customer service.



d.
As more companies become knowledge-based, they must promote and capture learning at the employee, team, and company levels.




e.
A changing environment means that all employees must embrace a philosophy of learning.
A learning organization embraces a culture of lifelong learning, enabling all employees to continually acquire and share knowledge.



7.
Skills Requirements



a.
The demand for specific skills is being replaced by a need for cognitive skills—mathematical and verbal ability—and interpersonal skills related to being able to work in teams or to interact with customers in a service economy.



b.
Most companies related these skills to educational attainment, using a college degree as a standard to screen prospective new employees.



8.
Changes in the Employment Relationship



a.
Layoffs and bankruptcies have played a major role in changing the employment relationship.
The psychological contract describes what an employee expects to contribute and what the company will provide to the employee for these contributions.



b.
In the new economy a new type of psychological contract is emerging.
Companies demand excellent customer service and high productivity levels.
Employees are expected to take more responsibility for their own careers, from seeking raining to balancing work and family.
In exchange, employees want companies to provide flexible work schedules, comfortable working conditions, more autonomy in accomplishing work, training and development opportunities, and financial incentives based on how the company performs.



c.
Alternative work arrangements include independent contractors, on-call workers, temporary workers, and contract company workers.




9.
Demanding Work, but with More Flexibility



a.
The globalization of the world economy and the development of e-commerce have made the notion of a 40-hour work week obsolete.



b.
Many companies are taking steps to provide more flexible work schedules, protect employees’ free time, and more productively use employees’ work time.



10.
There are several ways that companies are trying to win the war for talent; all involve HRM practices.
They include finding creative ways to avoid layoffs and retirement, creating a positive work environment, and expanding the search for talent to the global labor force.



B.
The balanced scorecard: Measuring performance to stakeholders



1.
The balanced scorecard gives managers the opportunity to look at the company from the perspective of internal and external customers, employees and shareholders.The balanced scorecard should be used to:

a.
Link human resource management activities to the company’s business strategy.
b.
Evaluate the extent to which the human resource function is helping the company’s meet its strategic objectives.


Measures of human resource practices primarily relate to productivity, people, and process.


Competing Through Sustainability



A Long-term View Pays Dividends for Vanguard Group



For companies like Vanguard Group, sustainability is based on a business model that emphasizes commitment and loyalty to employees as well as high performance.
Vanguard’s human resource management focus has changed over the 28 business years it has been in business.
The company realizes that sustainability is not based on a program but a value system and actions based on the values that meet employee and customer needs.




C.
Customer Service and Quality emphasis



1.
Customer excellence requires attention to product and service features as well as to interactions with customers.



2.
Total Quality Management can be defined as "a cooperative form of doing business that relies on the talents and capabilities of both labor and management to continually improve quality and productivity using work teams." This is one strategy companies are currently using.




a.
The TQM movement has alerted management to the key role HRM plays in the achievement of quality. (See TM1.6 and Table 1.6 in the text for HRM practices TQM companies use.)



b.
The Malcolm Baldrige National Quality Award was established by President Reagan to publicize successful quality strategies. (See Table 1.7 for the award criteria)



Example: Xerox Corporation received the Malcolm Baldrige Award in 1989 after changing its policies 71) to satisfy the customer and (2) to make quality improvement the job of every employee.
Operations Management International, Inc. (OMI) is a 2001 Baldrige Award winner in the service category.




c.
The ISO 9000:2000 standards were developed by the International Organization for Standardization in Geneva, Switzerland.
ISO 9000 is the name of a family of standards (ISO 9001, ISO 9004) that includes requirements such as how to establish quality standards and document work processes to help workers understand quality system requirements.
ISO 9000:2000 has been adopted as a quality standard in nearly 100 countries.



d.
The Six Sigma process refers to a system of measuring, analyzing, improving, and then controlling processes once they have been brought within the narrow Six Sigma quality tolerances or standards.
The objective of Six Sigma is to create a total business focus on serving the customer—that is, deliver what customers really want when they want it.



D.
Changing demographics and diversity of the workplace

1.
The labor force of current is often referred to as the internal labor force.



2.
The external labor market includes persons actively seeking employment.



3.
As figure 1.6 shows, the U.S. workforce is becoming increasingly diverse.
To successfully manage a diverse workforce, managers must develop a new set of skills, including:



a.
Communicating effectively with employees from a wide variety of cultural backgrounds.



b.
Coaching and developing employees of different ages, educational backgrounds, ethnicity, physical ability, and race.



c.
Providing performance feedback that is based on objective outcomes rather than values and stereotypes that work against women, minorities, and handicapped persons by prejudging the persons’ abilities and talents.




d.
Creating a work environment that makes it comfortable for employees of all backgrounds to be creative and innovative.



E.
Legal and Ethical Issues



1.
Five main areas in the legal envi ronment have influenced HRM practice: equal opportunity legislation, safety and health legislation, pay and benefits legis lation, employee privacy, and job security. Most recently, the Americans with Disabilities Act regulations require employers to accommodate mental and physical limitations of otherwise qualified individuals, unless undue expense would be incurred.



2.
Ethical criteria should be used in company and employee interactions. Ethical principles include the emphasis of mutual benefits, employees assuming responsi bility for company actions, a meaningful corporate vision for employees, and fairness.




Example: The Raytheon Company has a checklist including several questions that is distributed to each employee to use when considering whether an action is ethical.
Raytheon reinforces its ethics codes with formal training programs, a toll-free “ethics line,” and full-time ethics offices and officers in all of its major business units.
Mandatory one-hour ethics training for all employees uses case studies to make employees aware of the ethical problems that may occur at work.



a.
The Sarbanes-Oxley Act of 2002 sets strict rules for corporate behavior especially in regards to accounting procedures.



F.
The Global Challenge - In order to survive, U.S. companies must, improve HRM practices, develop global markets and better prepare employees for global assignments.



1.
Development of Global Markets.
The most admired and successful companies in the world have not only created multinational corporations, but have created organizations with workforces and corporate cultures that reflect the characteristics of the global markets in which they operate.




2.
Globalization has affected not only businesses with international operations.
Companies without international operations buy or use goods that have been produced overseas, hire employees with diverse backgrounds, and compete with foreign-owned companies operating within and outside of United States.
Globalization is not limited to a particular sector of the economy or product market.



3.
Global business expansion has been made easier by technology.
Using the Internet, e-mail, and video-conferencing, business deals can be completed between companies thousands of miles apart.



4.
To compete in the world economy, U.S. companies need to put greater effort into effective HRM practices.



5.
U.S. companies must do a better job of preparing employees and their families for international assignments, especially given the current high failure rate.



6.
Globalization also means that U.S. companies may move jobs overseas.
Offshoring refers to the exporting of jobs from developed countries to less developed countries.





Competing Through Globalization



Building a Common Company Culture in a Global Company



Pricewaterhouse Cooper is a global organization that provides assurance, tax, and advisory services.
The company developed a global leadership training program to encourage development of innovative ideas and relationships between future company leaders.
Thanks to the program, individuals from different countries and cultures interact with each other to learn.
The company found that the program is highly valuable.





G.
The Technology Challenge

1.
Advances in sophisticated technology along with reduced costs for the technology are changing many aspects of human resource management.
Technology is pushing the boundaries of artificial intelligence, speed synthesis, wireless communications, and networked virtual reality.


2.
High&#8209erformance Work Systems - To gain competitive advantage with foreign competitors, U.S. companies must learn to integrate technology and structure into high‑performance work systems. Additionally, advances in telecommunications, micro processors, etc., are having a major impact on necessary to job success.


a.
Change in Employees' Work Roles and Skill Requirements— New technology often requires new basic skills. For example, computer‑integrated manufacturing requires employees to monitor equipment and troubleshoot problems with sophisti cated equipment. More organizations want employees cross trained in multiple jobs. Interpersonal skills are increasingly important as teams are used.



b.
Increase in the Use of Teams to Perform Work—Employees are now expected to make decisions at point of sale to improve customer service. Work teams form one method of increasing both employee responsibility and control. These teams frequently assume managerial activities. Quality control may be integrated into the team’s functions. Cost savings may be real ized by the use of teams.




c.
Through technology, the information needed to improve customer service and product quality becomes more accessible to employees.
Popular methods for increasing employee responsibility and control are through work teams, and cross training.




3.
Changes in skill requirements – High performance work systems have implication for employee selection and training.



4.
Working in partnerships – Technology has allowed companies to form partnerships with one or more other companies.



5.
Changes in Company Structure and Reporting Relationships – In the adaptive organizational structure, employees are in a constant state of learning and performance improvement.
They are free to move wherever they are needed in the company.



6.
Increased Availability of human resource management databases and e-HRM—A human resource information system may contain large amounts of data regarding the company's employees. An HRIS may be used to support strategic decision-making and ensure legal compliance, as well as to evaluate programs and policies or support daily operating concerns.



Example: Northern Telecom has facilities in 90 countries and has created a central database of employee records worldwide. Headcount, salary, and recruiting data are updated regularly. This system has improved Northern Telecom's ability to meet customer needs and address global staffing issues.



6.
Electronic-human resource management (e-HRM) – E-HRM has the potential to change all traditional human resource management functions. (See Table 1.9 TM 1.8)




7.
Competitiveness in High&#8209erformance Work Systems—In order for new technologies or new structures to support organiza tional improvement, the integration of these two factors must be carefully examined. Human resource practices that support high‑performance work systems are listed in Table 1.10. These practices are designed to provide employees with skills, incen tives, knowledge, and autonomy. GE Fanuc Automation achieved its reputation and recognition for quality as a result of the use of high-performance work practices.
Central to its practices is the idea that employees closest to the work have the best improvement ideas.
As a result, employees must be encouraged to voice their opinions and make changes.



VII.
Meeting the Competitive Challenges through HRM Practices - HRM practices that help companies respond to the four competitive challenges can be grouped into the following four dimensions.
(See Figure 1.7 and 1.8)



A.
Managing the Human Resource Environment—HRM practices should be linked to strategy, legal compliance must be assured, and work should be designed to maximize employee effectiveness and motivation.



B.
Acquiring and Preparing Human Resources—Human resource planning must occur, employees must be recruited and placed in appropriate jobs, methods of employee selection must be determined, and ensuring employees have skills to perform particular jobs.



C.
Assessment and Development of Human Resources—This area of HRM deals with measuring employee performance, preparing employees for future work roles, identifying and responding to employee interests, and creating a beneficial and effective work environment.


D.
Compensating Human Resources—Appropriate pay systemsmust be developed, employee contributions must be rewarded,and employees must be provided with benefits.



E.
Special Issues—Labor unions create an additional HRM challenge.



VIII.
The final paragraph and Table 1.12 present the overall organizationof the text.


Chapter Vocabulary

These terms are defined in the "Extended Chapter Outline" section.

Competitiveness
Human Resource Management
Self-service
Outsourcing
Intellectual Capital
Knowledge Workers
Empowerment
Learning Organization
Psychological Contract
Alternative Work Arrangements
Balanced Scorecard
Total Quality Management (TQM)
Malcolm Baldrige National Quality Award
Six Sigma Process
Internal Labor Force

External Labor Market
Sarbanes-Oxley Act of 2002
Offshoring
Human Resource Information Systems (HRIS)
Electronic Human resource Management (e-HRM)


A Look Back

The opening vignette illustrated how managing human resources contributed to helping save Xerox.

Questions

1.
Consider the major dimensions of HRM practices shown in Figure 1.8.
For each dimension describe the role it can play in helping a company in a downsizing strategy such as Xerox faced.
What might be different if the company was prosperous and growing?


Student answers may vary.
Managing the Human Resource Environment would involve linking HRM practices to the company’s business objectives, ensuring that HRM practices comply with federal, state, and local laws, designing work that motivates and satisfies the employee as well as maximizes customer service, quality, and productivity.
Acquiring and preparing human resources involves identifying human resources requirement, and training employees to have skills needed to perform their jobs.
Assessment and development of human resources involves measuring employees’ performance, preparing employees for future work roles and identifying employees’ work interests, goals, values, and other career issues.
Compensating human resources involves creating pay systems, rewarding employee contributions, and providing employees with benefits.
Student answers regarding what might be different if the company was prosperous and growing will vary.

2.
What role should line managers play in helping support Xerox’s HR practices?
That is, what should they do?
Explain.

Student answers may vary.
The following suggestion may be used to explain the importance of line managers in HRM.


Line managers certainly play a very important role, regardless of the size of the company because they are the ones that know specifically what the jobs are and how employees perform.



Discussion Questions

1.
Traditionally, human resource management practices were developed and administered by the company's human resource department. Line managers are now playing a major role in developing and implementing HRM practices. Why do you think non-HR managers are becoming more involved in developing and implementing HRM practices?



As the relationship between various HRM practices and the productivity and performance of employees has been recognized, line managers have strong reasons to become involved in the development and implementation of HRM practices. The information from line managers is critical to determining needed and appropriate policies and practices that will reinforce the strategic and operational needs of the organization. For instance, if quality needs improvement, then it is critical that incentive/compensation practices be developed to reward quality improvement, rather than volume production.



2.
Staffing, training, compensation, and performance management are important HRM functions. How can each of these functions help companies succeed in the new economy?
Meet stakeholders’ needs?
High-performance work system challenges?
Global challenges?



Training may improve deficient skills, performance appraisal techniques may emphasize and measure quality improvement, and compensation practices may be used to reinforce needed change. Diverse work force, low skill levels of entry‑level workers could be helped by training and performance management techniques. Obviously, using selection processes that are fair and nondiscriminatory while selecting the best employees is important. And again, compensation systems that reward the acquisition of needed skills could help here.



In terms of meeting stakeholders needs challenge, students might comment that the selection of employees with appropriate skills would help meet the organization’s needs.



The work systems challenge necessitates that employees adjust to working in team structures (training and selection), that they receive higher‑level and basic skills training, and once again that compensation and performance management systems are used to motivate and reinforce appropriate behaviors.




Finally, the global challenge may produce the organizational need for employees who speak multiple languages, who are trained to adjust to new cultures and who are sensitive to cultural differences. Employees with the flexibility and skills needed to work in various cultures or with individuals of diverse backgrounds may be selected or developed through training. Appropriate rewards will maintain these valuable employees.




3.
This book covers four human resource management practice areas: managing the human resource environment, acquiring and preparing human resources, assessment and development of human resources, and compensating human resources. Which area do you believe contributes most to helping a company gain a competitive advantage? Which area do you believe contributes the least? Why?


Each student’s answer will vary to this question based on opinion. An example can be: Acquiring and preparing human resources is the most important HRM practice because well-trained employees are the most important aspect of a business. On the other hand, compensating human resources is not at all important because money is not what drives all employees. However, contradictory arguments can be made for each of these points as well.

4.
What is the balanced scorecard? Identify the four perspectives included in the balanced scorecard. How can HRM practices influence the four perspectives?


The balanced scorecard gives managers an indication of the performance of a company based on the degree to which stakeholders’ needs are satisfied. This gives managers the opportunity to look at the company from the perspective of internal, external, employees, and shareholders. Communicating the scorecard to employees gives them a framework that helps them see the goals and strategies of the company, how these goals are measured, and how they influence the critical indicators.

5.
Is HRM becoming more strategic? Explain your answer.


Technology is allowing the typical HRM to steer away from everyday tasks to focus more on the strategic side. Self-service, empowerment, and outsourcing are also increasing strategic roles. There is an increase in managers in charge of human resource function being included on high-level committees that are shaping the strategic direction of the company.


6.
Explain the implications of each of the following labor force trends for HRM: (1) aging workforce, (2) diverse workforce, (3) skill deficiencies.


The aging workforce means that employees will increasingly face issues such as career paltering, retirement planning, and retraining to avoid skill obsolescence. High healthcare costs will also be a struggle with the aging employees.

The diverse composition of the workforce challenges HRM to ensure that the talents and skills of all employees are fully utilized for the good of the organization. Immigration is an important factor contributing to the changing workforce. HRM need to be aware of the positive and negative aspects of immigration.

Projections show that the supply of entry-level labor will not have the education and skills to meet basic job demands. Individuals will have low reading, writing, and impersonal skills.

7.
What role do HRM practices play in a business decision to expand internationally?


With opportunities opening up in many emerging markets, it is important that a company creates a multinational culture that is characteristic of the culture in which it operates in. Successful companies realize that the most important asset to a business is its employees. This translates into HRM playing a big role in international expansion.

8.
Is business emphasis on quality a fad? Why or why not? What might a quality goal and high performance work system have in common in terms of HRM practices?


Business emphasis on quality has proven to be more than just a fad. The idea of quality has allowed companies to gain a competitive advantage. In terms of HRM practices, a quality goal and high performance work system have a lot in common. Both require increased use of teams, empowering employees and changing employee roles and company structure. A quality goal is central to the high performance work systems.


9.
What disadvantages might result from outsourcing HRM practices? From employee self-service? From increased manager involvement in designing and using HR practices?


The primary disadvantages of outsourcing HRM practices include the loss of strategic ties in the organization because someone else is handling the company’s HR practices. It also may have a potential negative impact on employee behavior and attitudes. The disadvantages of employee self-service include possible derailment of their careers. The possible disadvantage from increased managerial involvement might be too much structure and authoritative HR practices. It would also go against the quality and high performance work system principles.

10.
What factors should a company consider before offshoring?
What are the advantages and disadvantages of offshoring?


A company should consider a variety of issues, including interests of stakeholders, economic situations in potential offshoring locations, governmental implications offshoring, and the morale of employees.



Advantages of offshoring usually include higher profit levels due to lower labor costs and operational costs.
Disadvantages include displaced local workers, low employee morale, and lower customer loyalty.

Manager’s Hot Seat Exercise: Ethics: Let’s Make a 4th Quarter Deal

I.
Introduction


This scenario provides a depiction of the ethical dilemmas that may arise when two legitimate organizational goals are in conflict:
generating revenue and legal business practices.
The manager in this situation is presented with a risky course of action that, if it worked, would have tremendous benefits for his company and department.
Interpersonal communication styles and influence tactics are also demonstrated and provide a stimulus for discussion among students.


II.
Learning Objectives


    To analyze the causes of ethical dilemmas
    To evaluate influence techniques and communication styles
  • To apply principles of ethical decision making


III.
Scenario Description:


Overview:
A Wall Street trading company, Smith/Blackwell, is coming to the end of their 4th Quarter.
BesTel Inc. has offered to invest 4 million dollars with Smith/Blackwell which would not only save the department from lay-offs but would create year-end bonuses for the manager’s team, which in recent months has been plagued by very low morale.
In order to process the investment before year’s end, some procedural steps would need to be ‘abbreviated/skipped/ignored’.
The shareholders of BesTel will not meet until mid-January and therefore will not approve the investment until then.

Therefore, Gina presents a situation in which she plans to side-step procedure to arrive at the advantageous outcome for the department and wants Jason’s support.
While the Chairman of the Board, Jack, has assured Gina that the vote will pass and that he’s talked to the key shareholders personally, the truth is, he does not have the power to make this decision without the boards’ actual vote.


Profile:
·
Jason Powell, Director of New Accounts, manages a team of 25 people.
Held position for 4 years but in the past two years, investments have fallen by over 50% department-wide.

·
Gina Travers, Assets Manager.
Gina has been working in the financial industry for over nine years.
She has been with Smith/Blackwell for four years and was hired by Powell.
To date, for the year 2002, Gina has brought in 3.5 million dollars in investments.


References:
The references included in the DVD are:
·
Concepts in Ethical Behavior
(PPT 2-3)

·
Source for Code of Ethics (PPT 2-5)
·
Three Principles to Guide Ethical Conduct (PPT 2-9)
·
Approaches to Social Responsibility (PPT 2-11)

Back History:
As the economy worsens, the new accounts department is facing layoffs and downsizing.
If the 4th Quarter numbers do not improve substantially, up to 50% of their department will be laid off and the year-end bonuses will be foregone for the entire department. The environment is stressed and morale is low.


Scene Set-up: Travers makes an appointment with Powell to discuss some recent opportunities.


Scene Location: Powell’s office.

The Meeting - Summary:
Gina, the account representative approaches Jason with a “deal” from BesTel that will save the department and allow everyone to earn their bonuses for the year.
Jason is reluctant to jump on board with this plan because the transaction would not be approved by the company’s shareholders in time for the 4th quarter.


Jason attempts to figure out a way to ensure that they “do things right” by getting the deal in writing and talking to Jack personally with Gina.
Gina argues that following procedure is not going to work in this case and that sidestepping the rules will serve the greater good.
She also suggests that taking risks is critical to success.
The conversation escalates and Gina clearly becomes agitated and stressed.
She threatens to go over Jason’s head, gives him an ultimatum, and tries to appeal to their long-time working relationship to influence Jason.
Eventually, she storms out of the room leaving the situation unresolved.


Afterthoughts – Summary:
Jason did not think the meeting went well.
He was concerned with Gina’s insistence on rushing to action without thoroughly thinking through the consequences.
He noted that there is always tension between the salesperson and accounting/legal department’s roles in an organization.
He suggested that he was playing the accounting role to ensure that they did not engage in an illegal deal.


Dossier:
The specific artifacts included in the DVD are:


1.
Document outlining the “restructuring” of the New Accounts Department, should they not reach their financial goals for the 4th Quarter.

2.
Gina’s notes on the financial impact the deal will have – if were to go through, and if it were not.

3.
BesTel Chairmans’ voice-mail assuring Gina that “all systems are go” and that he’s counting on her to make this happen.


IV.
Discussion Questions:


The References and related Discussion Questions may be found in PowerPoint slides 2-1 to 2-11 on the Instructor side of the text’s Website.

Learning Objective #1:
To analyze the causes of ethical dilemmas


1.
What is the source of this conflict and what role has the organization played to contribute to this dilemma? (PPT 2-2)

The account representative is very motivated to make this deal go through because the organization has tied her bonuses and salary increases to the dollar amount of investments she brings to the company.
The manager also is rewarded if his department makes sales.
On the other hand, he has the responsibility to protect the company’s assets and reputation by engaging in lawful business agreements.
Thus, these competing goals set-up difficult ethical decisions for the manager and employees.


´
1.
Why is Jason skeptical?

a.
Client isn’t reliable
b.
Gina isn’t reliable
c.
Deal seems fishy

Deal seems fishy -
Jason is skeptical because the initial approach and information provided by Gina raises some red flags regarding the legality and appropriateness of pursuing this deal.


2.
Which of the “Concepts in Ethical Behavior” are relevant to this scenario?
Why? (PPT 2-2 & 2-3)


Most would be considered in this scenario, students’ answers will vary.
Certainly Professional ethics would be a driving factor in this case because there are rules and procedures that individuals in this profession are bound to.
Also, organizational stakeholders should be considered because if the company engages in an unlawful deal, the reputation and ultimate longevity of the company could be at risk – on the other hand some of the employees may lose their jobs if revenue doesn’t come in this quarter.



3.
Organizations can effectively help their employees to behave and act ethically by creating and communicating a code of ethics.
What factors in this scenario should be considered if you were to develop the code of ethics for Smith/Blackwell? See “Sources for Codes of Ethics” PPT 2-4 & 2-5.


Students should address each of the three aspects affecting the organization’s code of ethics and provide relevant examples based on the information provided in the case.

Learning Objective #2:
To evaluate influence techniques and communication styles


1.
Describe Gina’s initial approach with Jason.
What influence tactic does she use initially?
Was it successful? (PPT 2-6).


Initially, Gina relies on the common goals she and Jason share with regard to making their 4th quarter numbers to save the department from potential lay-offs.
Her emotion and elation at the solution is an attempt to appeal to Jason emotionally so that he’ll gloss over the important details that may prevent the deal.
Clearly, she was not successful as he began to examine the deal more closely.


´
2.
Jason’s behavior demonstrates:


a. caution


b. distrust

c.
nit-picking


Jason appears cautious because he is trying to understand the legitimacy of acting on the deal before the shareholders have voted.

2.
How does Gina handle Jason’s hesitation to move forward on the deal?
How does her reaction affect Jason? (PPT 2-6)


Gina gets flustered and does not use rational case to argue her point.
She begins to get defensive and uses a “bull-dozer” approach to influencing Jason rather than a thoughtful, rational one.
This causes Jason to become even more skeptical of the deal and becomes increasingly reluctant.
He does effectively to stay calm and does not raise his voice.


´
3.
Gina’s over-excited.
Jason should:

a.
Calm Gina
b.
Acknowledge her point
c.
Request written proposal

Jason may want to show Gina that he shares her goal of helping the employees of the department earn bonuses and keep their jobs.
By agreeing with some aspect of her proposal he may decrease her defensiveness and show her that he’s willing to consider alternatives that do not jeopardize the organization.


Jason ends up requesting a written proposal which agitates Gina further because she doesn’t see how that will accomplish her goal.

3.
Upon realizing the Jason will not easily sign-off on the deal, Gina attempts a variety of different influence techniques.
Describe them and evaluate how effective and appropriate they were given the situation. (PPT 2-7)


Gina tries to threaten Jason by saying she will go over his head and make the deal happen with Jason’s boss.
Depending on the culture of the organization, this may have a negative consequence for Gina because she is skipping organizational levels.
In addition, senior management are likely to have the same reaction as Jason which, again, will have a negative effect on her reputation.


Gina also tries to appeal to Jason based on their friendship and says that she cares about him.
After using a threat, this approach is quite ineffective because the sincerity on which this strategy relies is questionable.



´
5.
How should Jason respond to a threat?

a.
Ignore it
b.
Reprimand Gina
c.
Get to Ralph first

The threatening, or retribution, approach to influence is only appropriate under certain circumstances.
For example, when the initiator has complete control, the issue is extremely important (e.g., safety), and the relationship is relatively unimportant.
In this case, Gina is the subordinate (relatively less power) and the relationship with Jason is important.
Therefore, her choice of this influence strategy is poor and proves to be ineffective.
Jason should reprimand her and explain the inappropriateness of her comments.


Learning Objective #3:
To apply principles of ethical decision making


1.
Consider each of the principles to guide ethical conduct (PPT 2-9).
Analyze this situation from both Gina’s and Jason’s perspectives using these principles.
Which model did Gina rely on most heavily?
Jason? (PPT 2-8)


Answers will vary.
An argument could be made for any of the models.
Gina’s comment about the “greater good” resembles the Utilitarian model.
However, the negative consequence of the action could impact even more people than those who might enjoy positive consequences.
Jason might argue the Moral Rights position in that not allowing the shareholders to vote before the deal is done violates their rights.

´
4.
What are the risk factors?

a.
Losing deal
b.
Misleading shareholders
c.
Making illegal deal

In terms of consequences for the company’s reputation and potential losses, making an illegal deal is a significant risk.
Losing the deal would be unfortunate but is part of doing business.
Misleading shareholders in and of itself may not have any direct consequences but one should consider the whether they will be uncomfortable in the long run with that decision (i.e., will you be able to sleep at night).



2.
Review the “Approaches to Social Responsibility” (PPT 2-11).
Which approach is most closely aligned with Gina’s behavior?
Which approach most closely matches Jason’s?
Explain your choices. (PPT 2-10)


Gina’s behavior seems to resemble the Obstructionist approach because she is not concerned with the legality of her actions.
Jason is initially taking a defensive approach because he does not want to break the law.
However, he demonstrates willingness to explore other options which matches the Accommodative approach.


´
6. What should Jason do next?

a.
Let Gina make deal
b.
Call Jack himself
c.
Look for a new job

Jason can’t really let Gina make the deal without finding out more information.
If he calls Jack himself, he may make the company look disorganized and may discredit Gina who Jack knows and likes.
Looking for a new job may be long-term plan, but it won’t address the current issue.


Jason may want to let Gina cool down and then talk to her again.
Perhaps thinking about alternative courses of action together may solve the problem.
If the driving force behind her need to close this deal is revenue generation, perhaps there is another creative, yet legal, way to accomplish that goal.
Or there may be a legitimate approach to this deal that they have not explored.




Exercising Strategy

Container Store does Great HRM

Questions

1.
As the Container Store continues to expand and grow (the company has grown at an annual rate of 20-25 percent), do you think the manager’s role in human resource management should grow, shrink, or remain the same?
Explain.
Indicate what HRM practices managers should be responsible for and what practices should be the responsibility of human resource management staff.


Student answers will vary.
Some may say that managers should have more say than human resource management staff, others may take the opposite side.
Some students may say that HRM practices should be equally shared between all functions and levels.

2.
Consider our discussion in this chapter of HR roles and competencies.
What are the advantages and disadvantages for successfully performing these roles by having human resource management staff come from store level positions?


Student answers will vary.
One advantage may be that HR managers that came from store level positions will understand the employees better.
A possible disadvantage may be that these individuals may be too personally involved with employees to make difficult decisions.


Managing People



The Future of Work


Questions

1.
What can companies in their human resource management practices do to ensure that employees are flexible, creative, and provide good customer service?




Student answers will vary.
Creating an open avenue of communication between employees and HR is certainly important in fostering good customer service.
In addition, the HR department can screen applicants for customer service skills.
It can also emphasize customer service in training and development of employees.
Rewarding great customer service would encourage more employees to provide good customer service.



2.
Many companies have recognized the value of educating their workforce and as a result offer training programs and provide tuition reimbursement for employees seeking high school, or advanced degrees.
Many companies are concerned that they will not get a return on their investment in education.
That is, once employees finish their degree they will leave the company.
What should companies do to ensure that they get a return on the money they invest in education and training?




Student answers may vary.
Companies’ options for ensuring that they get a return on the money they invest in education and training are limited.
First, if an employee stands to improve his or her résumé greatly by taking expanded education or training, the company could ask for that employee to pay for all or part of the training/education costs.
Some companies require employees to sign “golden handcuff” clauses, in which the employee promises to stay with the company for a certain number of years after training is complete.
Some companies only pay for training/education programs for employees that have been with the company for a certain number of years.
These employees may be viewed as more reliable and personally invested in the company.



3.
Provide an example of a job that you believe will soon be replaced by technology or become so highly routinized that it will demand employees with low levels of skill who receive low pay.
Describe the job and discuss your reasons for choosing it.




Student answers will vary.






Additional Activities

Teaching Suggestions

The use of outside or additional assignments early in the term provides an opportunity for students to illustrate for themselves how HRM is an impor tant function within the organization. Following are several additional reading assignments, extra discussion questions for the "Competing through" boxes in the text, as well as a suggestion for an outside speaker. Getting students to provide examples of organizations that have effectively used HRM practices to help respond to competitive challenges sells them on the importance of the rest of the course!


Activities

1.
This chapter contains references to many current issues in business and HRM. One assignment that could help students see the relevance of what they will be studying is to ask them to read The Wall Street Journal, Business Week, or other resources and to bring in an article that discusses any of the topics covered in the chapter. In groups or in class, they could share their example and discuss the implications. Alternatively, this could serve as a written assignment if they were asked to write a summary of the article and to point out the implications for HRM.



2.
Assign students to read The Wall Street Journal article "Bob Stadler Has Lived All the Business Trends of the Past 50 Years," July 11, 1996, p. 1. Have students research recent related articles.



a.
Ask them to discuss the changes in employee expectations regarding work.



b.
How must employers respond to some of these changes?



c.
If your class contains students with work experience, it might be useful to ask them what types of quality improvement programs they have seen in their companies and what type of employee reactions occurred when the programs were implemented.




3.
Outside Speaker: In the first week or two of class, it is frequently useful to invite a CEO of a small firm or an HRM vice-president of a larger firm to talk with the class about the role of human resources in the success of the company. Very often, such speakers have good examples of how the knowledge of HRM has helped the company to deal effectively with competitive pressures and environmental change.

该用户从未签到

 楼主| 发表于 2010-10-3 20:09:11 | 显示全部楼层
Chapter Summary



This chapter describes the concept of strategy and develops the strategic management process. The levels of integration between the HRM function and the strategic management process during the strategy formulation stage are then discussed. A number of common strategic models are reviewed, and, within the context of these models, types of employee skills, behaviors, and attitudes are noted. Ways in which HRM practices aid the firm in implementing its strategic plan are described. Finally, a model that views the HRM function as a separate business within a given firm is presented, making it easier for the student to understand the need for strategic thinking among HRM practitioners.



Learning Objectives



After studying this chapter, the student should be able to:

1.         Describe the differences between strategy formulation and strategy implementation.

2.         List the components of the strategic management process.

3.         Discuss the role of the HR function in strategy formulation.

4.         Describe the linkages between HR and strategy formulation.

5.         Discuss the more popular typologies of generic strategies and the various HR practices associated with each.

6.         Describe the different HR issues and practices associated with various directional strategies.

7.         List the competencies the HR executive needs to become a strategic partner in the company.



Extended Chapter Outline

Note: Key terms are boldface and are listed in the "Chapter Vocabulary" section.



Opening Vignette: Flying High at Jet Blue



This vignette describes the success of JetBlue Airways despite recent financial problems in the airline industry.  The company plans a strategy for growth, but this is a risky undertaking.  As JetBlue handles the challenges of growth, it must continue to preserve its personal touch.  



I.          Introduction —The JetBlue example illustrated that business organizations exist in an environment of competition.  The goal of strategic management in an organization is to deploy and allocate resources in ways that provide a competitive advantage. To be maximally effective, the HRM function must be inte­grally involved in the company's strategic management process.







II.         What Is Strategic Management?



A.        Strategic Management is a process for analyzing a company's competitive situation, developing the company's strategic goals, and devising a plan of action and allocation of resources (human, organizational, and physical) that will increase the likelihood of achieving those goals.



B.         Strategic human resource manage­ment is the pattern of planned human resource deployments and activ­ities intended to enable an organization to achieve its goals.



C.        Components of the Strategic Management Process—There are two distinct phases of this process (Figure 2.1 in the text).



1.         Strategy Formulation: During this phase, strategic planning groups decide on a strategic direction by defining the company's mission and goals, its external opportunities and threats, and its internal strengths and weaknesses.



2.         Strategy Implementation: During this phase, the organization follows through on the strategy that has been chosen. This includes structuring the organization, allocating resources, ensuring that the firm has skilled employees in place, and devel­oping reward systems that align employee behavior with the strategic goals.



D.        Linkage between HRM and the Strategic Management Process: The strategic choice really consists of answering questions about competition.  These decisions consist of addressing the issues of where to compete, how to compete, and with what to compete (See Figure 2.2).



E.         The Role of HRM in Strategy Formulation—Both strategy formula­tion and strategy implementation involve people‑related issues and therefore necessitate the involvement of the HR function. Four levels of integration exist between the HR function and the strategic management function, as shown in Figure 2.3 in the text).



1.         Administrative Linkage—This is the lowest level of integration, in which the HRM function's attention is focused on day‑to‑day activities. No input from the HRM function to the company's strategic plan is given.





2.         One‑Way Linkage—The firm's strategic business planning func­tion develops the plan and then informs the HRM function of the plan. HRM then helps in the implementation.



3.         Two‑Way Linkage—This linkage allows for consideration of human resource issues during the strategy formulation process. The HRM function is expected to provide input to potential strategic choices and then help implement the chosen option.



4.         Integrative Linkage—This is based on continuing, rather than sequential, interaction. The HR executive is an integral member of the strategic planning team.



III.       Strategy Formulation—This includes five major components (see Figure 2.4 in the text).



1.         A mission is a statement of the organization's reasons for being; it usually specifies the customers served, the needs satisfied and/or the value received by the customers, and the technology used.



2.         Goals are what the organization hopes to achieve in the medium‑ to long‑term future; they reflect how the mission will be operationalized.



3.         External analysis consists of examining the organization's operating environment to identify strategic opportunities and threats.



4.         Internal analysis attempts to identify the organization's strengths and weaknesses.



5.                   Strategic choice is the organization's strategy, which describes the ways the organization will attempt to fulfill its mission and achieve its long­ term goals.





Example: Delta Airlines and HRM’s Role in Strategy Formulation—Delta’s employees were so loyal to the company that in the 1980’s the employees pitched in a bought the airline a new plane. The “Leadership 7.5” program arguably got rid of Delta’s only competitive advantage. Ideas could have been generated to find a more effective way of cost cutting in an alternative strategy.



Competing Through Technology



After a Turnaround, Quality Problems Creep Up at Nissan



Despite a highly successful turnaround beginning in 1999, Nissan now feels considerable growing pains as quality problems on the Armada, Quest Minivan, and Titan full-sized pickup dragged down its quality rankings.  Many of the problems stem from Nissan’s Mississippi factory where consumers complained of poor workmanship.  The source of these problems seems to be Nissan’s “peddle to the metal” mentality since 1999.  Nissan lacked the engineering resources to check thoroughly for defects before cranking up the assembly lines.  The lack of sufficient engineering talent and inexperience of the assembly line workers represents a mismatch between the strategy and human resources of Hissan, which may cost it in the long run.



IV.       Strategy Implementation—Five variables determine success in strategy implementation (see Figure 2.5 in the text).



HR has responsibility for three of these: task, people, and reward systems. The role of the HRM function is one of (1) ensuring that the company has the number of appropriately skilled workers and (2) developing "control" systems that ensure that those employees contribute to goal achievement. This is accomplished through various HR practices (see text Figure 2.6 and TM 2.5).



A.        HRM Practices—The HR function has six "menus" of practices from which companies can choose to fit their strategic direction (see Table 2.2 in the text).



1.         Job analysis is the process of getting detailed information about jobs. Job design deals with making decisions about what tasks should be grouped into a particular job. Jobs can range from very narrow sets of tasks that demand a limited set of skills to a complex array of tasks that requires multiple, high‑level skills.Many jobs today are being broadened.





2.         Recruitment is the process through which the organization seeks applicants for employment. Selection refers to the process of identifying applicants with the appropriate knowledge, skills, and ability to help the company achieve its goals.



3.         Frequently, employees need new skills when jobs are modified. Training refers to a planned effort to facilitate learning of job-­related knowledge, skills, and behavior. Development involves the acquisition of knowledge, skills, and behavior that improves employees' ability to meet the challenges of a variety of existing jobs or jobs that do not yet exist. TQM programs require exten­sive training of employees.



4.         Performance management is used to ensure that employee activities and outcomes are congruent with the organization’s objectives.



5.         Pay structure, incentives, and benefits have an important role in implementing strategies. High pay levels help to attract and retain high‑quality employees. Performance‑based pay plans help motivate appropriate performance. The pay system includes the base pay as well as incentives and benefits.



6.        Labor and employee relations refer to the general approach the company takes in interacting with its employees, whether unionized or not. Companies can choose to treat employees as assets, resources to be invested in for the long term.





B.         Strategic Types—Several different "typologies" of strategies exist.



Porter's Generic Strategies—Michael Porter has hypothesized that competitive advantage comes from creating value by (1) reducing costs (overall cost leadership) or (2) charging a premium price for a differentiated product or service (differentiation).





C.        HR Needs in Strategic Types—Different strategies require different types of employees with different skills and also require employees to exhibit different "role behaviors." Role behaviors are the behav­iors required of an individual in his or her role as a jobholder in a social work environment.



1.         Cost strategy firms seek efficiency and therefore carefully define the skills they need in employees and use worker participation to seek cost‑saving ideas.



2.         Differentiation firms need creative risk takers.



D.        Directional Strategies—Five types follow. The human resource implications of each of these strategies are quite different:



1.         External growth strategies include vertical and horizontal integra­tion as well as diversification.



2.         Concentration strategies focus a company on what it does best in its established markets.



3.         Internal growth strategies include market development, product development, innovation, or joint ventures.



4.         Mergers and Acquisitions include consolidation within industries and mergers across industries.



Competing Through Globalization



Questioning the Value of Global Megamergers



Mergers are more frequently consisting of global mega-mergers despite some warnings that these might not be effective.



An exploration into the rationales and effectiveness of mega-mergers is provided in this case scenario.  





5.         Downsizing is the planned elimination of large numbers of personnel, designed to enhance organizational effectiveness.  Table 2.3 lists some of the major company downsizings, while Table 2.4 contains the results of a survey that indicates that only about one‑third of the companies that went through downsizings actually achieved their goal to increase produc­tivity.



Downsizing presents a number of challenges and opportunities for HRM: careful reduction of the work force, boosting the morale of employees who remain, increasing fresh ideas, and changing the company's culture.



Example: Delta Air Lines and HRM’s Role in Strategy Implementation—HRM seemed to play a significant role in Delta’s “Leadership 7.5” strategy’s implementation.  Costs were cut $1.6 billion through workforce reductions because labor costs are the largest single controllable cost for an airline.  The negative outcome of the strategy in terms of employee morale, customer service, and operational performance have stemmed from the strategy itself, but one wonders if the same strategy might have been implemented differently with less negative impact.



Competing Through Globalization



Offshoring and the Future of Work



Numerous U.S. manufacturing jobs are being sent overseas to lower wage countries.  However, advances in telecommunications and information technology have resulted in more “white collar” jobs being sent overseas as well. Critics of offshoring suggest that moving jobs overseas will result in a net loss of jobs in the U.S., and will depress U.S. wages and undermine the employment base.  Advocates argue that multinational companies can take advantage of high skills and lower wages.  The trend towards offshoring is expected to continue and grow.



E.         Strategy Evaluation and Control—This is the final component of the strategic management process that includes the monitoring of the effectiveness of strategic choice and implementation.





V.        The Role of Human Resources in Providing Strategic Competitive Advantage



A.        Emergent Strategies—Those that evolve from the grass roots of the organization: that is, what actually is done versus what is planned (see text Figure 2.1). HR plays an important role in facilitating the communication of emergent strategies between levels in the hierarchy.



Example: When Compaq Computer was founded, building compact computers that were free of defects was its intended strategy. In 1992, after a difficult period of price competition, Compaq's strategy was changed to being a low‑cost producer.



B.         Enhancing Firm Competitiveness—By developing a rich pool of talent, HR can assure the company's ability to adapt to a dynamic environment.



C.        Strategic Human Resource Executives—The increasing importance of these roles demands new skills. Four basic competencies are illustrated in the text Figure 2.7.



1.         Business Competencies—Understanding the company's economic and financial capabilities.



2.         Professional/Technical Knowledge—In HR practices such as selection techniques and compensation systems.



3.         Change Processes or Organizational Development Techniques— The ability to diagnose the need for change and develop and implement the appropriate intervention.



4.         Integration Competencies—A generalist perspective with the skills of a specialist in the above three areas.





A Look Back


The opening vignette on JetBlue shows that despite current profitability within the company, JetBlue faces many challenges in its quest for growth.  



Questions



1.         How successful do you think JetBlue will be in maintaining its culture while it quadruples in size over the next few years?



Students’ answers may vary, but could include the following.  Although JetBlue is currently a small airline, there is one large airline in the industry that is also experiencing higher than normal rates of return: Southwest.  Some students may suggest that JetBlue follow the strategy of Southwest.  Other students may state that increasing the size and workforce of the company quickly may lead to quality problems within the labor force, which could adversely affect the company’s culture.   



2.         What must JetBlue do with regard to aligning its strategy and its people that it does not already seem to be doing?



Students’ answer may vary, but could include the following.  A primary focus of HRM should be on the highly committed workforce. Employees’ participation should be sought to empower workers and make them feel like they play a critical role in the company’s strategy.  The company should reestablish its fun-loving culture and emphasize this culture as it employs more and more workers and increases its market share.  





Chapter Vocabulary



These terms are defined in the "Extended Chapter Outline" section.



Strategic Human Resource Management

Strategy Formulation

Strategy Implementation

Goals

External Analysis

Internal Analysis

Strategic Choice

Job Analysis

Job Design

Recruitment

Selection

Training

Development

Performance Management

Role Behaviors

External Growth Strategy

Concentration Strategy

Internal Growth Strategy

Downsizing





Discussion Questions



1.         Pick one of your university's major sports teams (e.g., football or basketball). How would you characterize the team's generic strategy? How does the composition of the team members (in terms of size, speed, ability, etc.) relate to that strategy? What are the strengths and weaknesses of the team? How do these dictate the team's generic strategy and its approach to a particular game?



This question ought to generate some interesting discussion. Let's use football at the University of Michigan as an example. This team's generic strategy is probably best characterized as a focus strategy—development of the ground game. They have a very large offensive line and big, fast running backs. Additionally, they have a large fullback who is able to successfully block for the tailback.

Michigan's strengths are really the quality of (1) the offensive line and (2) the running backs (speed and size). The primary weakness is the lack of a well‑developed passing game, which leaves them in a difficult position when they get behind as time is running out. (Students from schools where the football teams are not undefeated will be able to list a greater variety of weaknesses.)



The basic Michigan strategy is to run the football, score points, keep the clock moving, and, with their skilled defense, keep the ball out of their opponents' hands. The strengths clearly support this strategy, and yet their weakness limits their options in more difficult games (for example, last season's three tie games).



2.         Do you think that it is easier to tie human resources to the strategic management process in large or in small organizations? Why?



Most likely, it is easier to integrate HR and the strategic management process in a medium to large organization due to two factors: first, larger organizations tend to have a more formalized approach to strategic planning, while many small firms do not plan; and second, larger firms generally invest time in standardizing or centralizing HR policies and practices. In smaller firms, decisions are made less frequently and may have more of an ad hoc nature.





3.         Consider some of the organizations you have been affiliated with. What are some examples of human resource practices that were consistent with that organization's strategy? What are examples of practices that were inconsistent with its strategy?

Get students to talk about student organizations or former employers. Talk about the different strategies, categorize them, and then discuss the various policies and how they fit.



Student answers will vary based on the industry and job they select.  



4.         How can strategic management within the HRM department ensure that HRM plays an effective role in the company's strategic management process?



By instituting strategic management within the HRM department, the department will be forced to review their mission, customers, and so forth. In doing so, it will become evident that for HRM to fulfill its mission, the function must take a role in overall corporate planning and implementation.



5.         What types of specific skills (e.g., knowledge of financial accounting methods) do you think HR professionals will need in order to have the business, professional/technical, management, and integrative competencies necessary in the future? Where can you seek to develop each of these skills?



HR professionals will have to have a basic understanding of the various business functions. More specifically, knowledge of statistics, accounting, information systems, and, obviously, the specific functions of HR are critical to future HR practice. Additionally an in‑depth knowledge of organizational change methods will enable an HR executive to take a leadership role in setting the direction of the organization as well as in implementing changes called for by the strategy of choice. These skills can be learned in colleges of business and through experience gained within an organization.





6.         What are some of the key environmental variables that you see changing in the business world today? What impact will those changes have on the HRM function in organizations?



Students can list many different general and/or specific changes that will be affecting firms and their HR practices. For example the anticipated national health‑care program will result in changes to HR benefit structure and will affect the firm as a new tax. NAFTA will encourage U.S. firms to actively move into both the Mexican and Canadian markets, if they haven't already. In some cases, this may mean opening plants in these countries. Both countries have different employment legislation and will therefore require separate HR policies or practices. The changes in demographics in the United States (fewer entry‑level workers, less well‑educated entry level employees, more older workers, more women, more people of color. etc.) will have many and varied impacts on HR practice. The key is to push students to think about what types of impacts they will hate on HR and how HR can respond in an effective manner



Manager’s Hot Seat Exercise: Privacy: Burned by the Firewall




I.  Introduction



The interaction in this scenario lends itself to a rich discussion of the role of Human Resources and their interaction with line management.  Issues related to email privacy and appropriate disciplinary actions given certain situations are brought to the forefront.  For those seeking careers in HR, this scenario provides an example of what not to do to be a strategic business partner.



II.  Learning Objectives



To assess students’ understanding of issues with company property use and the appropriate disciplinary process when company policies are violated.   
To analyze the role of HR in enforcing company policies.  


III.  Scenario Description:



Overview:  An employee, Willy Kushing, has been put on administrative leave by the HR department for misuse of company property [internet service and telephone].  His Manager is just returning from vacation and had no prior notice that Willy would be put on leave.  The Manager goes to meet with HR to find out what has happened.





Profile:

·        Lynn Couchara is the Director of Distribution for Bank Street Films, a film distribution house.  Lynn manages a team of 30 people and is responsible for the film distribution in 40 states, distributing over 75 films per year.  

·        Janet Pierce is the Director of Human Resources and has been working with Bank Street Films for eight years and was promoted to Director five years ago.  Since becoming Director, Janet has restructured her department and revamped many policies at the company.  Janet has known Lynn since she hired her six years ago.  Janet was not involved in the hiring of William Kushing.  



References:  The references included in the DVD are:

·        EPolicy Do’s and Don’ts (PPT 4-3 to 4-6)

·        The Electronic Communications Privacy Act of 1986 (PPT 4-8)

·        2003 E-Mail Rules, Policies and Practices Survey (PPT 4-10)



Back History:  Willy Kushing has been working at Bank Street for just over two years [recruited from Bank Street’s #2 competitor]. He has proven to be a vital employee for Couchara, handling the largest accounts in the distribution department as well as being the ‘go-to man’ for many others in the office when they are in need of advice, support or help.  



While Couchara was on her annual two-week vacation, Kushing was called for an immediate meeting with the head of HR, Janet Pierce.  Pierce informed Kushing that he was being put on administrative leave for misuse of computer property, effective immediately.  HR had reviewed all of his internet and phone usage, as is their right to do, and found numerous and regular abuses.  There were “constant’ calls being made to Iowa, to a number having no business relation to Bank Street Films.  In addition semi-regular calls have been made to Gloucester Massachusetts and Schenectady, New York.  The internet abuses consisted of many hours logged on various news and auction sites, that also fall outside the realm of Bank Street business.



Kushing had explained to HR that he’d been making more personal calls from work to his mother in Iowa and siblings in Gloucester and Schenectady because his father has been seriously ill for months, in and out of emergency rooms and hospitals.  He assured them these calls had not interfered with his ability to do his job and that his performance had been consistent and solid.  This information was deemed to be irrelevant by Pierce, and Kushing was sent to tie up loose ends and prepare for his leave.



Scene Set-up: Couchara has made an appointment with Pierce to discuss the Kushing situation.





Scene Location: Pierce’s office, HR department, Bank Street Films, Inc, Monday 11am



The Meeting - Summary:  Lynn Couchara meets with Janet Pierce to discuss the surprising punitive actions taken against one of her employees, Willy Kushing.  Lynn is very concerned that this action was taken while she was on vacation and without notifying or discussing the situation with her.  She also values Willy’s contributions very much and notes that he has had no performance issues.  Janet defends her actions based on the company’s policies and says that it was merely coincidence that this happened when Lynn was on vacation and that she was very sorry for that.  When Lynn tries to suggest that Willy be reinstated, Janet becomes defensive and then goes on to suggest that Lynn has also misused company property.



Lynn wants to resolve this issue by having Willy, a key employee, return to work and suggests going over Janet’s head to the VP of HR if she needs to.  The situation remains unresolved at the end of the scenario.



Afterthoughts – Summary:  Lynn does not think the meeting went well because the HR Director did not understand her point of view.  She notes frustration with the way the HR enforced the company’s policies absent of management involvement.  Specifically, she is upset that she was not consulted or informed about the pattern of misuse or the punitive action.  She also doesn’t agree that email or phone use should be entirely forbidden for personal use if it doesn’t impact productivity or the budget.  She plans to go to Janet’s boss with the intent of explaining her feelings about the way this was handled and to figure out how to get Willy reinstated.



Dossier:  The specific artifacts included in the DVD are:

1.      Excerpts from Kushings’ computer/telephone usage report

2.      Excerpts from Coucharas’ computer/telephone usage report

3.      Company Policy on use of company property





IV.  Discussion Questions:



The References and related Discussion Questions may be found in PowerPoint slides 4-1 to 4-10 on the Instructor’s side of the text’s Website.



Learning Objective #1: To assess students’ understanding of issues with company property use and the appropriate disciplinary process when company policies are violated.



Lynn is shocked by the action taken by HR and does not feel the violation of company policy warranted the punitive action that was taken.  In the long-term, what actions does the company need to take to prevent this situation from occurring in the future (see PPT 4-3 to 4-6)?


Training employee’s to ensure that they understand the company’s policies and the penalty for violating them is imperative.  Simply signing the employee handbook is not enough to ensure that company resources are not wasted on non-business-related matters.



´        1.  Lynn’s key concern should be:

A.     HR Protocol

B.     Workflow

C.     Willy’s wrongdoing



Lynn’s key concern seems to be workflow because she continually mentions how important Willy is to her department.  Therefore, her focus is to reinstate Willy so that the workflow is not disrupted further.  She is interested in Willy’s wrongdoing to understand why such drastic measures were taken without her input and to attempt to refute the action on the basis that it was not warranted.



The ePolicy Do’s and Don’ts suggest that some personal email and internet use may be tolerated by U.S. companies.  What are the pro’s and con’s of this approach?


Pro’s – don’t have to worry about invasion of privacy when monitoring electronic systems, less risk of viruses, less use of the system’s resources because fewer emails and attachments to handle

Con’s – employees who are expected to work long hours do not get the chance to communicate with friends and family or do occasional “errands” on-line.  This can cause them stress and potential dissatisfaction with the organization.  Employee privacy becomes a concern when monitoring the system.





´        2.  The concern over phone [and internet] use is:

A.     Budgetary

B.     Performance related

C.     Misuse of property



Janet’s concern over the phone use seems to be that it is in violation of the  policy on the appropriate use of company property.  She did not present any data on the budgetary or performance impact of Willy’s actions.



How do the stipulations provided in the Electronic Communications Privacy Act of 1986 (see PPT 4-8) affect your view of personal email and internet use at work?


If companies allow the use of the internet and email for personal reasons, then the company will likely monitor those communications to prevent viruses and to detect excessive use or draining of bandwidth, etc.  Therefore, privacy becomes a key concern.  However, if companies don’t allow any personal use of email or the internet, then privacy becomes less of a concern when monitoring the system, but satisfaction of employees may suffer.



Companies create disciplinary processes to meet their business needs.  What is your reaction to the disciplinary process that occurred in this scenario?


Typically, organizations have a disciplinary process that is gradual and calls for progressively increasing punitive action unless there is an extremely grievous act (e.g., possession of a weapon, embezzlement, etc.).  In this case, Willy’s records were being monitored but no action was taken until his usage of company property “reached a threshold”.  Counseling and warning the employee (the typical first steps in a progressive discipline process) were skipped.  Once the threshold was passed, serious punitive action was taken.  Allowing the employee to explain his situation and being sure that he/she understands the consequences of their actions serves to prevent misuse and retain valuable employees.



´        4.  What justifies Janet’s decision?

A.     Existence of policy

B.     Fair warning given

C.     Nothing



Technically, the existence of the policy justifies Janet’s decision.  In reality, however, her action not only caused harm to a valuable employee, it also damaged the relationship with line management whom HR is there to serve.





Learning Objective #2: To analyze the role of HR in enforcing company policies.



1.        Evaluate the actions taken by the HR director in this scenario.  What was done well, poorly?



Janet apologized to Lynn for her actions, but beyond that the HR Director’s actions did not take into consideration the big picture or the organization’s business needs.  While violating the company policy may impact the organization’s bottom line, putting the employee on leave without a fair warning will likely be perceived as unfair to the employee and cause him to be disgruntled.  HR’s role should be to provide advice and guidance to line management on disciplinary actions – not take them themselves.  Thus, not including Lynn in the decision was a mistake and will damage Janet’s relationship with Lynn.



´        3.  Why does Janet bring up Lynn’s records?

A.     To warn Lynn

B.     Enforce the policy

C.     Intimidation



Bringing up Lynn’s records was inappropriate in this context.  If there was a valid concern, then Lynn should be counseled about this issue and given fair warning.  Bringing this up to prevent Lynn from going over Janet’s head is a form of intimidation.  This behavior indicates to Lynn that Janet’s interests are not aligned with achieving the organization’s goals, but rather are more self-interested.  Janet’s actions will not effectively support line management in achieving the organization’s goals.



´        5. Lynn’s next step should be:

A.     Meet VP of HR

B.     Reinstate Willy herself

C.     Meet Janet again



From the interaction in the scenario, it doesn’t appear that Janet is willing to work with Lynn to reinstate Willy.  Also, the disciplinary process that was taken and it’s impact on line management should probably be communicated to the VP of HR. However, going over the HR Director’s head may not be appropriate depending on the organization’s culture.  Perhaps Lynn could wait a day and try to work with Janet again before approaching the VP of HR.  Reinstating Willy herself would make the organization appear fragmented and to be “stepping on each other’s toes”.







Exercising Strategy



Strategy and HRM at Delta Airlines



Questions



1.         How does the experience of Delta Air Lines illustrate the interdependence between strategic decisions of “how to compete” and “with what to compete?”  Consider this with regard to both strategy formulation and strategy implementation.



Students’ answers may vary, but could include the following.  Delta Air Lines show the interdependence between “how to compete” and “with what to compete” by getting rid of its one source of sustainable competitive advantage, its employees.  The strategy did reduce costs, but it also lost its employees.  The company replaced its previous employees with outside contractors and less experienced employees who do not do the jobs as well as the previous ones did.



2.         If you were in charge of HRM for Delta Air Lines now, what would be your major priorities?



Students’ answer may vary, but could include the following.  As head of the HRM department for Delta Air Lines, one’s priorities should be focused on the highly committed workforce.  Since these employees had the incentive to buy a plane to help the company, they may have some good ideas on how to run the airline more efficiently and effectively.  Employees’ participation can be sought on what to change to help the company become a “well oiled machine.”  As the book states, the employees may even have ideas on “how to make any necessary workforce reductions and perhaps would be willing to take temporary pay cuts to help the firm get back on its feet.”  Since the union is now involved, there would have to be discussions and contracts written up between Delta Air Lines and the union, if the pay cuts were to occur or any other decisions were to be made.

   





Managing People



Reprogramming Amazon



Questions



1.         Is the strategy described in this article a change in strategy for Amazon?  Is it a change in where to compete, how to compete, or with what to compete?



            Student answers will vary.  Most will say that Amazon’s plans are a change in strategy, because rather than just selling books online, they are now going to offer software and computer services that are linked to book selling.  This is not a change in where the company competes, because is is still competing on the Internet.  It is a change in how the company competes, and adds new competitors like Microsoft to the list.  It is a change in with what the company competes, because it is offering more products and services.   



2.         What are the HR implications for Amazon’s transformation to a technology company?



            Student answers will vary.  The article stated that Amazon is preparing to hire hundreds of software engineers and computer scientists in the next year to facilitate this new strategy.  For the HR department, this means increased recruiting, selecting, and training.  It also means providing adequate compensation and rewards, because these positions generally garner high wages.  The HR department will be busy in the coming year in helping new and old employees alike to understand the new strategies and culture the company is pursuing.  Overall, the HR department will be very busy because of these projected changes.



Additional Activities



Teaching Suggestions



Students who have not yet had a course in business strategy will find this material challenging and new. Therefore, any exercises that ask the student to identify various strategies and discuss HRM implications should be helpful. Following are suggestions of activities that provide this type of learning activity. The HBR case, People Express, can be used in a whole class period, most effectively in small group discussion initially.  



The Frost, Inc. case is simpler and could be given as homework assignment and thenused in class discussion. An outside speaker is suggested, if the time and person is available.





A.        Harvard Business School Case 9‑490‑012, People Express Airlines: Rise and Decline by M. Beer, Teaching Note 5‑491‑080 by Beer.

(Two case supplements have also been developed: People Express Supplement, 9‑487‑054. by C. Heckscher, and People Express, Update— January 1989, 9‑489‑022, by D. Q. Mills and G. B. Friesen.)



This case describes the innovative approach to organizing and managing employees by People Express and describes the company's eventual demise. This material can be used to inform about leading‑edge human resource practices and to raise questions about what went wrong with the organi­zation. Two videotapes (9‑890‑507 and 9‑890‑508) are available as well.



Suggested Questions for "People Express"



1.         How did People Express business strategy relate to its HR strategy? Was the business strategy always closely linked to the HR strategy?



1.                  Would the People Express HR system remained effective if it had chosen to remain in the low‑density routes dominated by local and regional carriers? Why? What aspects of the People Express HR system needed to be modified if it was going to successfully compete with the major carriers?



2.                  What factors do you believe were most responsible for the success of People Express from 1981 to 1985? In comparison to other factors, how important was the management of human resources for People Express's success between 1981 and 1985?



4.         The following case describes an organization's attempt to modify its strategy and problems that result from the attempted change. Questions for students to discuss or respond to in writing follow the case.





B.         Frost, Inc.

(Raymond Noe, Michigan State University)



Frost. Inc. is a manufacturer of overhead conveyor trolleys, used primarily in the auto industry. With sales of $20 million, Frost is located in Grand Rapids. Michigan. Concerned about depending too heavily on one cyclical industry, Frost's president made several attempts to diversify the business. The attempts to move into manufacturing lawn mower components and material handling systems, such as floor conveyors and hoists, failed. The engineers did not know how to design unfamiliar components, production people did not know how to make them, and sales people did not know how to sell them.



The president diagnosed the problem as inflexibility ("We have single‑purpose machines and single‑purpose people, including single‑purpose managers."). He decided that automating production was the key to flexibility. Twenty‑six old‑fashioned screw machines on the factory floor were replaced with 11 computer-numerically controlled machines and 18 robots. He decided to build an automated store and retrieval inventory‑control system and to completely automate the front office to reduce indirect labor costs. He did not plan to approve additional hires as a result of the automation.



Questions



1.         What directional strategy was Frost pursuing?



Frost initially pursued a concentration strategy and then changed to an external‑growth strategy—diversification.



2.         What HR practices should be in place to support this strategy?



In order to successfully pursue a strategy of diversification, people with the needed skills must be hired or developed. Therefore, as the company moved into a new area, engineers with experience in lawn mower components, etc., would need to be hired, or current employees would need to be provided with training. In this case, the CEO had decided that no new hiring would take place. Therefore, the emphasis should be on the provision of training for new skills. To support the engineers and production people in this development effort, a skill‑based compensation program along with profit‑sharing would be useful.





3.         What types of information should the HR function provide to support the new strategy?



One of the unstated problems in this case may be with employee attitudes toward the change, given that it appears that they did nor participate in the strategic choice. A survey of attitudes may be useful. Secondly, a survey of employee skills and experience would help make staffing and training decisions more effective. While automation may resolve some production problems, it doesn't solve the design­ engineering issues. Additionally, even with automated lines, people must be available who have the skills to monitor production activity and who can maintain and repair the equipment. Once again, HR could provide the human side of the automation process so that planning for the change process would include all necessary elements for a successful corporate culture change.

With the need for higher skills and potentially for team organization, the HRM function would need to determine the aspects of the organization's practices that no longer fit the new technology and structure of the firm.

(See the following reference: "Frost Inc.—Technological Renewal and Human Resource Management: A Case Study," Steven P. Galante, Human Resource Planning 10, no. 1, pp. 57‑66.)



C.        Outside Speaker: An outside speaker for this chapter could be used very effectively. Identify an HRM or organizational development practitioner who has recently participated in the implementation of a TQM or other quality process—or any kind of organizational restructuring. Ask this indi­vidual tO describe the precipitating events leading up to the change and the people issues that were visible during the implementation.

Business literature is filled with examples of businesses that are reevaluating their strategies and making significant changes. Ask students to review recent issues of Business Week or Fortune and to bring in an example that they can discuss in groups or share with the class as a whole. It may be that these examples will readily fall into strategic categories that can then be used to reinforce the chapter concepts.

该用户从未签到

 楼主| 发表于 2010-10-3 20:10:12 | 显示全部楼层
Chapter Summary


The first section of this chapter discusses the analysis of work process within a given work unit. Having provided an understanding of the broader context of jobs, the chapter discusses the need for and usefulness of both job analysis and the techniques for performing job analysis. Finally, the chapter concludes by presenting the various approaches to job design to provide managers with an understanding of the costs and benefits of emphasizing different characteristics of jobs when designing or redesigning them.



Learning Objectives


After studying this chapter, the student should be able to:



1.         Analyze a work-flow process, identifying the output, activities, and inputs in the production of a product or service.

2.         Understand the importance of job analysis in strategic and human resource management.

3.         Choose the right job-analysis technique for a variety of human resource activities.

4.         Identify the tasks performed and the skills required in a given job.

5.         Understand the different approaches to job design.

6.         Comprehend the trade-offs among the various approaches to designing jobs.



Extended Chapter Outline


Note: Key terms appear in boldface and are listed in the "Chapter Vocabulary" section.



Opening Vignette: Structural Realignment at Microsoft: Opening New Windows of Opportunity


In the 1990s, revenue growth was high for Microsoft.  Since then, Microsoft’s dominance has been reduced due to competition and external and internal pressures.  One of the problems at Microsoft was that the decision-making process was slow.  In addition, turnover was higher than expected because key employees lacked intrinsic motivation.  





To turn this situation around, the company’s CEO restructured the organization to respond to these new competitive pressures.  It became clear that Microsoft was too centralized given its size, and too much decision-making power rested with the CEO and founder.  CEO Bill Balmer took steps to decentralize the organization and create semiautonomous business divisions, but founder Bill Gates resisted.  Balmer and Gates found that the solution was to create a matrix like organization structure that relied on seven autonomous divisions.  These divisions divided the work up into separate units for operating systems, desktop applications, business services, servers systems, mobil devices, Internet services, and gaming applications.  



This new system revealed ho much money was being lost in certain divisions relative to more profitable divisions.  This provided a benchmark from which to measure improvement.  This new structure also motivated individuals to sink or swim in their new, more autonomous roles.  This increased intrinsic motivation and in turn reduced turnover rates.  Many Microsoft managers now believe that this new type of attitude will propel Microsoft back to the lofty rates of growth that it once enjoyed.   



I.                    Introduction—Designing the work to be performed is one of the first tasks of strategy implementation discussed in Chapter 2. The way a firm competes can have a profound impact on the way tasks are organized, and the way the tasks are designed may provide the company with a competitive advantage. Also, the way jobs are designed can, in fact, affect company work‑unit performance. There is no “one best way” to design jobs and structure organizations.  The organization needs to create a fit between its environment, its competitive strategy and philosophy on the one hand, with its job and organizational design on the other.  Job analysis and job design are interre­lated.



  II.       Work‑Flow Analysis and Organization Structure (Workflow analysis, analyzing work outputs, processes, and inputs; see text Figure 4.1)  



-         work-flow design – the process of analyzing the tasks necessary for the production of a product or service, prior to allocating and assigning these tasks to a particular job category or person.

-         organization structure – the relatively stable and formal network of vertical and horizontal interconnections among jobs that constitute the organization.

            

A.        Work-flow Analysis



The workflow process is useful because it provides a means for the managers to understand all the tasks required to produce a high-quality product as well as the skills necessary to perform those tasks.





B.         Analyzing Work Outputs



1.         Work outputs are products of, or services provided by, a work unit.



Example: A work output for GM is a new Buick off the assembly line; a work output for Gordon Landscaping Company is a mowed lawn.



2.         Once outputs have been identified, it is necessary to specify the standards for the quantity or quality of these outputs.



3.         ProMES (productivity measurement and evaluation system) is a productivity improvement technique that focuses attention on both identifying work‑unit outputs and specifying levels of required performance for different levels of efficiency.



C.        Analyzing Work Processes



1.         Work processes are the activities that members of a work unit engage into

produce a given output.



Example: Work processes needed to produce an automobile include assembly, painting, and so forth.

Every process consists of operating procedures that specify how things should be done to develop the product or service.



D.        Analyzing Work Inputs



1.         Work inputs are the "ingredients" that go into the work processes and can be broken down into three categories (text Figure 4.1).



a.                   Raw materials consist of the materials, data, and informa­tion that will be converted into the work unit's products.





b.                  Equipment refers to the technology, machinery, facilities, and systems necessary to transform the raw materials into the product or service.



Example: Raw materials for the assembly of automobiles include various parts (steering wheels, tires, door panels, etc.) and equipment used, including robotic welding machines.



c.                   Human skills refer to the worker's knowledge, skills, abil­ities, and efforts necessary to perform the tasks.



III.       Organization structure – provides a cross-sectional overview of the static relationship   between individuals and units that create the outputs.



A.        Two of the most important dimensions of structure are centralization and departmentation.



1.         Centralization is the degree to which authority resides at the top of the organizational chart.



2.         Departmentalization refers to the degree to which work units are grouped   based upon functional similarity or similarity of workflow.



B.         Two types of Structural Configuration of organizational structure tend to emerge in organizations:



1.         A functional structure (See Fig. 4.2 in the text) employs a functional departmentation scheme with high levels of centralization. Functional structures are very efficient.  However, they tend to be inflexible and insensitive to subtle differences across products, regions, or clients



2.         A divisional structure (see Figures 4.3, 4.4, 4.5 in the text) employs a workflow departmentation and low levels of centralization.  Because of their workflow focus, their semi-autonomous nature, and their proximity to a homogenous consumer base, divisional structures tend to be more flexible and innovative.  However, they are not very efficient.





C.        Structure and the Nature of Jobs



1.         Jobs in functional structure need to be narrow, highly specialized, and people need to work alone.



2.         Jobs in divisional structures need to be more holistic, team-based structure with greater decision making authority.



IV.       Job analysis – the process of getting detailed information about jobs.



            A.        The Importance of Job Analysis to HR Managers



1.         Job analysis has been called the building block of everything that the personnel department does.



2.         Some of the human resource activities that use job‑analysis information includes selection, performance appraisal, training and development, job evaluation, career planning, work redesign, and human resource planning.



-         Work redesign – a firm will seek to redesign work to make it more efficient or effective.

-         Human resource planning – planners analyze an organization’s human resource needs in a dynamic environment and develop activities that enable the firm to adapt to change.

-         Selection – identifying the most qualified applicants for employment.

-         Training – trainer identifies the tasks performed in the job.

-         Performance appraisal – getting information about how well each employee is performing.

-         Career planning – matching an individual’s skills and aspirations with opportunities that are or may become available in the organization.

-         Job evaluation – assessing the relative dollar value of each job to the organization to set up internally equitable pay structures.





Competing Through Globalization


Wanted: Jobs that do not travel well


As the example of the call-center industry in this case study shows, many industries are moving from highly developed countries like the U.S. and Germany, to countries with lower wages.  This has become a necessity of competition in our global society.  The U.S. is experiencing a lag in job growth, and this is affecting unemployment and the standard of living in the U.S.



B.         The Importance of Job Analysis to Line Managers



1.         Managers must have detailed information about all the jobs in their work group to understand the work‑flow process.



2.         Managers need to understand the job requirements to make intelligent hiring decisions.



3.         Since the manager is responsible for ensuring that each indi­vidual is performing his or her job satisfactorily, the manager must clearly understand the tasks required in every job.

            

Competing Through Sustainability


Clean Rooms: Clean in name only?


This case study demonstrates that there has been growing concern about the safety associated with working in “clean rooms” at IBM and other semiconductor chip manufacturers.  



Despite their name, clean rooms are filled with many potentially toxic substances.  Many workers feel that under the pressure to get the job done, shortcuts are taken that put the workers at risk.  The problem with clean rooms stems from a combination of lack of knowledge, need for innovation, and speed in manufacturing operations.





C.        Job Analysis Information



1.         A job description is a list of the tasks, duties, and responsibili­ties (TDRs) that the job entails. (Text Table 4.1)



2.         A job specification is a list of the knowledge, skills, abilities, and other characteristics (KSAOs) that an individual must have to perform the job.



Example: Job specifications for an employment assistant would include: (1) a four‑year college degree with major course work in human resources or an equivalent combination of experi­ence, education, and training; (2) considerable knowledge of principles of employee selection and assignment of personnel; (3) the ability to express ideas clearly in written and oral communication; (4) the ability to independently plan and orga­nize one's own activities.



3.         Sources of Job Analysis Information



a.                   In general, it will be useful for the manager to go to the job incumbents to get the most accurate information about what is actually done on the job.  However, the incumbents might exaggerate their job duties.



b.                  Managers should ask others familiar with the job, such as the supervisor, to look over any information received from the incumbents.



c.                   Research has shown greater agreement between supervisors and subordinates when rating general job duties than when rating specific tasks.  Also, incumbents may be the best source for accurate estimates of time spent on job tasks, but supervisors may be more accurate on the importance of job duties.



d.                  Research is somewhat inconclusive about the relationship between the performance level of the job analyst and the job-analysis information he or she provides, but recent research has shown that effective and ineffective managers tend to give the same job-analysis ratings despite their performance level.





D.        Job Analysis Methods



1.         Position Analysis Questionnaire (PAQ)



a.         The Position Analysis Questionnaire (PAQ) is a standardized job‑analysis questionnaire containing 194 items representing work behaviors, work conditions, or job characteristics that are generalizable across a wide variety of jobs.



b.         The 194 items are organized into six sections, and the job analyst is asked to rate each item on six scales. A computer program generates a job report based on the ratings.  The six sections are:



-         information input

-         mental processes

-         work output

-         relationships with other persons

-         job context

-         other characteristics



c.         Research has indicated that the PAQ measures 13 overall dimensions (text Table 4.2). Knowing the dimension scores provides some guidance regarding the types of abilities that are necessary to perform the job.



d.         One of the main problems with the PAQ is that it requires the reading level of a college graduate to complete the ques­tionnaire.



2.         Task Analysis Inventory



a.         The task analysis inventory method refers to several different methods that focus on analyzing all the tasks performed in the focal job. It is not uncommon to have over 100 tasks for a job.





b.         The task inventory‑CODAP method has SMEs generate a list of tasks and then the SMEs rate each task on various dimensions such as the time spent on the task, frequency of task performance, relative importance of task, and relative difficulty of the task.



c.         The task inventories focus on providing detailed information about the work performed in a given job.  The detail of the information can be helpful in developing both selection exam plans and performance appraisal criteria.



3.         Fleishman Job Analysis System (FJAS)



a.         This approach defines abilities as enduring attributes of indi­viduals that account for differences in performance. The system is based on taxonomy of 52 cognitive, psychomotor, physical, and sensory abilities that adequately represent all the dimensions relevant to work (see Table 4.3 in the text).



b.         The FJAS scales include behavioral benchmark examples of the different levels of the ability along a seven‑point scale. SMEs indicate the point on the scale that best repre­sents the level of that ability required in a certain job (see Figure 4.6).



                        4.         The Occupational Information Network (O*NET)



a.         The U.S. Department of Labor replaced the Dictionary of Occupational Titles in 1998 with O*NET because jobs in the new economy were so qualitatively different from jobs in the old economy, and the DOT no longer served its purpose.



b.         Instead of relying on fixed job titles and narrow task descriptions, the O*NET uses a common language that generalizes across jobs to describe the abilities, work styles, work activities, and work context required for various occupations that are more broadly defined.



c.         Although it is still being developed, the O*NET is already being used by many employers and employment agencies.  It is also designed to help job seekers.

For more information on O*NET, visit http://online.onetcenter.org/






E.         Dynamic Elements of Job Analysis



Although we tend to view jobs as static and stable, in fact, jobs tend to change and evolve over time.  The job analysis process must also detect changes in the nature of jobs.  Advances in technology have made it hard to keep up with some of the major changes in jobs, and automation has led to the elimination of certain jobs or the offshoring of tasks or even shifting the tasks from the worker to the customer.



Competing Through Technology


If You Want the Job Done Right . . .


This study discusses the trend toward self-service.  For some organizations, the goal is to totally eliminate human involvement from certain transactions.  In the last few years, technological developments have allowed the shift from customer-service to self-service.  For example, self-service kiosks are becoming increasingly ubiquitous in today’s economy.  Self-service checkout is also becoming popular in grocery stores.  This trend is driven by the computer savvy of customers today.   

Airports are using kiosk technology so that customers can print tickets themselves and avoid long waits in line.  Because there are still glitches in these automated systems, some organizations think they can compete better by making customer service a warm and friendly experience.  Increasingly, it appears that many customer feel that there is no service like self-service.  





V.        Job design is the process of defining the way work will be performed and the tasks that will be required in a given job. Job redesign refers to changing the tasks or the way work is performed in an existing job. Jobs can also be characterized on different dimensions of job design (Table 4.4 in the text).



A.        Mechanistic Approach



1.         The mechanistic approach to job design has its roots in classical industrial engineering and focuses on designing jobs around the concepts of task specialization, skill simplification, and repetition.





2.         Scientific management, one of the earliest mechanistic approaches, sought to identify the one best way to perform the job through the use of time‑and‑motion studies.



3.         The scientific management approach was built upon in later years and resulted in a mechanistic approach that calls for the job to be designed very simply. The organization reduces its need for high‑ability individuals, and workers are easily replaceable (a new employee can be trained to perform the job quickly and inexpensively).



B.         Motivational Approach



1.         The motivational approach to job design focuses on the job characteristics that affect the psychological meaning and moti­vational potential, and it views attitudinal variables as the most important outcomes of job design.



2.         The prescriptions of the motivational approach focus on increasing job complexity through job enlargement, job enrich­ment, and the construction of jobs around sociotechnical systems.



3.         A model of how job design affects employee reaction is the “Job Characteristics Model”.   



a.         According to this model, jobs can be described in terms of five characteristics: skill variety, task identity, task significance, autonomy, and feed­back.



b.         These five job characteristics determine the motivating potential of a job by affecting three psychological states: experienced meaningfulness, responsibility, and knowledge of results.



c.         When the core job characteristics are high, individuals will have a high level of internal work motivation, higher quan­tity and quality of work, and higher levels of job satisfac­tion.





d.         Much of the work on job enlargement, job enrichment, and self‑managing work teams has its roots in the motivational approach to job design (Table 4.4 in the text). However, most of the research shows these interventions increase employee satisfaction and performance quality, but not necessarily increase quantity of performance.



Example: Alston & Bird of Atlanta have designed the paralegal job so that it entails a great deal of autonomy and clear communication channels between attorneys and paralegals.  The result:  In an industry where turnover among paralegals averages 20 percent per year, turnover at Alston & Bird averages 7 percent per year.  The firm also receives 15,000 applications for the 200 positions available per year.  



C.        Biological Approach



1.         The biological approach to job design comes primarily from the sciences of biomechanics (the study of body movements), and it is usually referred to as ergonomics, or the concern with examining the interface between individuals' physiological characteristics and the physical work environment. The goal of this approach is to minimize the physical strain on the worker by structuring the physical work environment around the way the body works.



Example: At Toyota's high‑tech Tahara No. 4 line, new electric vehicle carriers were installed to minimize stress on the workers' bodies. They adjust a car's height at every workstation. Toyota reports a major reduction in turnover during the plant's first year of operation.



2.         The biological approach focuses on outcomes such as physical fatigue, aches and pains, and health complaints.



3.         The biological approach has been applied in redesigning equip­ment to reduce the physical demands so women can perform the jobs and to reduce occupational illnesses such as carpal tunnel syndrome.




D.        Perceptual-Motor Approach


1.         The perceptual-motor approach to job design has its roots in the human‑factors literature and focuses on human mental capabilities and limitations. The goal is to design jobs in a way that ensures that they do not exceed people's mental capabilities.



2.         This approach generally tries to improve reliability, safety, and user reactions by designing jobs in a way that reduces the information processing requirements of the job.



3.         This approach, similar to the mechanistic approach, generally has the effect of decreasing the job's cognitive demands.



4.         Recent changes in technological capabilities hold the promise of helping to reduce job demands and errors, but in some cases, these developments have actually made the problem worse.  This is referred to as “absence presence”



            Example:  An example of absence presence is talking on a cell phone while driving a car.  



E.         Trade-offs among Different Approaches for Job Design (See Table 4.5 in text)



1.         One research study found job incumbents expressed higher satisfaction with jobs scoring highly on motivational approach.

However, the motivational and mechanistic approaches were negatively related, suggesting that designing jobs to maximize efficiency is likely to result in a lower motivational component to those jobs.



2.         Jobs redesigned to increase the motivating potential result in higher costs in terms of ability requirements, training, and compensation.



3.         In designing jobs, it is important to understand the trade‑offs inherent in focusing on one particular approach to job design.







A Look Back


The chapter opening of Microsoft showed how drastically restructuring the nature of work could increase both the effectiveness and efficiency of operations.  The specific changes in how work was designed created a better fit between the organization and its environment, as well as between the organization and its internal strategy.



Questions


1.         Based on this chapter, how would you characterize the changes that were made in terms of the degree of centralization and departmentalization?



Students’ answers may vary, but could include the following.  The changes that Microsoft made created a matrix like structure with increased decentralization to the degree that workers in seven newly established units had more autonomy.  The company was able to use this structure to determine which units were least profitable, and it was able to establish benchmarks for improvement.  This is expected to reduce turnover at Microsoft because workers are more intrinsically motivated.   



2.         What would be some characteristics of the environments or internal strategy that might force a different firm to move in the opposite structural direction?



Students could include the following in their answers:  dynamic, unstable environments with difficulty anticipating demands for resources, coordination requirements between jobs are not consistent over time, need for flexibility, and competition on differentiation or innovation are some characteristics that might force a firm to move in the opposite structural direction.



3.         How would each of these changes in structure “trickle down” and affect the jobs of individual workers?



Students’ answers will vary, but may include the following.  These changes could affect other employees because now if employees have questions, they could formally deal with the issue.  In the past, confused employees would send e-mails to Bill Gates.  Benchmarking in each department would encourage employees to improve and involve them directly in the process, increasing intrinsic motivation.





Chapter Vocabulary


These terms are defined in the "Extended Chapter Outline" section.



Centralization

Departmentation

Job Analysis

Job Description

Job Specification

Task Analysis Inventory

Job Design

Job Redesign

Ergonomics


Discussion Questions


1.         Assume you are the manager of a fast‑food restaurant. What are the outputs of your work unit? What are the activities required to produce those outputs? What are the inputs?



Some examples of outputs for a fast‑food restaurant include the food orders and the service provided. Activities required to produce these outputs include cooking, cleaning, preparing orders, taking orders, and so forth. The inputs include the raw materials (the ingredients for the food orders), the equipment (stove, cash register), and the human resources (the ability to cook, the knowledge of what ingredients go into a menu item).



2.         Based on question 1, consider the cashier's job. What are the outputs, activities, and inputs for that job?



Inputs for the cashier's job include the raw inputs (food ordered, prices, tax), equipment (cash register), and human resources (the skill to operate the register, the knowledge of the prices of the menu items, and the ability to answer customers' questions).



3.         Consider the "job" of college student. Perform a job analysis on this job. What are the tasks required in the job? What are the knowledge, skills, and abilities necessary to perform those tasks? What environmental trends or shocks (e.g., computers) might change the job, and how would that change the skill requirements?





Tasks would include attending class, completing homework assignments, and participating in-group assignments. Some examples of the knowledge, skills, and abilities needed are knowledge of prerequisite course material, college‑level reading skills, and ability to work together with others. Some environmental trends that might change the job would result from changes in the job market, such as new knowledge that employers would expect college students to learn. An example might be knowledge of sexual harassment guidelines or ADA legislation.



4.         Discuss how the following trends are changing the skill requirements for managerial jobs in the United States: (a) increasing use of computers, (b) increasing international competition, (c) increasing work‑family conflicts.



Students should have no trouble discussing how these trends are changing the skill requirements for managerial jobs in the United States. Managers are increasingly expected to be computer literate. Managers are also expected to be knowledgeable about other cultures, and knowledge of a second language is more commonly preferred. Managers are also expected to be more sensitive to work-family conflicts (day‑care and elder‑care issues for example) and to be knowledgeable about various legislation that deals with such situations (such as the FMLA).



5.         Why is it important for a manager to be able to conduct a job analysis? What are the negative outcomes that would result from not understanding the jobs of those reporting to the manager?



The chapter has a section on the importance of job analysis to both HR managers and line managers. The students' answers will probably reflect information in these sections as well as possible reasons of ­their own. The negative outcomes of a manager not understanding the jobs of his or her subordinates are that the manager may not make intelligent hiring decisions, may not be able to adequately evaluate the performance of subordinates, and will have trouble understanding the work‑flow process if individual jobs are not understood.





6.         What are the trade‑offs between the different approaches to job design? Which approach do you think should be weighted most heavily when designing jobs?



As discussed in the chapter, the trade‑offs appear to be between increased satisfaction and motivation and reduced efficiency due to increased costs. For example, the motivational approach that increases satisfaction results in lower utilization levels and increased training time. Table 4.5 in the text summarizes the positive and negative outcomes of each approach. Students' answers will vary as to which approach they think should be weighted most heavily depending on their value of the various outcomes for each approach.



7.         For the cashier in question 2, which approach to job design was most influential in designing that job?  In the context of the total work-flow process of the restaurant, how would you redesign the job to more heavily emphasize each of the other approaches?



Students' answers will vary. All of the approaches could be used to design the cashier's job. To redesign the job to emphasize the mechanistic approach, students should discuss concepts such as more specialization. To redesign the job to emphasize the motivational approach, students should discuss making the job more complex. To redesign the job to emphasize the biological approach, students should discuss adjusting or making changes in the equipment or job environment. To redesign the job to emphasize the perceptual/motor approach, students should discuss ways to make the job less demanding mentally.



Manager’s Hot Seat Exercise: Virtual Workplace: Out of Office Reply


I.  Introduction



Telecommuting is a flexible working option that many companies have embraced.  The advantages and disadvantages of telecommuting are highlighted in this scenario.  Instructors of Management or Human Resources may use this situation to discuss managing teleworkers and the need for clear and consistent policies.



II.  Learning Objectives



1.                  To assess students’ understanding of the challenges of telecommuting.

2.                  To analyze and evaluate managerial actions related to telecommuting.





III.  Scenario Description:



Overview:  Three months ago, Ralph Ramos assigned a number of employees to work as telecommuters to alleviate the lack of space in their office building.  Among them was Angela Zononi, an employee and friend for over four years, who was delighted to work from home since her commute to the office was particularly time-consuming.  Although things went relatively smoothly for the first six weeks, since then communication and performance have taken a steady downturn.  Angela has biweekly meetings with Ralph in his office.  Lately they have had unprecedented arguments and frequent misunderstandings.





Profile:

·        Ralph Ramos is the Senior Manager of Claims, managing a 75 employee department, at Saber Union Insurance, an international insurance company.

·        Angela Zanoni is a Claims Investigator at Saber Union.  She has handled very large accounts, including insurance fraud cases, resulting in multi-million dollar recoveries for her company.



References:  The references included in the DVD are:

·                    Advantages and Challenges for Employees (PPT 12-3)

·                    Advantages and Challenges for Employer (PPT 12-5)

·                    Telework Assessment Tool (PPT 12-7)

·                    Supervisor Checklist for Telecommuters (PPT 12-9)



Back History:  Angela was one of five employees that moved to home offices three months ago.  She had volunteered right off the bat because she could spend more time with her family if she eliminated all that commuter time [on a bad day she was losing over three hours roundtrip!]  



The telecommuting has had it’s ups and downs but her relationship with Ralph is going downhill.  They’ve been having frequent misunderstandings, and a few small arguments. Angela feels that Ralph’s operating on an out of sight out of mind mentality.  He doesn’t even seem to read the email reports very closely.  They do meet every two weeks at the office as a check-in but it’s a pure formality. To make matters worse, a colleague who works in the office told her about all the high stakes claims another investigator, Bob, has been working on. Angela hasn’t had anything ‘hot’ in weeks.





Ralph is happy with the way things are working out.  The office is no longer over-crowded or disturbingly noisy.  The telecommuters are doing their work and reporting regularly.  He is having more trouble staying on top of their reports and their projects but that’s probably because he’s so busy.



Scene Set-up: Ralph and Angela are meeting to discuss their recent miscommunications and Angela reveals her dissatisfaction with her recent treatment.



Scene Location: Ralph’s Office



The Meeting - Summary:  Ralph brings up the missed deadline and Angela says she was not informed that the deadline had been moved.  Ralph explains how the information was conveyed.  She says she feels that she is out of sight, out of mind and that since she began telecommuting it has hurt her career.  She notes that Bob has received more high-stakes claims than her recently.  Ralph explains that assignments to employees are based on their talents and experience and that Bob had experience with one client which is why he chose him.  Ralph says he didn’t realize she was feeling this way and wasn’t prepared.  He suggests they meet later.



Two weeks later – Angela misses an appointment with Ralph and deliberately fails to provide work on time.  Ralph wants to understand what has happened to change the good working relationship they had.  Angela threatens to resign, but Ralph maintains his composure and urges her to try to work out a solution with him.  They decide that for the next two weeks they will talk on the phone everyday.  Then, they will decide if Angela should come back to the office rather than telecommute.



Afterthoughts – Summary:  Ralph states his strategy was to listen to Angela.  He notes that he often has issues with people working from home because it’s difficult to keep track of what is going on.  He feels voicemail and email are impersonal.  He drafts written plans with employees who want to work from home that outlines goals.  The plans are executed on a temporary basis until it is clear that they will work and then he gives final approval.  



Dossier:  The specific artifacts included in the DVD are:

1.                  Emails between Ramos and Zanoni

2.                  Voicemail from Sorento to Zanoni

3.                  Ramos’ Assistant’s Voicemails to Zanoni





IV.  Discussion Questions:



The References and related Discussion Questions may be found in PowerPoint slides 12-1 to 12-9 on the instructor’s side of the text’s Website.



Learning Objective #1: To assess students’ understanding of the challenges of telecommuting.



1.      What advantages and disadvantages to telecommuting is Angela experiencing?  See PPT 12-3.



Angela enjoys not having to commute and likes to be closer and more available for her family.  The voicemail from her friend indicates that she may be losing the personal relationships she enjoyed at work because of decreased interaction.  She clearly feels left out of certain decisions and is concerned her career may be impeded.



´              1.  How should Ralph respond [when Angela says “since I’ve been working from home, I’m out of sight”]?

A.     Compassionately

B.     Clarify issues

C.     Debate point



Ralph attempts to clarify the issues to calm Angela down.



2.      What advantages and disadvantages to managing a telecommuter is Ralph experiencing?  See PPT 12-5.



His intention was to improve moral and job satisfaction of Angela by cutting down her commuting time.  In actuality, the opposite result has occurred.  He blames this on the difficulty coordinating and controlling staff and monitoring their performance.  He is clearly frustrated by not being able to reach Angela readily when important work-related matters arise.



´        2. What is Ralph’s strategy?

A.     Emphasize strengths

B.     Hide biases

C.     Change topic



Ralph tries to emphasize Angela’s strengths and explains that Bob was chosen for the higher stakes assignment because of is past experience, not because he is in the office more regularly.





´        3.  How was this [initial] meeting?

A.     Successful

B.     A failure

C.     A good start



It appeared that the meeting was a good start, but Angela’s actions in between the two meetings indicates that she is very upset and feels discriminated against because of her working arrangement.



Learning Objective #2:  To analyze and evaluate managerial actions related to telecommuting.



What should Angela have done to prepare herself for telework? Refer to PPT 12-7.


Angela needed to really assess whether her work style was suited for telecommuting.  While there are obvious advantages (shorter commute time), there are some less obvious disadvantages.  One needs to assess themselves on the degree that the reality of telework will impact them.  The questions found in PPT 12-7 would be a good start in this assessment.



´        4. Angela is dejected. Ralph should focus on:

A.     Her value

B.     Her poor performance

C.     His mistakes



Ralph should help Angela realize her value to the organization and explain that he did not intend to make her feel left out.  



What should Ralph have done to prevent this problem from occurring?  Refer to PPT 12-9.


Completing the Checklist provided in the slide would be effective.  Specifically, for their situation, a more formalized communication process needs to be created.  



´        5. What should Ralph do [when Angela says she want to resign]?

A.     Accept resignation

B.     Try new approach

C.     End meeting





Because Angela is a valuable and long-time employee he should try a new approach with her to urge her to work out this problem.



´            6. What’s Ralph’s next step?

A.     Find replacement

B.     Communication often

C.     Reassign work load



Ralph’s next step is to communicate more frequently with Angela to see if performance improves and if the telecommuting arrangement will work in the future.





Exercising Strategy


From Big Blue to Efficient Blue


Summary


In 1993 IBM racked up over $8 billion in losses when it was blindsided by the switch in consumer preferences from mainframe computers to smaller, networked personal computers.  The new incoming CEO, Lou Gerstner, created a new vision and strategy for the company.  The strategy was to switch from being an old-fashioned manufacturing company to a modern service provider.  In addition, Gerstner wanted to restructure operations to reduce costs and promote efficiencies.  



The human resource division was most impacted by these changes, which was downsized and became centralized.  Technology was integral to facilitating this centralized strategy.  The sprawling, geographically dispersed units were replaced with an efficient three-tier system.   Despite the radical downsizing of the human resource unit, employee satisfaction with service increased to 90 percent.  The restructuring and redesign of these IBM jobs have formed a “blue”-print for many other HRM departments in other organizations.



1.         In terms of our discussion of organizational structure, in what ways did the structure at IBM change under Lou Gerstner and what impact did this have on individual jobs?





            The changes that IBM made created a functional structure with extreme centralization to the degree that the HRM department was condensed to one unit with less than 1,000 employees from worldwide offices with over 3,500 employees.  The company departmentalized the department into a 3-tier unit, each with its own specialized employees.



2.         Compare and contrast the direction of structural change at IBM with the direction of change we saw in our opening story regarding the structural realignment at Microsoft.  



            Student answers may vary.  The main difference between the strategies of these companies is that while Microsoft took on a more decentralized approach, IBM took on a more centralized, departmentalized approach to its HR department.  Both organizations were plagued with drops in market share and profitability.  Both organizations hoped these strategies would lead to improved efficiency.  For Microsoft, improved efficiency would come from reduced turnover due to higher intrinsic employee motivation.  For IBM, improved efficiency would come from downsizing the HR department.  



3.        Since both IBM and Microsoft achieved their goals by changing their structures and job design in opposite directions, what does this say about the relationship between organizational structure and job design on the one hand and organizational performance and job satisfaction on the other?



            Student answers may vary.  One the one hand, downsizing an organization’s structure and redesigning jobs to be more efficient can improve the profitability of a company.  However, downsizing can negatively affect an organization by lowering employee morale (this was not the case at IBM).  On the other hand, restructuring an organization and redesigning jobs to improve employee motivation can be costly for the company.  (Because of reduced turnover, this was not the case for Microsoft).  There are definite tradeoffs between these two strategies, and these examples demonstrate that there is no one best strategy within an industry.  Managers must carefully evaluate strategies before implementing them.





Managing People:  From the Pages of Business Week



Tech Jobs are Sprouting Again


Summary



For the first time in three years, the tech job market is showing signs of life.  This has significant implications for job creation.  Thanks to Corporate America’s robust demand for tech gear and services, the hiring is relatively broad-based.  Technical firms like Symantec and Qualcomm are hiring at twice the rate of last year.  These companies are also filling positions that many feared would be sent to China or India.  Management consulting is another hot area in terms of job creation.  These positions are particularly resistant to moving offshore.  People adept at overseeing the development of software products are also in demand.  In some sectors, such as telecommunications, layoffs will continue.  



1.         What were some of the factors that led to widespread unemployment among technical workers prior to the recent upsurge in hiring?



            Student answers may vary.  Overcapacity is the major factor that let to widespread unemployment among technical firms.  In the late 90s, tech firms were eager to hire many employees because of the technological boom.  After 2000, the tech bubble burst, and because of overcapacity, companies were forced to layoff employees in order to remain competitive.  



2.         In what ways are the technology jobs that are now emerging in the U.S. economy different from those that existed ten years ago?



            Because to the recent trend in outsourcing, tech jobs today have to be strongly resistant to moving offshore.  This is good news to management consultants and other managers.  Today, lower-level support and maintenance tasks are concentrated overseas, while high-tech, high profile jobs are staying in the U.S. sector.



3.         If you were a worker in the tech industry like Mark Herleman, what steps would you take to help buffer yourself from the ups and downs of this labor market?



            Student answers will vary.  An important key attribute in the tech industry is being able to stay on top of innovation changes in technology.  Mark could do this by enrolling in classes or researching new technologies or inventions.  In addition, Mark could safeguard his job by obtaining management skills, particularly as a consultant or product development manager.  Obtaining an MBA would be a great way to do this.  





Additional Activities


In‑Class Activity 1


Many students are or have been employed. Ask students to choose a company for which they have worked and analyze the corporate culture at that company. Answers may include some of the following:



·                    Did a sense of teamwork and cooperation prevail?

·                    Was employee morale high?

·                    Who was primarily responsible for the attitudes that dominated in the workplace?

·                    Was the company successful?

·                    Was the company a good one to work for? Why? Why not?

·                    What could have been done differently to improve the culture?



In‑Class Activity 2


·                    Invite students to discuss their experiences traveling by air. Have any of them flown Southwest Airlines? If yes, does the airline measure up to everything that is being said about it?

·                    Do they have a favorite airline? Why?

·                    What, in their opinion, are the most important reasons for choosing an airline?

该用户从未签到

 楼主| 发表于 2010-10-3 20:10:32 | 显示全部楼层
Chapter Summary


This chapter describes the process an organization uses to plan and recruit so that there will be adequate human resources. The steps described are linking business strategies to future needs, forecasting labor demand and supply, determining in what positions there will exist a labor shortage or surplus, setting goals regarding future human resource needs, and identifying strategies to solve the problems of shortages and/or surpluses.  The ability to recruit successfully is described as depending upon personnel policies such as job posting (an internal strategy), level of pay in comparison to the market, and the extent of job security.  The chapter describes sources for recruiting as well as the use of yield ratios and costs to evaluate the effectiveness of sources.  Lastly, the role of the recruiter and how to enhance his or her impact on the candidate is presented.



Learning Objectives


After studying this chapter, the student should be able to:



1.         Discuss how to align a company's strategic direction with its human resource planning.

2.         Determine the labor demand of workers in various job categories.

3.        Discuss the advantages and disadvantages of various ways of eliminating a labor surplus or avoiding a labor shortage.            

4.         Describe the various recruitment policies organizations adopt to make job vacancies more attractive.      

5.         List the various sources from which job applicants can be drawn, their relative advantages and disadvantages, and the methods for evaluating them.

6.         Explain the recruiter’s role in the recruitment process, the limits the recruiter faces, and the opportunities available.            





Extended Chapter Outline


Note: Key terms appear in boldface and are listed in the "Chapter Vocabulary" section.



Opening Vignette: History Repeating Itself?  The Impact of Offshoring


The introductory vignette takes a historical perspective on offshoring.  First, it describes how introduction of new technologies created offshoring opportunities in the 1800s.  This threatened the Indian textile industry and benefited the British textile industry.  Today, advances in fiber-optic technologies are causing outsourcing of call center workers, software developers, mortgage processors, medical technicians, financial analysts, tax accountants and other jobs.  The main beneficiaries are Indian workers, much to the angst of U.S. workers, managers, and politicians.  There is much dissent over viable solutions to the job creation problem this causes in the U.S.   



I.          Introduction—Firms and the workers they employ do not exist in a vacuum, as indicated in the above vignette, create both constraints and opportunities, and employers can respond in many ways. Keys to utilizing labor markets to the organizations own competitive advantages are:



1.         Companies must have a clear idea of their current level of human resources (strengths and weaknesses).

2.         Companies must be aware of where they are going in the future and how the current configuration of human resources relates to what they will need in the future.

3.         Where there are discrepancies between present configurations and those needed in the future, organizations must have programs to address these issues.



II.         The Human Resources Planning Process—The process consists of forecasting, goal setting, and strategic planning, program implementation, and evaluation



A.        Forecasting determines the demand and supply of human resources and predicts in what positions either surpluses or shortages will exist in the future.



1.         Determining Labor Demand





a.         Labor demand can be   predicted by the use of statistical techniques such as leading indicator (text Figure 5.2), which is an objective measure that accurately predicts future labor demand (for example, sales may be directly related to a need for salespeople, but not sales support positions).



b.         Statistical planning models are useful when there is a long, stable history that can be used to reliably detect relationships among variables.



c.         Statistical models are almost always complemented by subjective judgments of people who have expertise in the area.  



2.         Determining Labor Supply



a.         Internal labor supply is determined by a detailed analysis of how many people are currently in various job categories, modified to reflect changes in the near future caused by retirements, promotions, transfers, voluntary turnovers, or terminations.



b.         A transitional matrix is a table used to project internal labor supply. The matrix shows the propor­tion or number, of employees in different job categories at different times and how people move from one job to another in the organization and how many people enter and leave the organization (text Table 5.1). This method is best combined with judgmental methods.



3.         Determining Labor Surplus or Shortage—By comparing fore­casts for labor supply and demand for specific jobs, the organi­zation can determine what it needs to do.



Example: General Motors Corporation has undergone several hiring freezes over the last few years.  By 2005, the average age of its workforce will be 48.  To reduce its workforce, GM allows natural attrition and retirement processes perform the downsizing rather than laying off workers.  This creates a problem because GM is responsible for the pension and health care costs of all these new retirees.   





B.         Goal Setting and Strategic Planning—These tools define specific goals regarding a desired end and provide benchmarks for deter­mining the relative success of a program aimed at addressing labor surpluses or shortages. A specific timetable is set for when results should be achieved (Text tables 5.2 and 5.3). The point should be clear that without planning by the organization, employees suffer the most, especially in a labor surplus because of layoffs and pay reductions.



            1.         Options for reducing an expected labor surplus:

-         Downsizing

-         Pay reductions

-         Demotions

-         Transfers

-         Work sharing

-         Hiring freeze

-         Natural attrition

-         Early retirement

-         Retraining



2.         Options for avoiding an expected labor shortage:

-         Overtime

-         Temporary employees

-         Outsourcing

-         Retrained workers

-         Turnover reductions

-         New external hires

-         Technological innovation



Competing Through Globalization


Low Cost Strategy Cuts Both Ways


In 1998, labor costs in Mexico and NAFTA helped make Jabil Circuit’s manufacturing operations highly successful.  By 2002, demand for the company’s products slowed significantly because Chinese firms could produce the same products at significantly lower labor costs.  In the past three years, Mexico has lost approximately 400,000 jobs to China.  To deal with this, some firms are producing more complex products.  Mexican firms are also competing on speed.  Finally, Mexican firms are increasingly trying to compete on flexibility.





3.         Downsizing often has immediate positive effects. However, the long‑term effects are often negative in terms of morale.



a.         There were four major reasons that organizations engaged in downsizing:



-     Cost reduction through decreased labor.



-     Closing outdated plants or introducing technological change reduced the need for labor.



-     Mergers and acquisitions reduced the need for large bureau­cracies.



-     For economic reasons, many firms relocated parts of oper­ations. Although the jury is still our on whether these downsizing efforts have led to enhanced organizational effectiveness, some early reports indicate the results have not met expec­tations.



Example: Universal Studios downsized operations in Los Angeles and increased operations in Orlando because costs of producing television shows are 40 percent higher in Los Angeles.



b.         One study of 52 Fortune 100 firms shows that most firms that announce a downsizing campaign show worse, rather than better, financial performance.  Reasons include:



-    Although the initial cost savings are a short-term plus, the long-term effects of an improperly managed downsizing effort can be negative.

-         Many downsizing campaigns let go of people who turn out to be irreplaceable assets.

-         Employees who survive the staff purges often become narrow-minded, self-absorbed, and risk-averse.  





Competing Through Sustainability


Productivity Improvement: Who Wins and Who Loses?


The most recent turn of recession and recovery cycle was unique in two important ways: productivity rates during the recession were actually rising and these rates during the recovery were almost unprecedented. The winners in a world of increasing productivity were: corporate profits, managers and executives (who were the only ones whose real wages rose), investors due to stock gains, and flowing of foreign capital into the U.S. due to stock gains and finally, the consumers and retirees. The losers were: young inexperienced workers and uneducated older workers.   



4.         Early Retirement Programs—The average age of the U.S worker is getting older. Because of improved health of the older worker and decrease in physical demand of the job individuals are able to work long past the traditional retirement age. Although older workers bring experience to the table, they also create problems like being more costly than younger workers because of seniority, higher medical costs, and higher pension contributions. Many employers are stressing early retirement by offering incentive programs to older employees.



5.         Employing Temporary Workers—Some strategies can be turned on and off fairly painlessly, such as the use of overtime and/or temporary employees. Operational flexibility is the primary reason for this, although the use of temps also frees the firm from many admin­istrative tasks and financial burdens (health insurance, pension, worker's compensation, life insurance, etc.). Smaller companies may use temporary agencies to do their employment screening for them. After 90 days, if the employee works out, he or she is often offered a permanent position. Training may be done by the agency as well. Finally, temporary employees bring a fresh perspective to the firm, particularly if they have temped in a number of other organizations. The potential conflicts between permanent and temporary employees need to be managed. There are several key issues:



a.         The organization needs to have bottomed out first in terms of

any downsizing before it starts bringing in temporaries.





b.         If the organization is concerned about the reactions of full‑time workers to the temporaries, it may want to go out of its way to hire “nonthreatening” temporaries.



c.         The organization, however, must be careful not to create the perception that temporary workers are second‑class organiza­tional citizens.



d.         Finally, HR staff can also prevent feelings of a two‑tiered society by ensuring that the temporary agency provides benefits to the temporaries that are at least minimally comparable with those enjoyed by the full‑time workers with whom they interact.



Example: "Try before you buy" lets you evaluate a temp before hiring the person full‑time. This strategy is becoming increasingly popular, since you can judge the quality of a person's work without making a permanent commitment this reduces turnover (since the temp can also try out the job and organiza­tion) and the high cost of advertising, interviewing, and training a new employee.



Although the hourly rate for a temp might be higher than for a permanent employee, costs can be controlled through flexible staffing (i.e., if you do not need the temp, there are no costs, compared to a full‑time employee who is paid regardless of the workload). Planning, of course, should be used to determine if a temp should be offered a permanent job. The organization should consider these issues when deciding whether a temp should become a full‑time employee:



·                    Is the job low‑ to mid‑level? A higher‑level job should prob­ably have an external search.

·                    Is the temp interested in a full‑time job?

·                    What does the temporary firm charge if you hire one of their temps?

·                    Will the temp's skills transfer to a permanent job?

·                    Is the temp's salary and background in line with the perma­nent job's?





(Source: Max Messmer, "When Hiring Temporaries Can Be a Permanent Solution," Human Resources Professional 5, no. 3, pp. 25‑26.)



6.         Outsourcing—This occurs when a firm is interested in a broad set of services performed by an outside organization.



a.         Outsourcing may be driven by lower labor costs at the outside firm, by economies of scale, or when a firm simply does not have the expertise or facilities to complete the services them­selves. Design engineering jobs are more frequently outsourced and are expedited by the use of computer networks. One concern, especially with outsourcing to international firms, is that the U.S. firms will lose their expertise in certain areas, resulting in a downward spiral.



b.         Offshoring is a special case of outsourcing where the jobs that move actually leave one country and go to another, like the example in the opening vignette.



c.         Steps to take that help ensure the success of outsourcing strategies include:

-           It is a good idea to outsource only those jobs that are repetitive, predictable, and easily trained.  

-           When choosing an outsourcing vendor, it is usually the bigger and older the better.  

-           Jobs that are proprietary or require tight security should not be outsourced.  

-           It is a good idea to start small and monitor constantly.



7.         Altering pay hours—Many companies that are facing labor shortages are forced to get more hours out of existing employees. Although employers are forced to pay higher wages, many prefer this to hiring and training new employees.   



Strategic use of HRM


Lincoln Electric is committed to a no-layoff strategy even though it does not experience uniform demand for its products.  It does this by constantly retraining employees and moving them around.  Although some employees don’t like the constant change, they are glad they will always have work and benefits.





C.        Program implementation and evaluation are critical elements to ensure that an individual is held accountable for achieving the stated goals and has the necessary authority and resources to accomplish goals. Program evaluation examines results:



1.         It considers whether the company has avoided any predicted shortages or surpluses.



2.         It examines the programs implemented to ensure that they had the desired results and whether the programs were implemented as planned.



3.         It is also crucial to examine the process in addition to the results, however. For example, ultimately, human resources planning should reduce instances of over‑and- under supply of human resources. It will do this with increasing effectiveness only if forecasts and projections become increasingly accurate.



D.        The Special Case of Affirmative Action Planning


Consider underutilization for voluntary affirmative action to promote diversity and/or as a requirement of serving as a contractor to the federal government.



1.         Work‑force utilization review is a comparison that examines the proportion of a company's workers in a subgroup to the proportion that each subgroup represents in the relevant labor market. This comparison is used to determine whether any subgroups may be underutilized. Underutilization occurs when the proportion of the subgroup in the organization is lower than that of the labor market.



a.         A voluntary affirmative action program may be imple­mented in which selection programs may be reviewed and revised to decrease the underutilization and/or increase the diversity of the organization and decrease the likelihood of a lawsuit. Steps for implementation are identical to the generic planning process discussed earlier.





b.         Students should be aware, however, that voluntary affir­mative action programs may be subject to lawsuits alleging reverse discrimination. Court‑ordered affirmative action plans are mandated by the courts as part of the settlement of discrimination complaints.



III.       The Human Resource Recruitment Process—Human resource recruitment is any orga­nizational activity that is designed to affect the number of people who apply for vacancies, the type of people who apply for them, and/or the likelihood that those applying for vacancies will accept positions if offered. The goal of recruitment is to ensure that when a vacancy occurs, the organization has a number of reasonably qualified appli­cants to choose from.



A.        Personnel policies, in relationship to recruitment, affect the nature of the vacancies for which people are required and impact the ability of the organization to recruit and its level of requiring success.



1.         Internal versus External Recruiting—A decision must be made on whether to recruit from within or outside the organization. Companies that recruit from within often use a job posting program in which vacancies are publicized on bulletin boards, in company newsletters, or in memos. Employees can apply and typically receive first consideration before the organization searches externally. Such policies make it clear that there are opportunities to advance and increase the level of skills within the organization.



Example: McDonald's restaurants provide a good example of promoting from within. Employees who once cooked for a restaurant now own several.



2.         Lead‑the‑market pay strategy is a policy of paying higher‑than ­current market wages. This creates a distinct advantage in recruiting since higher pay can make up for a job's less attrac­tive features. There is also pressure on human resources to deliver the best employees, so higher productivity, quality, and so on, are achieved.





3.         Employment-at-Will Policies



a.         Employment‑at‑will policies state that either party in the employment relationship can terminate that relationship at any rime, regardless of cause. Many legal protections exist that make it difficult for organizations to manage employment entirely “at will.”



b.         Companies that do not have employment at will typically have extensive due process policies that describe steps an employee can take to appeal a termination decision. Employment‑at‑will companies are only considered equally attractive to due process companies when the lack of job secu­rity is offset by higher‑than‑average total compensation.



4.         Image advertising promotes an organization as a good place to work in general and may be particularly important for organi­zations in highly competitive labor markets that perceive them­selves as having a bad image.



Example: Dow Chemical found that the negative publicity asso­ciated with its production of Agent Orange hurt its ability to hire the people the company wanted. Dow spent $60 million on a campaign to communicate a positive message about the company.



B.         Recruitment Sources—Since recruitment sources are unlimited, an organization must decide how to reach the best sources of potential employees.



1.         Internal versus External Sources—Relying on internal sources is useful since employees are well known and are knowledgeable about the organization and jobs. However, there may not be enough internal recruits (especially for entry‑level jobs); internal recruiting does not encourage creativity or change; and, if the current force is not diverse, internal recruiting will lead to a homogeneous work force, which can pose legal threats



2.         Direct Applicants and Referrals



a.         Direct applicants are people who apply for a vacancy without prompting from an organization.  





b.         Referrals are people who are prompted to apply for a vacancy by someone within the organiza­tion.



c.         Direct applicants may have already investigated the organiza­tion and are sold, which creates a self‑selection that is easier for the organization. Direct applicants tend to be one of the best sources, since they are also low cost.



3.         Advertisements in newspapers and periodicals – it is typically are less effective than direct applicants or referrals and is more expen­sive. Jobs should be described very specifically in advertisement. Local newspapers, although commonly used do not target skill levels very well. Journals, periodicals, and/or cable television may be useful in reaching the appropriate audience.



4.         Electronic Recruiting – The growth of the information highway as opened up new vistas for organizations trying to recruit talent.  Ways to get into “e-recruiting include:



            a.         use an organization’s own web page to solicit applications.



b.         use large, well known job sites such as Monster.com, HotJobs.com, or CareerBuilder.com.



5.         Public and Private Employment Agencies—Agencies will search their computerized inventory of individuals searching for work for an organization at no charge.



a.         Public employment agencies serve primarily the blue-collar labor market.



b.         Private employment agencies serve primarily the white-collar labor market.  Theses agencies charge the organization for referrals.





c.         Executive search firms are often referred to as “head hunters” because they recruit executives who tend to be employed. A high level of anonymity for the executives and the organization is employed until it is clear that both sides wish to pursue an inter­view. This is an expensive tool since the charge is roughly one-­third to one‑half of the annual salary of the executive who is placed. Often, organizations consider the successful recruitment of top executives so critical that the charge is appropriate.



6.         Colleges and universities may be an important source for entry ­level professionals. To increase effectiveness, organizations employ internship programs to get early access to potential appli­cants and to assess their capabilities directly. Interns also receive more realistic information about a potential job at the organiza­tion. A job fair is a place where employers gather for a short time to meet large numbers of potential applicants. These are often held on campuses and can inexpensively increase visibility.



Competing Through Technology


Taming the Monster


Until recently, Monster.com, along with eBay, Google, and Amazon.com was considered one of the few success stories that survived the dot.com bust.  However, in 2003, revenues were down, and Monster.com had to layoff 250 employees.  One of the main reasons for this shrinkage in revenues is intensive competition from CareerBuilder.com.  Both big hiring boards and niche boards are threatened by new entrants, like DirectEmployers.com.



7.         Evaluating the quality of recruiting sources can be done by compiling yield ratios that express the percentage of applicants who successfully move from one stage of the recruitment and selection process to another. Comparing yield ratios and the costs of various sources helps determine which sources are most effective. Quality of recruits should also be considered. A caution is provided that the sources depend on the skills that are needed and where it is logical to recruit, given those needs. (Table 5.4 and provide examples of yield ratios for different sources.)





C.        Recruiters—The recruiter comes late in the process, at which point an applicant has a good idea whether he or she is interested in the job. Since recruiters are perceived by the applicant to be "selling" the job, their input may be discounted. Recruiters may have more impact depending on a number of factors (see Figure 5.7).



1.         The Recruiter’s Functional Area—The recruiter is likely to be perceived as more credible if he or she is from the same func­tional area the recruit is being considered for.



2.         The Recruiter's Traits—Critical traits appear to be warmth and “informativeness.”



3.         The Recruiter's Realism—Deceiving candidates about the nega­tive elements of a job may increase later turnover; however, telling candidates about negative elements does not appear to inoculate them against disappointment nor make negative elements go away. Personnel policies that affect the job's attrib­utes are likely to be more critical than recruiter realism.



4.                  Enhancing Recruiter Impact—Recruiters can provide timely feedback about the status of the job search, avoid unfa­vorable behaviors that send poor messages about the organiza­tion, focus on inexperienced applicants with whom they might be more influential, and avoid mixing recruitment with selec­tion. Recruiting should be done in teams, including a person from the functional area and members who are female and minority. (See Table 5.5 for quotes from applicants who had bad experiences with recruiters.)







A Look Back



The chapter opener showed how technological and social changes can often disrupt existing employment patterns, affecting the ability of nations, the firm, and individuals to compete in contemporary labor markets.  The speed and effectiveness of how nations, firms, and individuals respond to these changes is critical in determining their long-term success and well-being.



Questions


1.         At the national level, what steps can politicians and labor leaders take to help their citizens compete in an ever-changing market?



            Student answers will vary, but could mention that some politicians advocate removing of tax credits for companies that outsource jobs.  



2.         At the firm level, what steps can organizational leaders and human resource professionals take to help their companies prosper in times of rapid change and turbulence?



            Student answers will vary, but some could mention that labor leaders could encourage less focus on cost reduction, and a higher degree of focus on flexibility, speed, quality, etc.



3.         At the individual level, what can you and other job seekers do to ensure that you will always have access to a high-paying, secure job that you find interesting and challenging?



            Student answers will vary.  Some students may pursue higher education to set themselves apart from other workers.  Some may advocate retraining and cross-training activities for workers.





Chapter Vocabulary


These terms are defined in the "Extended Chapter Outline" section.



Forecasting

Leading Indicator

Transitional Matrix

Downsizing

Outsourcing

Offshoring

Work‑Force Utilization Review

Human Resource Recruitment

Employment at Will Policies

Due Process Policies

Direct Applicants

Referrals



Discussion Questions


1.         Discuss the effects that an impending labor shortage might have on the following three sub-functions of human resource management: (a) selection and placement, (b) training and career development, and (c) compensation and benefits. Which sub-function might be most heavily impacted, and in what ways might these groups develop joint, cooperative programs to avert a labor shortage?



Labor shortages may have the following effects on:

Selection and placement—There will be increased pressure on this function to recruit more widely, search for nontraditional recruiting sources, and perhaps to decrease the selection requirements simply to provide more desperately needed employees.

Training and career development—Training will also be pressured to design a training program thee will produce "home grown" employees from within the company who can be taught the job. If people with fewer qualifications are selected, it will also be the responsibility of this group to provide skills reining, perhaps even basic‑skills training, that will result in reducing the gap between the people that the organization can hire and the skilled employees that must be available to meet the job demands.



Compensation and benefits —There will be pressure to hire the employees who are at a premium at a rate that may challenge internal equity and even external equity. The consequence, if this is done, will be a high level of demand to reevaluate existing jobs because of the perceived inequity of incumbents.

These pressures will strain the system, and a decision will have to be made regarding the cost/benefit of strategies to deal with the problem. For example, a solution may involve a short‑term recruiting strategy that would offer higher salaries than the organization would normally pay for the job in demand. A longer-­term solution might involve a combination of recruiting, selection, and training approaches that would recruit people who have a core of skills relative to those in the jobs that are experiencing shortages. These people may be paid at salaries that fit more comfortably within the compensation system, and they can be trained in the additional required skills.



2.         Discuss the costs and benefits associated with statistical versus judgmental forecasts for labor demand and labor supply. Under what conditions might either of these techniques be infeasible? Under what conditions might both are feasible but one more desirable than the other?



Certainly, judgment by experts may be intuitively appealing since it appears faster and less expensive than compiling and analyzing statistics.

There may not be employees available who have the skills to work with data for planning purposes. Hiring consultants to analyze the data might be very expensive, and the perception could be that the planning could be done without this laborious and expensive step. However, if the situation is relatively stable, these statistics may be the best source of projections for future planning strategies, since one could expect the future to be very similar to the past.



Both these techniques might be infeasible to use when the environment is so unstable that neither past experience nor expert judgment will provide a reliable base of information.

Both may be feasible when there is some instability in the environment. Statistical information could be examined in comparison to leading indicators. This might help define a relationship between actual changes in the environment and what happens to human resources. For example, in a bank, although mortgage interest rates may be unstable, there is a strong, negative correlation between mortgage interest rates and the number of analysts necessary to process mortgage applications Judgment of experts may be needed, however, to attempt a projection of what may happen to mortgage interest rates in the future.





3.         Some companies have detailed affirmative action plans, complete with goals and timetables, for women and minorities and yet have no formal human resource plan for the organization as a whole. Why might this be the case? If you were a human resource specialist interviewing with this company for an open position, what would this practice imply for the role of the human resource manager in that company?



Organizations may have derailed affirmative action plans because they must. They might be government contractors or they could be under a court‑ordered decree that forces them to have such a plan. There are no legal requirements for human resource planning and, therefore, many organizations might not possess the expertise to recognize the need for one or how to plan.

The position of the human resource manager in terms of career development might be highly problematic. If the organization does not consider it necessary to plan in reference to human resources, it is also likely that human resources are considered more as a cost than as an investment that will pay off in the future. Any manager might have difficulties in moving toward his or her career goals. Also, the human resources manager might legitimately be concerned that his or her role will not involve strategizing how human resources can contribute to business goals and helping the organization achieve its goals, but will be a role or managing paperwork and the process of human resources.



4.         Recruiting people for jobs that entail international assignments is becoming increasingly important for many companies. Where might one go to look for individuals interested in these types of assignments? How might recruiting practices aimed at these individuals differ from those one might apply to the "average" recruit?



Some colleges and universities have majors in international trade or business. Students often have a double major in a foreign language and business. They are more likely to be interested in going abroad. People who have language skills other than English might also be a possibility, particularly if they have lived in other countries. Placing job advertisements in special‑interest magazines might be helpful.

Recruiting practices might differ from those used for the “average” recruit, since most individuals (and their families) would probably not be interested in an international assignment. If the job will require international travel or reassignment, clearly it should be discussed early in the recruiting process. You may need a person on the recruiting team who is familiar with the language and culture where the person would be assigned. This team member would be better equipped than the average recruiter to assess the person's ability to adjust to a different culture. The family of the candidate should also be interviewed, since their interest and adaptation are critical in a successful expatriation.





5.         Discuss the relative merits of internal versus external recruitment. What types of business strategies might best are supported by recruiting externally, and what types might call for internal recruitment? What factors might lead a firm to decide to switch from internal to external recruitment and vice versa?



Merits of internal recruitment are that the organization will be more familiar with the motivation and work habits of the candidate, and it provides motivation to employees, since internal promotions do occur.

Merits of external recruitment include:



·                    Candidates may he recently educated/trained and therefore have up‑to‑dare skills and knowledge.

·                    Candidates might be more likely to be interested in and open to change and innovation.

·                    An organization is more likely to improve the situation of under­utilization of minorities and women.



Strategies related to high levels of competition would be best supported by internal recruitment, since losing employees' knowledge could hurt the organization. There would likely be evidence internally if employees were committed to the organization. Strategies that require constant change and innovation would be better served by external recruitment.



Clearly, an organization might need to switch recruitment strategies when the business strategy no longer matches. For example, when the Bell companies were divested from AT&T by the federal government the climate changed from an emphasis on stability and bureaucracy to one of entrepreneurial emphasis, flexibility, creativity, and customer orientation. Recruitment could not remain the same, and internal recruitment might not have been as effective, since the behaviors required of employees were very different (unless training and development was strongly oriented to increasing critical skills).





Manager’s Hot Seat


Diversity: Mediating Morality



I.  Introduction



The diversity of employees and the potential conflict that may arise as a result is the focus of this scenario.  Specifically, sexual orientation as a dimension of workplace diversity is presented here.  This scene provides a background for a rich and compelling discussion of how to handle difficult issues related to individual differences.  Applying this situation to other dimensions of diversity (e.g., religion, gender) will further the discussion and highlight to students the challenges of managing a diverse workforce.  



II.  Learning Objectives



To assess students’ understanding of workplace diversity and diversity management.
To analyze and evaluate approaches to managing diversity in the workplace.
To identify equal employment laws relevant to diversity management.


III.  Scenario Description:



Overview:  At the conclusion of a previous meeting between Syl Tang and two employees, Daniel Simmons and Bob Franklin, Daniel asked Tang about the same-sex partner benefits that he had requested earlier, incidentally informing Bob Franklin that he was homosexual.  Bob expressed both discomfort and disapproval, and refused to continue to work with Daniel as a result of this revelation.  Tang suggested meeting with each of them separately to discuss their concerns and then to regroup and discuss a solution.  The individual meetings have taken place and now they are meeting together.





Profile:

·        Syl Tang is the Senior Supervisor of Research and Production at EarthFirst Pharmaceuticals, a company dedicated to the research and production of vaccines for various diseases.  She manages eight different teams with specific areas of specialization.

·        Daniel Simmons is a Vaccines Manager.  He is an expert in tropical plants and botanical remedies.  He leads a team of scientists and collaborates with other research teams at EarthFirst.  He is also active in procuring funding for further research.

·        Bob Franklin is a Natural Resources Manager and is an expert in certain types of chemical re-actives.  In his role as Natural Resources Manager, he leads a team of scientists and collaborates with other research teams at EarthFirst.  He is also active in procuring funding for further research.



References:  The references included in the DVD are:

·        Dimensions of Workplace Diversity (PPT 10-3)

·        Managerial Roles and Diversity (PPT 10-4)

·        Promoting Effective Management of Diversity (PPT 10-6)

·        Major Equal Employment Laws (PPT 10-9)



Back History:  Syl has been the supervisor for seven years and is highly respected and regarded.  Her teams often cross-collaborate on special projects – each team has a specialized field of expertise.  The teams are quite large, totaling over 100 people.  With such a large department, Syl has less one on one contact with the employees than she would like.  



Simmons and Franklin have not worked together often, just on one or two short-term projects.  They are currently collaborating on a high-pressure project, heading down the homestretch.  Nothing out of the ordinary has happened at the office for a while.  Tang is compiling her end of the month reports, without much stress or pressure.  Simmons and Franklin are at the stage of their joint project where they are separately recording the joint findings as it pertains to their field of expertise.   



Simmons is gay and just started planning a commitment ceremony with his partner of 4 years.  Simmons has never made any declaration in the office about being homosexual – some people know, some don’t.



Scene Set-up: Simmons, Franklin, and Tang meet to discuss how they can meet the project’s objectives.





Scene Location: Tang’s office



The Meeting - Summary:  Syl explains that she and Bob have discussed a solution that would enable two of Bob’s employees to take on a larger responsibility and still meet the needs of the project.  Daniel states that Bob simply doesn’t want to work with him anymore because he is a gay man.  He is concerned that if the word gets out that Bob was uncomfortable working with Daniel and found another solution, then a precedent will be set that negatively impact Daniel’s career.  Syl argues that if Daniel didn’t get along with another co-worker she would not make Daniel work with him/her if there was another equally viable solution.  She says that no one’s personal life will be discussed outside of this meeting and Daniel strongly disagrees.  Syl suggests that Daniel come up with an alternative proposal and discuss it with her on Monday.



Afterthoughts – Summary:  Syl admits that Daniel has a good point because Bob probably will tell others that Daniel is gay.  She doesn’t feel that Daniel is being discriminated against because as long as the project is completed well, he will receive the credit regardless of how the other team was staffed.  She suggests that Bob should probably attend sensitivity training but says that there’s not much that can be done about employee’s personal feelings.



Dossier:  The specific artifacts included in the DVD are:

1.      Benefits Policy Excerpt

2.      Daniel Simmon’s Engagement Invitation

3.      Harassment Policy



IV.  Discussion Questions:



The References and related Discussion Questions may be found in PowerPoint slides 10-1 to 10-9 on the instructor’s side of the text’s Website..



Learning Objective #1: To assess students’ understanding of workplace diversity and diversity management.



The Dimensions of Workplace Diversity are provided in PPT 10-3.  List all of the dimensions of diversity you observed in this scenario.


Students will point out evidence of the diversity, beyond sexual orientation present among the three employees.  Race, ethnicity, age, gender, behavioral style, occupation





Which of the managerial roles shown on PPT 10-4 did Syl demonstrate?


Syl took on the role of the disturbance handler and liaison.



How effective was Syl in performing the roles you cited in #2?


Syl’s approach was very task-focused and she was interested in finding ways to separate the two co-workers rather than figuring out a way for them to work together.



Learning Objective #2: To analyze and evaluate approaches to managing diversity in the workplace.



Provide examples from the scenario in which Syl either effectively or ineffectively promoted diversity.  Use the information in PPT 10-6 to support your answer.


Syl did not effectively increase the accuracy of perceptions or encourage flexibility.  She seemed very willing to let Bob change the staffing of the project simply because he was uncomfortable around a gay man.  Thus, he has predicted that a negative interaction will take place simply because Daniel is gay, when they have been working together fine in the past.  Syl should attempt to challenge Bob on this belief.  She also did not empower Daniel to challenge discriminatory behavior.  When Daniel did, she shut him down and explained that this was an appropriate solution.  She provided an analogy of two co-workers who don’t get along – a similar solution would work for them.



´        1.  What is Syl’s objective [when suggesting to delegate certain portions of Bob’s work to Ralph and Jennifer]?

A.     Avoid conflict

B.     Complete project

C.     Mend team



Syl states that her goal is to complete the project.  It seems she is also interested in avoiding conflict by separating the two co-workers.  There is no indication that she wants to mend the team.



´        2. Daniel’s perception [that Syl is letting Bob’s personal opinion affect a working relationship] is:

A.     Clouded

B.     Accurate

C.     Biased





Syl believes that Daniel’s perception is clouded.  Students should debate this issue.  It seems that it is accurate given the fact that the working relationship is ceasing and Syl is trying to find alternative staffing models to complete the project.



´        3.How should Syl respond [to personal attacks in the meeting]?

A.     Mediate

B.     Change topic

C.     End meeting



Syl believes she should change the topic.  Again, students should discuss the pros and cons of this action.  What are the dangers of not addressing the issue at hand, what are the advantages?



´        4. What is the main issue [when Syl states there is not going to be a discussion of anybody’s personal life]?

A.     Confidentiality

B.     Completing project

C.     Upholding morals



Syl believes the main issue is completing the project.  Confidentiality as a concern doesn’t make much sense since there was an email invitation to Daniel’s engagement party distributed to the entire company.



´        5. Why does Syl propose this [having Daniel come up with an alternative solution]?

A.     Shirk responsibility

B.     Mediation tactic

C.     Empower Daniel



Syl is attempting to empower Daniel.  Students should discuss the effectiveness of this approach.  Isn’t it clear that Daniel wants to continue the working relationship?  What valid reason has been given to change the working relationship?





Learning Objective #3: To identify equal employment laws relevant to diversity management.



According to Title VII of the Civil Rights Act of 1964, is Daniel being unlawfully discriminated against (see PPT 10-9)?


No, Title VII does not protect people from discrimination on the basis of sexual orientation.  However, same-sex sexual harassment was found to be illegal by the Supreme Court in Oncale vs. Sundowner Offshore Service, In. (1998).



According to EarthFirst’s Harassment policy, is Daniel being discriminated against?


Yes, the policy clearly states that employees who feel they are being discriminated against on the basis of their sexual orientation have recourse.



What are the long-term implications of allowing Bob to refuse to work with Daniel?


If Syl allows and condones this action, it could become a slippery slope.  Daniel has a point (and Syl admits this in the Afterthoughts) that this will set a precedent for other employees to not accept Daniel because he is different from them.



What actions should Daniel take next if he is separated from other employees?


Daniel should follow the procedures clearly defined in the company’s harassment policy.





Exercising Strategy


Southwest Airlines: Focused on Take-Offs, Not Layoffs



In 2001, the airline industry was facing severe problems due to slumping business travel and vacationer demand.  Many airlines are still facing bankruptcy because of this slump.  Nonetheless, Southwest has never laid off employees and was still profitable during the most difficult times.  For this reason and others, the Southwest workforce is fiercely loyal, productive, and flexible.  



1.         In this chapter, we explored several alternatives to layoffs as a means of reducing a labor surplus.  Compare and contrast the list we generated with what was done at Southwest in the wake of the 9/11 attacks.  How did the response at Southwest differ from the other airlines?





Student answers may vary.  Southwest chose not to use a downsizing strategy because it felt that maintaining a loyal and trusting workforce was a core competency of the airline. Many of Southwest’s top managers took pay cuts and eliminated bonuses to ensure that workers’ jobs would be spared.  The company also used work sharing and retraining to make work more efficient, and because of this, the company operated under a hiring freeze for several months after the attacks. Southwest’s response differed from other airlines because every other major U.S. airline utilized downsizing.



2.         Southwest Airlines’ “no layoff policy” is an important component of its overall culture and strategy.  In what ways does this “no layoff policy”, which clearly hurts the airline in the short term, give Southwest a competitive advantage over other airlines in the long term?



Student answers may vary.  Although a “no layoff policy” may hurt Southwest in the short term, in the long term, it helps the company by reducing turnover.  Turnover is lowest at Southwest compared to its competitors.  Employees are loyal to the company because they trust management and know that their jobs are safe.  In addition, the company operates under a fun-loving corporate culture.  These qualities combined encourage workers to stay with Southwest.



3.         In the story that opened this chapter, we looked at the increased use of offshoring, where jobs are moved from the United States to other countries where labor rates are cheaper.  In what ways is offshoring similar and different from a simple layoff?  If there are some long-term benefits from avoiding layoffs, what might be the long-term advantages of trying to avoid offshoring?



Student answers will vary.  Offshoring is similar to simple layoffs because both cause loss of jobs in the U.S.  In fact, offshoring is one of the reasons why companies perform layoffs.  The difference is that most experts agree that offshoring creates comparative advantages.  International demand for U.S.-made products like computers goes up because consumers in other countries have more money to spend because of their new jobs.  With downsizing, demand for U.S.-made products is going down.   A long-term benefit to avoiding offshoring is that the manufacturing sector in the U.S. would increase, after declining over past years.





Managing People:  From the Pages of Business Week


Waking Up from the American Dream



Summary


Dead-end jobs and the high cost of college could be choking off upward mobility.  More than a quarter of the labor force is trapped in low-wage, dead-end jobs, partly blamed by the “Wal-martization” of America.  As a result, people are less able to move up the economic ladder over their lifetimes.  Many experts expected that the healthy economy of the 1990s would reverse this trend, but this did not occur.  Restoring mobility is less a question of knowing what to do than of making it happen.  



1.         This article makes reference to the Horatio Alger Association, which offers scholarships to disadvantaged youths.  Who was the real Horatio Alger, and why was he symbolic of the U.S. labor market for the last 200 years?



Horatio Alger was the founder of the Horatio Alger Association.  According to www.horatioalger.com:  



Mr. Alger’s association was founded in 1947, the Horatio Alger Association of Distinguished Americans continues to fulfill its mission of honoring the achievements of outstanding individuals in our society who have succeeded in spite of adversity and of encouraging young people to pursue their dreams through higher education.



The Association brings the “Horatio Alger Heroes” of today together with those of tomorrow by bestowing the Horatio Alger Award each year and by awarding more than $5 million annually in college scholarships to young people. Horatio Alger Scholars have faced challenges and realize that a college education is the avenue to a better future. The Association also provides its scholarship recipients with financial aid counseling, and internship and job placement assistance.







2.         In what ways is the current U.S. labor market different than it was over the last 200 years, and what is the objective evidence to indicate that there are fewer and fewer “Horatio Alger stories” out there today?


Student answers will vary.  The current U.S labor market is different than it was over the last 200 years because of advances in globalization and technology.  Because of globalization, demand for products has increased exponentially.  However, with the increase in demand, there is also an increase in competition, particularly in terms of labor costs.  Technology has made the world smaller and more accessible.  Because of technology, outsourcing and particularly offshoring, have become viable strategies for companies competing on a cost basis.  It is important to note that 200 years ago, technology set the stage for offshoring and global competition.



Students may not all agree that fewer “Horatio Alger Stories” exist today, depending on their personal take on globalization, outsourcing, and losses in the U.S. manufacturing sector.  Students may argue that because of lost jobs, the Horatio Alger Foundation is more critical today than ever.  



3.         What are some of the factors that have limited upward mobility in the contemporary U.S. labor market?



Student answers may vary.  Factors include the Wal-martization of America, the ever-growing wage gap, outsourcing, and globalization.



4.         What can be done at the national level by our government, the organizational level by U.S. firms, and the individual level by U.S. workers to help return to the days when upward mobility was the rule rather than the exception?



Student answers will vary. Politicians, companies, unions, the court system, customers, and employees themselves all have the power to reemphasize upward mobility.  Politicians and the courts must put an end to company practices that promote or allow discrimination.  Companies could establish strategies that encourage worker mobility as a way to gain employee trust and reduce turnover.  Employees could push for equal rights and fair pay.  Customers could buy products from companies that promote upward mobility, thereby giving companies the incentive to use such practices.

该用户从未签到

 楼主| 发表于 2010-10-3 20:10:50 | 显示全部楼层
Chapter Summary


The following requirements to use a personnel selection device are discussed: reliability, validity, generalizability, utility, and legality. The following methods of selection are discussed, in terms of the above factors, how the methods work in practice, and how they can be improved for use: interviews, reference checks, biographical information, physical ability tests, cognitive ability tests, personality inventories, work‑sample tests, honesty tests, and drug tests. The importance of the use of multiple measures and multiple judges in selection is emphasized.



Learning Objectives



After studying this chapter the student should be able to:



1.         Establish the basic scientific properties of selection methods including reliability, validity, and generalizability.      

2.         Discuss how the particular characteristics of a job, organization, or applicant affect the utility of any test.

3.         Describe the government's role in personnel selection decisions, particularly in the areas of constitutional law, federal laws, executive orders, and judicial precedent.

4.         List the common methods used in selecting human resources.

5.         Describe the degree to which each of the common methods used in selecting human resources meets the demands of reliability, validity, generalizability, utility, and legality.



Extended Chapter Outline



Note: Key terms appear in boldface and are listed in the "Chapter Vocabulary" section.





Opening Vignette: Standardizing Selection



The opening vignette discusses the problems Wal-Mart is facing in regards to selection for management positions.  Some Wal-Mart employees attest that selection for management positions was not standardized.  Women specifically held less than 1/3 of the managerial positions, although women made up over 2/3 of total hourly workers.  Wal-Mart has also taken a very minimalist approach to human resource management.  These issues and others explain why Wal-Mart is currently in a legal predicament with many of its employees.







I.          Introduction
   

The chapter will familiarize students with ways to mini­mize errors in employee selection and placement and by doing so, to improve the organization's competitive position. Five selection method standards will be discussed: reliability, validity, generalizability, utility and legality.



II.         Selection Method Standards



A.        Reliability is the degree to which a measure of physical or cognitive abilities, or traits, is free from random error. If a measure of a stable characteristic is reliable, the score a person receives will be consistent over time and over different contexts. Reliability is a necessary but insufficient characteristic of a good measuring device (See Figure 6.1 a & b).



1.         The concept of reliability is demonstrated by measuring height at different times. Even though height is supposedly a stable characteristic, slightly different results are generated every time height is measured. Each measurement is then composed of "true height" and "error of measurement." The average of all the errors (ignoring the positive or negative value) is referred to as the reliability of measurement.



a.                   The correlation coefficient is a measure of the degree to which two sets of numbers are related.  The correlation coefficient expresses the strength of the relationship in numerical form.  A perfect positive relationship equals +1.0; a perfect negative relationship equals –1.0.



b.                  When assessing the reliability of a measure, we might be interested in knowing how scores on the measure at one time relate to scores on the same measure at another time.  This is what is called assessing test-retest reliability.



2.         Standards for Reliability—Clearly, the more reliable the measure, the more likely decisions can be made on score differ­ences.  However, the actual score and the standard error of the measure have an impact on the ability to make a decision on the meaning of scores.





B.         Validity is the extent to which performance on the measure is asso­ciated with performance on the job (this definition assumes crite­rion‑related validity). Criterion‑related validation is demonstrated by a correlation coefficient that indicates a significant relationship between scores on the selection measure and job performance scores.



1.         There are two types of criterion‑related validity:



a.         Predictive vali­dation is an empirical relationship between scores on the selec­tion measure taken prior to persons being hired and, after roughly six months or more, their job performance scores.



b.         Concurrent validation is an empirical relationship between scores on the selection measure given to current employees and their job performance scores (processes are compared in Figure 6.3).



c.         Predictive validation is superior to concurrent validation since job applicants will be more motivated to do well on the test than job incumbents will. Also, current employees have learned on the job, and current employees tend to be homogeneous, which will lead to restriction of range and a lower correlation.



2.         Satisfactory levels of validity are typically defined by statistical significance (text Table 6.1).



Competing Through Technology


New Technology Redefines the Language of Personnel Selection



New developments in technology have forced many human resource professionals to subtly reexamine what some standard terms mean.  For example, because individuals can send unsolicited résumés to a various companies, the term “applicant” doesn’t mean what it used to mean.







3.         Content validation is the use of expert judgment to determine whether test items are a representative sample of the kinds of items, situations, or problems that occur on the job. One means to quantify the degree of content validity is to use a content‑vali­dation ratio (CVR), in which multiple judges determine whether items are essential or nonessential. The results are placed in a formula and rested for statistical significance. Limitations of content validation are that it is not used in situations when the person learns to do the job after he or she is hired and judges' ratings are made in reference to concrete behaviors; therefore, content validation is not appropriate when more abstract traits of individuals are being measured.



C.        Generalizability is the degree to which the validity of a selection method established in one context extends to other contexts such as different situations, different samples of people, and different time periods.



1.         It was believed that validity coefficients were situationally specific—that is, the correlation between test and job perfor­mance would vary across organizations. Research results suggest this is not the case and that tests such as cognitive ability show similar levels of correlation across jobs that are somewhat similar. The more complex the job, typically the higher the validity.



2.         It was also believed that tests showed differential subgroup validity, which meant that validity coefficients were different for people of different races or sex. Research suggests that there are very similar levels of correlations across different groups of people.



3.         Validity generalization is a process that determines whether a given selection technique (that has proven valid in another context for a specific job) is valid in another context where the job is very similar. The selection technique may be used in the different context without validating in the new context. Although there is evidence that supports the generalizability of many kinds of tests across situations and subgroups, there does not appear to be evidence that generalization can be done across time. In fact, it seems that correlations decrease as time Intervals increase.





D.        Utility is the degree to which the information provided by selection techniques enhances the effectiveness of selecting personnel in orga­nizations. Utility is impacted by reliability, validity, and generaliz­ability. Other factors will influence utility even when the latter is constant. For example, the selection ratio, which is the percentage of people selected versus the total number of applicants, will impact utility as well as the number of people selected (more people increase utility), race of employee turnover, and level of perfor­mance among chose who leave (to increase utility, turnover should be low with those leaving in low‑performance categories). (See Figure 6.4 a & b)



E.         Legality—All selection methods must conform to existing laws and legal precedents (Chapter 4 discusses these issues).



1.         Federal Legislation—The Civil Rights Act of 1991 (an exten­sion of the Civil Rights Act of 1964) protects individuals from discrimination based on race, color, sex, religion, and national origin. The 1991 act differs from the 1964 act in three different areas:



a.         It codifies employers' explicit obligation to establish the business necessity of any neutral‑appearing selection method that has an adverse impact on protected groups.



b.         It allows the individual filing the complaint to have a jury decide whether he or she may recover punitive damages in addition to lost wages and benefits for emotional injuries caused by the discrimination.



c.         It explicitly prevents the use of race norming as a means of giving preferential treatment to protected groups.







Competing Through Sustainability


Older but Wiser:  Demographic Shifts Create Opportunities for the Over-50 Set



A U.S. congressional committee determined that mandatory retirement of FBI workers led to a shortage of workers.  The commission determined that this may be one of the reasons the 9/11 attacks were not stopped.  The notion of rehiring older workers who were once retired is becoming an increasingly popular trend.





2.         The Age Discrimination in Employment Act of 1967 covers individuals who are between the ages of 40 and 69. Litigation related to this act has surged because of the general aging of the work force and downsizing. A 200 percent increase in litigation took place between 1991 and 1995. There is no protection for younger workers, and therefore there is no case for reverse discrimina­tion. There are provisions for jury trials and punitive damages.

                        

3.         The Americans with Disabilities Act of 1991 protects individ­uals with physical or mental disabilities:            



a.         "Reasonable accommodations" are required by the organi­zation to allow the disabled to perform essential functions of the job. Examples of accommodations include restruc­turing jobs, modifying work schedules, making facilities accessible, providing readers, and modifying equipment. An employer need not make accommodations that cause undue hardship such as undue cost or danger to the safety of other employees.



b.         There are restrictions on preemployment inquiries, such as asking if a person has a disability or requiring a medical examination prior to a job offer. (See “Additional Activities” for student involvement suggestions.)





Example: The first employment suit under the Americans with Disabilities Act was filed in November 1992 by the Equal Employment Opportunity Commission, charging a Chicago‑area security firm with illegally firing an executive director after he was diagnosed with terminal brain cancer (EEOC v. AIC Security Investigations Ltd).

In announcing the suit, John Hendrickson, regional attorney for the EEOC's Chicago district office character­ized the suit as a classic example of the type of discrimi­nation the ADA is intended to prevent.” Charles Wessel was fired, Hendrickson said, “apparently not because of his inability to do the job, but because of predictions about future health problems and because of stereotypical fears about disability. It appears that [he] was, in fact, performing his job up to the very time he was discharged."

The commission's suit seeks back pay and monetary damages for the emotional and other harm Wessel alleges that he has suffered, as well as his reinstatement and an injunction prohibiting the company from engaging in unlawful discrimination based on disability. (Adapted from "EEOC Files First Disabilities Act Suit on Behalf of Executive with Cancer," BNA's Employee Relations Weekly 10, no. 44, 1992,p.1209.)



4.         Executive orders relate to businesses that have government contracts with the federal government. Executive Order 11246 parallels the Civil Rights Act of 1964 but goes beyond it by (a) requiring affirmative action to hire qualified protected group applicants and (b) allowing the government to suspend all busi­ness with a contractor while an investigation is going on. The Office of Federal Contract Compliance (OFCC) issues guide­lines and monitors compliance.



III.       Types of Selection Methods



A.        Selection interviews are defined as a dialogue initiated by one or more persons to gather information and evaluate the qualifications of an applicant for employment. It is the most widely used selection method, although research suggests it can be low in reliability and validity. Face validity can also be low since highly subjective inter­views may upset candidates. The Supreme Court ruled in Watson v. Fort Worth Bank and Trust that interviews should he validated with criteria-related or content-validation procedures.





1.         The utility of an interview can be increased by the following sugges­tions:



a.         Interviews should be structured, standardized, and focused on accomplishing a small number of goals oriented to skills and behaviors that are observable.



b.         Ask questions that force the applicant to display job‑required behaviors or knowledge (situational interview items). Table 6.2 contains example items.



c.         Use multiple, trained interviewers who can avoid personal bias.



2.         A situational interview is an interview procedure where applicants are confronted with specific issues, questions, or problems that are likely to arise on the job.  They consist of:

            

a.         Experience-based questions

            b.         Future-oriented questions.



B.         References and biographical data gather background information on candidates. These techniques are, at best, weak predictors of future job success. Typically, references are very positive since only those who the applicants know will give a good reference are asked to do so, and there is a fear of being sued for libel if a bad reference is given. Over 10,000 such suits have been filed since 1983. In 70 percent of these cases, the recipient of the bad reference prevails, and the average award is over $500,000. Evidence on the utility of biographical information is more positive, particularly for clerical and sales jobs. Information is typically gath­ered via questionnaires





Competing Through Globalization


Private Contractors Go to War:  Impact of Work Context on Global Hiring Decisions



The military outsourced a full 30 percent of its services in the Iraq war, up from a mere 1 percent in the Gulf war of 1991. Many employees of the private contractors have been killed in the war providing services. Since military does not select and train these individuals, often people desperate for money are going into these types of jobs. According to Deming, better candidates are people who are financially stable, but attracted to the work because of sense of adventure or patriotism or high level of commitment to their employer.



C.        Physical ability tests may be relevant for predicting not only job performance but occupational injuries and disabilities as well. Criterion‑related validities tend to be quite strong, although adverse impact on the disabled and women is highly possible. The test, if demonstrably valid, can be used when evidence exists for physical ability being necessary for job performance and that failure to possess physical abilities could lead to risk of safety and health to the applicant, co‑workers, or clients.  These tests measure:



            1.         muscular tension

            2.         muscular power

            3.         muscular endurance

            4.         cardiovascular endurance

            5.         flexibility

            6.         balance

            7.         coordination



D.        Cognitive ability tests differentiate candidates on mental ability. Commercial tests are available that tend to be highly reliable and valid in predicting job performance. Although cognitive ability tests tend to be more valid for complex jobs, generalizability across different jobs and their relatively low cost make them one of the most useful selection methods. They often however, have an adverse impact on blacks, although differences appear to be declining. The abilities most commonly assessed (these can be combined in one score or used separately) are the following:



1.         Verbal comprehension refers to the ability to understand and use written and spoken language.





2.         Quantitative ability involves speed and accuracy in solving arithmetic problems.



3.         Reasoning ability involves the capacity to invent solutions to many diverse problems.



E.         Personality inventories categorize individuals by their personality characteristics. Common dimensions assessed are extroversion, adjustment, agreeableness, conscientiousness, and inquisitiveness (text Table 6.3 lists corresponding adjectives for these dimensions). Evidence for validity and generalizability of these selection methods is low. Job‑relatedness is a critical concern.



F.                  Work samples, such as typing rests, simulate the job. Work samples are job‑specific and tend to be high in criterion‑related and content validity and low in adverse impact. The disadvantages are that generalizability tends to be low, and they are relatively expensive to develop. The technique used in managerial selection is the assess­ment center, which uses a wide variety of multiple selection methods to rate applicants or job incumbents on their managerial potential. Validity tends to be quite high.



G.        Honesty tests and drug tests—The Polygraph Act of 1988 banned the use of polygraph tests for private companies except pharma­ceutical and security guard suppliers. Paper‑and‑pencil honesty testing attempts to assess the likelihood that employees will steal. Since these tests are new, there is little evidence on their effective­ness. Table 6.4 contains some sample items.



Example: In 1990, the Office of Technology and Assessment released a report on the validity of paper‑and‑pencil honesty tests. The conclusion of the report was that existing research was incon­clusive to determine the effectiveness of the tests. Wayne Camara. Ph.D., the American Psychological Association's director of scientific affairs, stated that there is a great range in quality of honesty tests, from effective to "nearly fraudulent." The problem of theft, however, is a serious one for employers. It is estimated that internal thefts cost companies between $15 billion and $25 billion a year. Since references are so difficult to obtain, organizations, particularly retail businesses, banks, toll collectors, and police and fire depart­ments, are looking for alternative methods to predict honesty.



Tests commonly gauge attitudes and perceptions about professional behavior. The tests predict the level of risk of theft for employees. The problem in relationship to privacy is that many items can be very offensive, such as questions asking about religion, sexual practices, and bathroom habits. Camara notes that these tests can send a message to applicants about the kind of workplace that the organization may be. The American Civil Liberties Union would like to see honesty testing go the way of the polygraph, since people should be judged on their work and not on arbitrary standards. Attorney Cliff Palessky says that people are looking for a magic bullet after the lie detector was banned.  He also believes that these tests will be a hotbed for discrimination claims. (Adapted from “Workplace Honesty Testing Raises “validity, Privacy Questions," BOAS Employee Relations Weekly] 1, pp. 64, ‑ a.)



Drug‑use tests tend to be reliable and valid, particularly when the screening tests" (e.g.. immunoassay tests) are followed up with more expensive “confirmation” tests (e.g, gas chromatography tests).



1.         The major controversies are whether drug tests represent an invasion of privacy, an unreasonable search and seizure. or a violation of due process. At this point, there has not been much legislation or litigation.



2.         These tests should be administered systematically to all appli­cants applying for the same job, and testing is likely to be more defensible when there are safety hazards associated with the failure to perform. Test results should be reported to the appli­cant, who should have an opportunity to appeal and be retested. If current employees are tested, there should be a reha­bilitation process available.



(Table 6.5 in the text provides a summary of personnel selection methods and their reliability, validity, generalizability, utility, and legality.)





A Look Back


The chapter opening demonstrates how important it is for organizations to standardize their employee selection process and allow equal opportunities to all applicants.  



Questions



1.         Based on this chapter, what are the best methods of obtaining information about job applicants?



Students’ answers will vary, but could include the following: interviews, references and biological data, cognitive ability tests, and personality inventories are the best methods of obtaining information about job applicants.



2.         What are the best characteristics to look for in applicants, and how does this depend on the nature of the job?



Students could include personality, interpersonal style, job knowledge and skill, and verbal, quantitative, and reasoning abilities as being the best characteristics to look for in an applicant.  Characteristics will differ from job to job because each job requires different situations and abilities (e.g. assembly line workers would not need the same degree of interpersonal skills as a salesperson).



3.         If you could use only two of the methods described in this chapter and could assess only two of the characteristics discussed, which would you choose, and why?



Interviews and references could be the two methods and interpersonal styles and job knowledge and skill could be the two characteristics students might include in their answers.  Interpersonal styles, that could be observed in an interview, and job knowledge and skill, which an employer could be informed of from a previous employer/reference, would be needed for any job and should be known before the applicant is hired.





Chapter Vocabulary



These terms are defined in the "Extended Chapter Outline" section.



Reliability

Validity

Criterion‑related Validity

Predictive Validation

Concurrent Validation

Generalizability

Utility

Situational Interview
Cognitive Ability Tests

Verbal Comprehension

Quantitative Ability

Reasoning Ability





Discussion Questions



1.         We examined nine different types of selection methods in this chapter. Assume that you were just rejected for a job based on one of these nine methods. Obviously, you might be disappointed and angry regardless of what method was used to make this decision, but can you think of two or three methods that might leave you most distressed? In general, why might the acceptability of the test to applicants be an important standard to add to the five we discussed in this chapter?



Students would likely find tests with lowest reliability and validity levels to be most unsatisfactory, such as personality tests and honesty tests. Most students would probably be disturbed by drug tests because of the perceived invasion of privacy. Face validity is indeed a critical issue and applicants should be asked about their perception of various tests that they take.

            

2.         Videotaping applicants in interviews is becoming an increasingly popular means of getting multiple assessments of individuals from different perspectives. Can you think of some reasons why videotaping interviews might also be useful in evaluating the interviewer? What would you look for in an interviewer if you were evaluating one on videotape?





The videotape, of course, can be reviewed to examine the candidate and the capabilities of the interviewer. Factors to observe in interviewer behavior include being prepared to start on time, putting the candidate at ease, asking questions that are clear (questions that do not have multiple questions within each) and that require more than a yes or no answer, not asking illegal questions, asking questions that are job‑related and demonstrate knowledge of the job, and closing the interview appropriately by informing the candidate about what happens next and when he or she will be contacted. If this is a recruitment interview, there should also be a strong description of the organization, pay and benefits, the job and working conditions, and so on.



3.         Distinguish between concurrent and predictive validation designs, discussing why the latter is to be preferred over the former. Examine each of the nine selection methods discussed in this chapter, and determine which of these would have validity most and least affected by the type of validation design employed.



The process of predictive validation means that a test is given and the score is not used for selection (the existing techniques are used to decide on hiring). Then, after six to nine months, performance information is gathered and the test score and performance scores are correlated.

Concurrent validation is a process in which current employees are given tests, performance information is gathered at the same time, and the test and performance scores are correlated.

Predictive validation is preferred, since applicants tend to be more motivated to take the test than current employees (current employees may be threatened by the test). In concurrent validation, one has no way of knowing what was learned on the job and what employees knew or could do before they were hired. Concurrent validation frequently results in restriction of range, since poor performers have probably left and therefore the correlation coefficient is lower. Predictive validation is also preferred by the EEOC.



Students can speculate on how validity may be different for each selection technique, depending on whether predictive or concurrent validation techniques are used.

            

4.         Some have speculated that in addition to increasing the validity of decisions, employing rigorous selection methods has symbolic value for organizations. What message is sent to applicants about the organization through hiring practices, and how might this message be reinforced by recruitment programs that occur before selection and training programs that occur after selection?





            Message to the applicants is clear that the organization wants the individuals best suited for the job and organization with the appropriate qualifications. This message can be reinforced through rigorous recruitment methods as well as offering training programs that fit the job requirements and the needs of the individuals.


Manager’s Hot Seat Exercise: Diversity in Hiring: Candidate Conundrum


I.  Introduction



Interviewing and hiring qualified candidates is an important and prevalent management activity.  This scenario depicts a debate between two managers regarding the hiring of two equally qualified individuals, one Caucasian and one African American.  Instructors of Management or Human Resources will find this scenario to be a valuable platform to discuss legal issues and biases that can influence hiring decisions and the legal ramifications.



II.  Learning Objectives



To assess students’ understanding of biases that may affect the hiring decision.
To analyze and evaluate a novel interviewing and hiring process.
To identify legal issues relevant to hiring.


III.  Scenario Description:



Overview:  Robert Gedaliah has interviewed fifteen candidates to fill the new Customer Outreach Representative position, and narrowed it down to two.  He invited Paul Munez, the customer service team leader, for the second interview of these two candidates.  The interviews proceed smoothly, with both candidates demonstrating appropriate levels of experience, skills and general intelligence.  Paul and Robert will discuss the two candidates and reach a decision.



Profile:

·        Robert Gedaliah is the Founder and President of Beck ‘n Call, a delivery service with a staff of over 200 with many fleets of transportation, and warehousing an eclectic inventory of products. Robert oversees all of the departments, including human resources, operations, and accounting.

·        Paul Munez is the Customer Service Director.  He oversees the customer outreach representatives, and is involved in marketing to the expanding client base.





References:  The references included in the DVD are:

·        Women-of-Color Managers Survey (PPT 14-7)

·        Title VII Civil Rights Act of 1964 (PPT 14-9)

·        Title VII Record of Charges 2002 (PPT 14-10)

·        Title VII:  Definitions (PPT 14-11)



Back History:  Beck ‘n Call is growing at a rapid rate – Robert knows its wise to take a precise and cautious approach towards growth, and scrutinizes all aspects.  He has interviewed 15 people for the new Customer Outreach Rep. and has narrowed it down to two very qualified and intelligent young women.  As is his policy, he has invited the team leader in for the follow-up interview.  He also has a preference for group interviews because the dynamics are richer and therefore better reflect reality.  



Robert and Paul have a very good relationship – when they do have disagreements they are understanding and accepting of their differences and compromise when necessary.  Robert doesn’t think too highly of the swinging bachelor persona, but it hasn’t affected job performance – anyway, it’s diversity that makes life interesting.



Discrimination and diversity issues have become more complex now that the company is over the 50-employee mark. Robert does try very hard to have a diverse staff, although he hasn’t had nearly as many minority applicants for any position, to make his ratios balanced.  This is further complicated by the fact that his customer base itself is increasingly diverse [studies overwhelmingly show that ethnic groups will choose or dismiss a company based on whether they feel their race/ethnicity is well represented and whether there’s a positive and successful diversity policy.]



Scene Set-up: Robert and Paul have finished interviewing two candidates and are meeting to decide whom to hire.



Scene Location: Robert’s office





The Meeting - Summary: During the interviews, Paul’s body language is very engaged and positive toward Sonya, who is Caucasian. He smiles and nods at her and seems to enjoy their interaction.  When Jacqueline, who is African American, is interviewed Paul’s body language is more closed, no smile.  After the interviews Robert indicates that he likes both candidates but leans towards Jacqueline because she had sales experience.  Paul likes Sonya more.  Robert admits that it would be beneficial to hire an African American because there are presently very few minorities who work for Beck n’ Call and none in management.  He reiterates, however, that his reasons are based on merit.  He feels Sonya’s high level of enthusiasm may be a detriment to her work performance because customers may be turned off by it.  He also suggests that Jacqueline would be better choice because she is married.  Paul disagrees but, in the end, Robert states that they will go with Jacqueline and see how it works out.



Afterthoughts – Summary:  Robert says he bases hiring decisions on the person’s “total mix” including experience, education, fit with coworkers and supervisor, personality, etc.  Based on that, he thinks Jacqueline is the best candidate.



Dossier:  The specific artifacts included in the DVD are:

1.      Jacqueline’s Resume

2.      Sonya’s Resume

3.      Beck n’ Call Job Posting



IV.  Discussion Questions:



The References and related Discussion Questions may be found in PowerPoint slides 14-1 to 14-11 on the instructor’s side of the text’s Website.



Learning Objective #1:  To assess students’ understanding of biases that may affect the hiring decision.



Errors in evaluation are often made because of common biases that influence one’s decision making (e.g., halo, horn, contrast, similar-to-me).  What evaluation biases seemed to be taking place in this scenario?


It’s possible that halo error may be affecting Paul’s judgment because he continually references her enthusiasm as her “selling point” but doesn’t offer any other substantive reasons for his preference.  It was also clear from his body language that he liked Sonya which may have been because of a similar-to-me effect.





Robert’s comment that a positive quality in Jacqueline is that she’s married may also be a similar-to-me effect.



´        1.  Robert is suggesting Paul has [when he says he leans towards Sonya]:

A.     A bias

B.     Different taste

C.     Poor judgment



Robert suggests that Paul prefers Sonya because he may be attracted to her.



´        2.  Paul’s point is [when he says Sonya would be a better fit]:

A.     Skewed

B.     Valid

C.     Irrelevant



Given what we saw in the interview and the fact that he doesn’t support his opinion with specific reasons, it is possible that his point is skewed.



What advantages and disadvantages are there when using panel interviews?


While disagreements may occur, this type of functional conflict can improve the decision making process.  Two sets of ears listening to responses and providing interpretations typically leads to a more valid decision.  However, when consensus cannot be reached through a rational discussion based on observed behaviors, someone has to make the final decision.  This may cause hard feelings among panel members.



´        6. Robert’s decision:

A.     Disregards Paul

B.     Is premature

C.     Is wise



Students’ answers will vary.  Paul may very well feel disregarded, whether the decision is wise or not remains to be seen.  The critical issue is whether Paul will embrace this decision and work hard to effectively interact with Jacqueline.





Learning Objective #2:  To analyze and evaluate a novel interviewing and hiring process.



What valid reasons did Robert provide to persuade Paul to choose Jacqueline?


Students’ answers will vary – it will depend on what they consider “valid”.  Discussing Jacqueline’s relevant experience and her job-related personality characteristics is relevant to the hiring decision.  When he brought up the fact she was married, he lost credibility because this fact is not job-related and could be discriminatory to unmarried applicants.  The debate among students will be whether the argument that there are very few African American employees and none in management is valid.



´        3.  The key argument is about:

A.     Qualifications

B.     Ethnicity

C.     Personality



The key argument should be about qualifications and job-related personality characteristics, first.  Ethnicity may also factor into the decision.

´        5. Robert’s argument [that Sonya was too enthusiastic] is:

A.     Relevant

B.     Dodging issue

C.     Inaccurate



This question should generate an interesting debate.  Good interpersonal skills and propensity for working with people is in the job description.  It is possible that over-exuberance may turn off customers.  The scenario doesn’t provide enough information to know for sure.  Robert lost credibility when he revealed a preference for married people so it is possible that Robert is using Sonya’s enthusiasm to dodge the real issue.



What aspects of the Women-of-Color survey are relevant to this scenario (PPT 14-7)?  How could Robert use this to support his position?


The fact that there are African American employees at Beck ‘n Call but none in management positions is relevant to women of color feeling that they don’t have role models in the company.  Thus, Robert’s argument that hiring Jacqueline in this management-training position is valid and may be important for the current employee’s career growth and satisfaction.





Learning Objective #3:  To identify legal issues relevant to hiring.



If Robert made the decision to hire Jacqueline because she was married, would Sonya have a case against Beck ‘n Call under Title VII? (see PPT 14-9 to 14-11).


No, Title VII does not protect an individual based on their marital status.  However, being married or not is typically not job-related, and therefore would be difficult to defend in a court of law.



Is Robert’s decision to hire Jacqueline legal?  Why or why not?


Yes, because Jacqueline and Sonya were at least equally qualified.  The courts have ruled, however, that you cannot hire a less qualified candidate simply because they are a minority unless the organization is rectifying past discriminatory behavior.  Using the argument that diversity is a strategic business necessity (e.g., to promote good will among minority customers) has not held up in court to date (See Taxman v. Board of Education of Piscataway (1993)).



´        4. Hiring based on quotas is:

A.     Illegal

B.     Legal

C.     Sometimes legal



The Supreme Court ruled in City of Richmond v. J. A. Croson Company (1989) that a rigid numerical quota system is unconstitutional.  However, in United Steelworkers v. Weber (1979) an affirmative action plan that included a voluntary quota system agreed to by both management and the union was allowed.  To reach affirmative action goals, targeted recruiting and other plans should be employed and general goals set, not specific quota systems.







Exercising Strategy:  Never Having to Say “You Never Know”



Questions


1.         People applying for jobs are always motivated to display themselves in the best light, and as a result, this can sometimes lead to inaccurate portrayals of abilities, skills, experiences, and personality.  Based upon what you have read in this chapter, how should you approach a job applicant’s written application and résumé if your goal is to make sure that they accurately reflect the person’s past experiences and accomplishments?



Student answers may vary.  Suggestions include standardizing the interview and selection process so that all applicants start on equal ground.  Employers could structure the interview process using situational judgement.  References should be checked accurately and in a timely manner.  The employer could use physical ability or cognitive ability tests.  Personal inventories, work samples, and honesty and drug tests can also help determine an applicant’s abilities and accomplishments.  



2.         In the face-to-face interview process, what steps can be taken to ensure that the applicant is being frank and honest with you, and what steps should you take if you feel that he or she is portraying an inaccurate picture of himself or herself?



Student answers may vary.  Interviews should be structured, standardized, and focused on accomplishing a small number of goals oriented to skills and behaviors that are observable.  Ask questions that force the applicant to display job‑required behaviors or knowledge (situational interview items). Use multiple, trained interviewers who can avoid personal bias.  If the interviewers detect inaccuracy in the way the applicant portrays himself or herself, they can conduct work samples, check references, or ask for drug tests.



3.         Beyond the traditional approaches of going over the application and conducting face-to-face interviews, what other steps can you as an employer take to ensure that the person who is being hired for the job has the right abilities, skills, past experiences, and personality?



            Student answers may vary.  The most important step is to ensure against bias.  This includes non-discrimination of any group.  The person selecting between applicants must be able to use the various methods described in the chapter in ways that are efficient, yet yield excellent selection recruits.





Managing People:  Coming Out in Corporate America



Questions:



1.         The story that opened the chapter focused on sex discrimination at Wal-Mart, and our discussion of the Civil Rights Act focused on race discrimination.  In what ways is discrimination on sexual orientation the same or different from these other types of discrimination?



            Student answers may vary.  Discrimination towards sexual orientation is one of the more recent forms of discrimination, because only recently have homosexuals felt comfortable to come out.  In addition, there are no federal laws that specifically protect against discrimination based on sexual orientation.  



2.         In what ways has the experience of employers with previous dimensions of discrimination left them better able to manage discrimination on the basis of sexual orientation, and in what ways is the challenge associated with discrimination based on sexual orientation different from what employers have experienced in the past?



            Student answers may vary.  Managers with previous dimensions of discriminations may be better able to manage discrimination based on sexual orientation because they may be more sensitive to rights and freedoms of diverse individuals.  Today, it is easier to come out than ever before, and more and more companies are outlawing discrimination based on sexual orientation.  The challenge associated with discrimination based on sexual orientation is slowly diminishing.  



3.         How does the age of the workforce affect implementation of programs aimed at reducing discrimination based upon sexual orientation, and what other factors associated with an organization’s workforce or customer base may complicate program implementation?



            Student answers may vary.  Because openly gay individuals tend to be younger, programs aimed at reducing discrimination based on sexual orientation are more prevalent in companies with a younger workforce.  The level of conservatism, as well as religious affiliation of companies may complicate program implementation.



               





Additional Activities



Teaching suggestions



The following suggestions and activities involve issues, such as race norming and interviewing individuals with disabilities, which are likely to be unfamiliar to students. These issues are likely to raise emotions, and instructors should probably lay some ground rules for discussion. For instance, feedback to other students should not attack the student himself or herself for a particular attitude, but discussion should relate to the diffi­culty in reconciling a wide variety of perceptions in the practice of human resources (in other words, an organization may not be able to). This may upset the students' sense of order, but it is critical that they understand that there are not always "right" answers. For example, the reality that an organization can be sued for reverse discrimination when affirmative action is ordered by a court will confuse students but represents reality.



Inviting other perceptions from students will allow students to under­stand the range of perceptions that may exist and how their particular attitudes fit into legal and societal contexts. The topic of "politically correct" attitudes will arise, and hopefully students can recognize that in an academic environment, everyone has the right to discuss his or her attitudes. All students have the academic requirement to recognize the conse­quences of actions in the organizational context, such as in issues related to sexual harassment.



1.         Discuss with students what special challenges interviewing individuals with disabilities may have.



Interviewing individuals with disabilities will pose special challenges:



Background: ADA covers an estimated 43 million people. As of July 26, 1994, all employers with over 15 employees are subject to the provisions of ADA. There must be no questions on the application blank or within the interview that ask applicants whether they have disabilities.



You can sensitize students to these issues by allowing them to role play an interview with a differently abled individual. It is possible that you could invite such an individual to help the class with this (there are often advocacy groups on campus who could be asked).





You might try the interview "cold" first and then discuss with both the interviewer and interviewee what the problems were and where the interviewer was unsure of how to proceed. The differently abled candidate could present his or her perceptions about what went well and what could be improved. Typically, the differently abled will discuss his or her desire to be treated like everyone else (which, of course, the legislation also emphasizes).



Following are some suggestions provided for interviewing the differently abled:



·                    Small talk might breach the law, such as what the candidate's spouse does for a living or whether the candidate has children (sex discrimination) or where a candidate went to high school (can be related to national origin). Stick to job‑related questions.

·                    Take notes with caution. An attorney recommends not writing down anything that you would not want a jury to see.

·                    You can't ask about health. You can ask the candidate to take a physical exam after making a conditional offer if you routinely make that request of all candidates.



Courtesy issues:



·                    Regarding shaking hands, take what candidates offer you, whether hand or hook.

·                    Do not ask how a candidate became disabled.

·                    With a blind individual: If you have an obstacle‑course office, place the person's hand on the back of the chair or just direct him or her verbally to the chair. If there is a guide dog, the dog will guide the person.

·                    Do not lean on a wheelchair. This is personal space and you could make it roll.

·                    Sit down when talking to wheelchair users.

·                    Do not be overly protective. These persons have typically learned to be independent, and protective behavior will often be perceived as patronizing.

·                    Do not use terms such as handicapped, deformed, deaf, or deaf mute. Acceptable terms are blind, visually or hearing impaired, or multiply disabled.

·                    Do not ask how a disabled individual will get to work or go to the bathroom (again, this is not asked of other candidates).

·                    It is never wrong to ask "Is there anything you need?" but typically you should ask only once.





For additional help:



Guidance is available from the Equal Employment Opportunity Commission's Technical Assistance Manual on the Employment Provisions (Title D of the Americans with Disabilities Act), which explains the law in lay terms. The second of two volumes is a resource directory listing groups that can help with accommodations. Call (800) 669‑EEOC. Hearing impaired individuals can call the TDD number at (800) 800‑3302.



The National Center for Disability Services provides training for inter­viewers across the United States.



Source: M. Hequet, "The Intricacies of Interviewing," Training, April 1993, pp. 31‑36.



2.         Case: Selecting Employees for Work Teams at Levi Strauss & Company.



Levi Strauss is implementing a new manufacturing process at its Blue Ridge, Georgia, plant. The manufacturing process makes extensive use of work teams. The team‑based approach, known as the alternative manufacturing system (AMS), ties compensation and incentives to team goals. In the "old" manufacturing system, employees worked on small parts of garments, such as sewing pockets, then passed the garment to another employee who added another part, such as rivets. Employees did not see the finished product. With the AMS, employees will work in a horseshoe configuration, which provides team members with the opportunity to see each other's work. Team members have to work together to solve problems, such as the pile‑up of unfinished work. In AMS, managers will serve to coach and facilitate. Teams will receive over eight hours training to increase their understanding of production, budgeting, work flow, and product mix. Since Levi-Strauss has made a large investment in a new manufacturing process, they want to make sure that it will work. As a result, they want to choose employees who will help ensure the success of this manufacturing system. This is a nonunionized environment. They will first choose from current employees at the plant, and then look for new hires. Several options that may be used for selection are personality tests, cognitive ability tests, interviews, and mechanical ability tests. Many other options may be possible too.



Pretend that you are the manager overseeing the new manufacturing operation. What recommendations would you have for HRM regarding what process and "tests" to use to select employees for the new manufacturing system? What types of information about the “tests” would you want in order to make you feel confident they would work?





Suggestions



A job analysis that specifies the job requirements should be done first. Students should be encouraged to consider techniques that are primarily job‑related and that focus on behavioral measurements. Since current employees are involved, the organization could consider a multiple‑hurdle approach.



For example, a work sample in a team setting with trained observers assessing team skills such as cooperativeness, persuasiveness, and problem‑solving ability might be an excellent approach. Rather than using cognitive ability tests on current employees, volunteers might be elicited for training and to construct a test that would be content validated. Trainees would have to pass these tests to be selected for the job.

Since there is so much more information available on current employees, an approach that emphasizes training and selection first from the inside might have higher utility than testing external applicants.



The effectiveness of the approach could be examined by a predictive validation process that would correlate scores in the work sample and reining with subsequent performance on the job. If the techniques appear valid, they could be expanded or adapted as needed for use with both internal and external applicants.



3.         Case: Selecting Patient Escorts at City Hospital.



City Hospital is located in the heart of a large Midwestern city. City Hospital has about 1,200 beds and employs 4,500 individuals. The job of patient escort is simple, requiring only minimal training and no special physical talents. When patients need to be moved from one location to another, patient escorts assist in the move. Of particular importance is that patient escorts always take patients who are being discharged from their hospital room to the front door of the hospital.



The job of patient escort is critical to the hospital, since the escort is always the last hospital representative the patient sees. Turnover among the escorts is quite high and has averaged 25 percent in recent years. In addition, upward mobility in the hospital is quite good. As a result, another 25 percent of the escorts typically transfer to other jobs in the hospital each year.





The hospital follows a standard procedure when hiring patient escorts. The HR department reviews the files of applications of persons who have applied for the position. The top two or three applicants are asked to come to the hospital for an interview. The interview is first conducted by HR and then the patient escort supervisor.



During the last two years, the hospital has experienced a number of problems with patient escorts that have affected the hospital's image. Several patients have complained that they have been treated rudely and/or roughly by patient escorts. Others stated that the escorts have been careless when wheeling them out of the hospital to their cars. It is difficult to identify which escorts are responsible for the problems.



As a hospital administrator, you would like to see a selection process implemented that would eliminate the hiring of rude, insulting, or careless patient escorts. Poor escort service is hurting your hospital's quality goals. You have several thoughts about this. Perhaps the application form does not elicit enough information regarding the potential employee's personality (e.g., hobbies, outside activities, likes and dislikes). Perhaps the job candidates are not under enough stress in the interview. It the candidates are under stress, which might give some indication about how they would treat irritable patients. Maybe a personality test could be used to determine if the candidates have the right personality for the job. (Adapted from Nkomo, Fottler, and McAfee, Applications In Human Resource Management [Boston: PWS‑Kent, 1993].)



a.         What recommendations would you have for HR regarding what process and tests to use to select employees for the patient escort position?



Job‑relatedness again is a critical issue. Personality tests, because of their unreliability, might pose legal problems. Focusing on previous experience working with people who are under high levels of stress would be important. Critical incidents in the interview could simulate particularly problematic patient situations to understand what the candidate would do. Since 25 percent of the patient escorts move into different hospital positions, perhaps this position should be seen less as entry‑level and more as requiring special traits. For example, the need for physical strength appears less critical than empathy and patience. Perhaps volunteers, retirees, or older job applicants would be more appropriate sources from which to recruit.





b.         What types of information about the tests would you want to make you comfortable that they would work?



Validity could be examined by predictive‑validation techniques in assessing the value of demographic information such as previous experience and the interview results of critical incidents. It would be helpful (for reasons of quality control) to design a performance review system that could track the performance of patient escorts. A type of concurrent strategy may be used to examine the characteristics and experiences of current, successful job patient escorts and use these characteristics in recruiting and selecting.



4.         Role Play in interviewing: Role-play can be done to illustrate how interviewing may miss critical issues. Students can be placed in dyads or triads ( two role players and one observer) to role play this interview. Students should be cautioned not to step out of role but play it seriously (sometimes role plays are difficult co maintain for long periods of time). Give the role play only to the person designated to play the role immediately before the role play will occur. Give students approximately 15 minutes to prepare to play their roles. Two role descriptions follow.



Applicant Role: Edward Cunningham



You are Edward Cunningham, the former assistant financial controller at

Universal Plastic Company. You were in that position for four years and worked for Universal for six years. Last week, you were unexpectedly fired by your boss at Universal. There were several factors that led to your dismissal. They were:



a.         Three years ago, you and two co‑workers purposely damaged a new high‑speed forming machine at Universal. The company was unable to prove anything, but they strongly suspected that you were trying to sabotage the company. Actually, you made a dumb decision to accept a bribe from an official at National Plastic Company to do this. You did it because you desperately needed the money. Universal ended the matter by giving you a strong oral reprimand that was not documented in your personnel file. Very few people know about this.



b.         Last year, you had three sexual harassment charges filed against you. You felt that the charges were not justified since all you did was "pinch" the girls in the office a little bit. The company had you sign a settlement agreement, and you stopped "pinching" the girls. A written reprimand was placed in your personnel file and nearly everyone knew about in





c.         Two years ago, you failed the state accountant's certification test. Universal requires that financial controllers (and assistants) be certified. You took some extra courses and tried to pass the exam. You took the exam again at the special interim testing two weeks ago and failed again. Normally, the test is given every four years so it will be two more years before you have a chance to take it again. Universal was not willing to wait that long for you to pass.



d.         The final straw that led to your dismissal was the IRS ruling last week. You had implemented some modified accounting procedures at Universal and had saved the company $175,000. However, the procedures were ruled illegal and Universal was fined $450,000 in penalties and back taxes. The company placed nearly all the blame on you.



Three weeks ago you saw an ad for a financial controller position at National Plastic Company. National is Universal's major competitor, and the competition has not only been fierce but downright nasty. You know that National likes to hire people from Universal, so you sent sour resume to John Hathaway, the administrative vice‑president at National. At the rime you sent the resume, you had not been fired and had nor heard from the IRS regarding their ruling. You really want the job at National, and today you are scheduled to have an interview with John Hathaway.



You will try to emphasize all your strengths in the interview, which you feel are the following:



·                    You have a master's degree in accounting (GPA 3.8).

·                    You are (were) the assistant financial controller at Universal and held that position for four years.

·                    You saved Universal $175,000 in the last three years by modifying some accounting procedures.

·                    You were awarded the Outstanding Community Member award by the Chamber of Commerce three years ago.

·                    You recently completed some advanced postgraduate courses in finance.

·                    You have worked for two companies (including Universal) since you graduated from college 10 years ago, and you progressed rapidly in both companies.





You do not want John Hathaway to find out any of the negative information about you; however; basically you are truthful, so you will not lie in the interview. The most important point to disguise is that you were fired by Universal. You will emphasize your strengths in the interview with John Hathaway.



Interviewer Role: John Hathaway



You are John Hathaway, administrative vice‑president of National Plastic Company. The company has nationwide manufacturing locations and distribution centers. Product lines have historically been home‑use plastic products, but the company has recently expanded into the industrial market. The new product lines are specialty plastic products for several industries, which include forest products, steel, and textiles.



As corporate administrative vice‑president, you are in charge of human resources, finance, public affairs, and office systems. Within the finance department are the usual functions, such as payroll, accounts receivable, accounts payable, internal auditing, and accounting. Four weeks ago, your financial controllers suddenly quit and rook a higher‑level position with National's major competitor, Universal Plastic Company. You tried to conduct an exit interview with the former controller, but were unable to get any useful information. The competition has been strong between National and Universal, and this is probably why the former controller was unwilling to talk.

You advertised the position opening for two weeks and received 75 resumes. Based on a review of the resumes, you selected 10 for interviews. You have already interviewed nine of them, and you have not been very impressed with any of them. Nevertheless, one of the nine did have better qualifications than the other eight, and you will offer him the job, if neces­sary. However, the applicant that you expected to be the best is going to come for an interview today. Because of some critically important financial project work that must be done, you will have to make a hiring decision today as soon as the interview is over.

The tenth applicant is Edward Cunningham, who is currently the assis­tant financial controller at Universal Plastic Company. He has an outstanding track record, and following are some of the key points on his resume:



·                    He has a master's degree in accounting (GPA 3.8).

·                    He is the assistant financial controller at Universal and has been in that position for four years.

·                    He saved Universal $175,000 in the last three years by modifying some accounting procedures.



·                    He was awarded the Outstanding Community Member award by the Chamber of Commerce three years ago.

·                    He recently completed some advanced postgraduate courses in finance.

·                    He has worked for two companies (including Universal) since he grad­uated from college l0 years ago, and he progressed rapidly in both companies.



Edward Cunningham looks like an exceptional candidate, and you would like to find out even more details about him in the interview. Since Edward works for Universal (the unfriendly competitor), you will not be able to check his references there. If you did, you know from past experience that the only information you would get would be "name, rank, and serial number." You are not really concerned about Edward's technical knowledge since he has a master's degree, has recently completed some courses and has apparently demonstrated his knowledge on the job. Further, the state normally gives a certification test for accountants every four years, and Edward passed the test six years ago.



Another consideration for the financial controller's position is the fact that Edward is a male. Although you know that a person's sex cannot be specified as a job requirement, it would be more helpful, from an EEO perspective, to have a male in that position. The entire department is 85 percent female, and one of your goals is to have equally balanced depart­ments in terms of race and sex.

该用户从未签到

 楼主| 发表于 2010-10-3 20:11:12 | 显示全部楼层
Chapter Summary



In this chapter, a systematic approach to training and designing effective training systems are discussed, including needs assessments, organizational analysis, company strategy, person analysis, task analysis, creating a learning environment, consideration of employee readiness for training, and transfer‑of‑training issues. Then, numerous training methods are described and reviewed. Next, the evalua­tion of training programs is presented. The chapter concludes with special issues of training, including cross-cultural preparation, managing work force diversity, joint union-management programs, socialization and orientation, and training and pay systems.



Learning Objectives



After studying this chapter, the student should be able to:



1.                  Discuss how training can contribute to companies’ business strategy.

2.                  Explain the role of the manager in identifying training needs and supporting training on the job.

3.                  Conduct a needs assessment.

4.                  Evaluate employees' readiness for training.

5.                  Discuss the strengths and weaknesses of presentation, hands‑on, and group training methods.

6.                  Explain the potential advantages of e-learning for training.

7.                  Design a training session to maximize learning.

8.                  Choose an appropriate evaluation design based on training objectives and analysis of constraints.

9.                  Design a cross-cultural preparation program.

10.              Develop a program for effectively managing diversity.



Extended Chapter Outline



Note: Key terms appear in boldface and are listed in the "Chapter Vocabulary' section.



Opening Vignette: Learning is Business at Nokia



The management approach at Nokia , known as the “Nokia Way”, consists of the Nokia values, its organizational competencies, and its operations and processes used to maintain operational efficiency.  Nokia’ top management is also committed to continuous learning.  



The value of continuous learning translates into personal and professional growth opportunities.  Nokia emphasizes that learning should result in improved operations and better business results.  



I.          Introduction



A.        The opening case study illustrates how companies use continuous learning to support their business strategy and gain competitive advantage.  Besides helping employees learn job skills, training helps the company retain and motivate employees.  From the company’s perspective, training is strategic for business goals related to human resources as well as productivity, customer service, and motivation.  



B.         Training can:

-         increase employees’ knowledge of foreign competitors and cultures,

-         help ensure that employees have the basic skills to work with new technology,

-         help employees understand how to work effectively in teams to contribute to product and service quality.

-         Ensure that the company’s culture emphasizes innovation, creativity, and learning.

-         Ensure employment security by providing new ways for employees to contribute to the company when their jobs change, their interests change, or their skills become obsolete.

-         Prepare employees to accept and work more effectively with each other, particularly with minorities and women.



II.         High‑Leverage Training Strategy: A Systematic Approach



A.                 In general, training refers to a planned effort by a company to facil­itate the learning of job‑related knowledge, skills, or behavior by employees.



High‑leverage training is linked to strategic business goals and objectives, is supported by top management, relies on an instructional design model to ensure the quality of training and to contain costs, and is compared or benchmarked to programs in other organizations.



B.        Continuous learning requires employees to understand the rela­tionship between their jobs, their work units, and the company and to be familiar with the company's business goals.





Example: Johnsonville Foods, an example of continuous learning, gives each employee $100 every year to spend on any type of development activity, work or non-work related.



III.       Designing Effective Training Activities—The training design process refers to a systematic approach for developing training programs (text Table 7.1).



A.                 Needs assessment—Needs assessment helps determine whether training is necessary. There are often pressure points that may suggest that training is necessary (text Figure 7.1). A needs assessment usually involves organizational analysis, person analysis, and task analysis.



1.                  Organizational analysis involves determining the appropriate­ness of training, given the company's business strategy, its resources available for training, and support by managers and peers for reining activities.



a.                   Support of Managers and Peers—The key factors to success are a positive attitude among peers and managers about participation in training activities, willingness to provide information to trainees about how they can apply what they learned, and the availability of opportunities for the trainees to apply what they learned.



b.                  Company Strategy—Business strategy is important for a company to gain competitive advantage.  



-         The business strategy of the firm has a major impact on the appropriate training practices for that firm.  Different business strategies (concentration, internal growth, external growth, and disinvestment) would have different training implications (text Figure 7.1).

-         Strategic training and development initiatives are learning related actions that a company should take to help achieve its business strategy.



Example: SunU, the training and development organization of Sun Microsystems realigned its training philosophy and types of training conducted to be more linked to Sun’s strategy of internal growth.





c.         Training Resources



-         It is necessary to identify whether the company has the budget, time, and expertise for training.

-         A company may use internal consultants or external consultants     for training.



Example: Most of Arthur Andersen's courses for continual professional development are taught by expe­rienced people from line operations. The company also has a division called the Andersen Professional Education Division, which handles its training design, development, and curriculum planning and provides training support.



-         Companies can identify outside consultants and vendors who   can provide training services by using requests for proposals (RFP).  Managers need to evaluate outside consultants on several dimensions before selecting one.  (See Text Table 7.3)



2.                  Person analysis helps the manager identify whether training is appropriate and which employees need training. When a problem, such as a performance deficiency, is identified, it is often unclear whether training is the solution.



a.         Person characteristics refer to the eomployees’ knowledge, skill, ability, and attitudes.



b.         Input relates to the instructions that tell employees what, how, and when to perform.



c.         Output refers to the job’s performance standards.



d.         Consequences are the incentives employees receive for performing well.



e.         Feedback is the information employees receive while they are performing.





f.          From the manager's perspective, training should be considered if any of the following is true:



-         The performance deficiency has the potential to cost the company a significant amount of money.



-         Employees do not know how to perform effectively.



-         Employees cannot demonstrate the correct knowledge or behavior.



-         Performance expectations are clear, there are no obstacles to performance, and nonperformers are not rewarded in some way.



-         There are positive consequences for good performance, while poor performance is not rewarded.   



-         Employees receive appropriate feedback.



-         Other options are too expensive or unrealistic.



3.         Task analysis identifies the conditions in which tasks are performed.  A job is a specific position requiring the completion of specific tasks.  A task is a statement of an employee’s work activity in a specific job.   



A task analysis involves four steps:



a.         Selecting the job to be analyzed.

b.         Developing a preliminary list of tasks by interviewing and observing employees and their managers.



c.         Verifying the importance of the tasks through task invento­ries or expert committees. A task  analysis questionnaire can be used (text Table 7.4).



d.                  Identifying the knowledge, skills, or abilities necessary to perform each task using interview and questionnaires.





B.                 Ensuring Employees' Readiness for Training—Motivation to learn is the desire of the trainee to learn the content of the training program. Managers can ensure this motivation by ensuring the following:



1.                  Self‑efficacy is the employees' belief that they can successfully learn the content of the training program. Managers can increase employees' self-efficacy level by:



a.                   Letting employees know that the purpose of training is to improve performance, not to identify areas where employees are incompetent.



b.                  Providing as much information as possible about the training program and purpose of training prior to the actual training.



c.                   Showing employees the training success of their peers.



d.         Providing employees with feedback that learning is under their control and they have the ability to overcome any learning difficulties



2.                  Understanding the Benefits of or Consequences of Training—Managers should communicate the potential job‑related, personal, and career benefits that employees may receive as a result of the training program.



3.                  Awareness of Training Needs, Career Interest, and Goals— To be motivated, employees must be aware of their skill strengths and weaknesses and of the link between the training program and improvement of their weak skills.



4.                  Work Environment Characteristics—Two critical determi­nants of motivation to learn are situational constraints and social support.



a.                   Situational constraints (lack of proper tools and equip­ment, materials and supplies, budgetary support, and time) are a critical determinant of motivation to learn.





b.                  Social support (managers' and peers' willingness to provide feedback and reinforcement) is a critical determi­nant of motivation to learn.



c.         To ensure that the work environment enhances trainees’ motivation to learn, managers need to:



-         Provide materials, time, job-related information, and other work aids necessary for employees to use new skills or behavior before participating in traning programs.

-         Speak positively about the company’s training programs to employees.

-         Let employees know they are doing a good job when they use training content in their work.

-         Encourage work group members to involve each other in trying to use new skills on the job by soliciting feedback and sharing training experiences and situations in which training content was helpful.

-         Give employees time and opportunities to practice and apply new skills or behaviors to their work.



5.                  Basic skills (reading, writing, and cognitive ability needed to understand the content of training programs) influence an employee's motivation to learn.



Example: The Brenlin Group has on‑site learning centers designed to bring employees to a minimum eighth‑grade reading and math level. Magnavox developed several classes to improve basic skills after learning that 52 percent of its hourly employees were functionally illiterate in reading.



a.                   Managers need to conduct a literacy audit to determine employees' basic skill levels (text Table 7.5).





b.                  Cognitive ability includes three dimensions:  verbal comprehension, quantitative ability, and reasoning ability.  Verbal comprehension refers to the person’s capacity to understand and use written and spoken language.  Quantitative ability refers to how fast and accurately a person can solve math problems.  Reasoning ability refers to the person’s capacity to invent solutions to problems.



c.                   Managers should be sure the readability (the difficulty level of written material) of training material does not exceed that required by the job. A readability assessment usually involves analysis of sentence length and word difficulty.



Competing Through Sustainability



Capitalizing on Local Talent



Companies and workers benefit from training partnerships that prepare economically disadvantaged workers.  



C.                 Creating a Learning Environment—For employees to acquire knowl­edge and skills in the training program and to apply this information in their jobs, the training program needs to include specific learning principles and to provide useful instructional events (see Table 7.6).



1.                  Employees need to know why they should learn.



a.                   Training objectives based on the training needs analysis help employees understand why they need training.  



b.         A training objective has three components:



-         A statement of what the employee is expected to do (performance or outcome).

-         A statement of the quality or level of performance that is acceptable (criterion).

-         A statement of the conditions under which the trainee is expected to perform the desired outcome (conditions).





2.         Employees need meaningful training content.



3.         Employees need to have opportunities to practice.



a.                   If practice is to be effective, it needs to actively involve the trainee and include overlearning (repeated practice).



b.         It is probably best to employ both whole practice (all tasks are practiced at the same time) and part practice (a task is practiced individually as soon as it is introduced) in a training session.



Example: In training retail sales clerks, it is best to have them practice just the ringing up of sales on the cash register and then to practice the whole process, starting with greeting customers and finishing with bagging the purchases and thanking the customers for their business.



4.         Employees need feedback.



a.                   To be effective, feedback should focus on specific behaviors and be provided as soon as possible after the trainees' behavior.



b.                  Positive trainee behavior should be verbally praised or rein­forced.



c.                   One of the most powerful tools for feedback is the use of videotapes in practice sessions.



5.         Employees learn by observing, experiencing, and interacting with others. One way employees learn by observing others is with behavior‑modeling techniques, discussed later in this chapter.



a.                   Communities of practice refer to groups of employees who work together, learn from each other, and develop a common understanding of how to get work accomplished.



6.         Employees need to commit training content to memory.  

Memory works by processing stimuli we perceive through our senses into short-term memory.





7.         Employees need the training program to be properly coordi­nated and arranged.  Training administration refers to coordinating activities before, during, and after the program.



a.                   Good coordination ensures that trainees are not distracted by events (such as an uncomfortable room or poorly orga­nized materials) that could interfere with learning.



b.                  Pretraining communication should let the trainee know the purpose of the training, where it will be held, and whom to contact if he or she has questions.



c.                   The physical arrangement of the training room should complement the training technique and should be physi­cally comfortable.



D.                 Ensuring Transfer of Training—Transfer of training refers to the use of knowledge, skills, and behaviors learned in the training environ­ment on the job.



1.                  Transfer of training is influenced by the climate for transfer, manager support, peer support, opportunity to use learned capabilities, technology, support, and self-management skills (text Figure 7.5).



2.                  Climate for transfer refers to trainees’ perceptions about a wide variety of characteristics of the work environment that facilitate or inhibit use of trained skills or behavior.



3.                  Manager support refers to the degree to which trainees’ managers (1) emphasize the importance of attending training programs and (2) stress the application of training content to the job.  Managers also facilitate transfer through reinforcement (use of action plans).  An action plan is a written document that includes the steps that the trainee and manager will take to ensure that training transfers to the job.  The action plan identifies (1) specific projects or problems that the trainees will work on and (2) equipment or other resources that the manager will provide to help the trainee. (See Table 7.7 in the text)





4.                  Peer support:  Transfer of training can also be enhanced by creating a support network among the trainees.  A support network is a group of two or more trainees who agree to meet and discuss their progress in using learned capabilities on the job.



5.                  Opportunity to use learned capabilities – Opportunity to perform refers to the extent to which the trainee is provided with or actively seeks experience with newly learned knowledge, skill, and behaviors from the training programs.  Opportunity to perform includes breadth, activity level, and task type.



6.                  Technological support:  Electronic performance support systems (EPSSs) are computer applications that can provide, as requested, skills training, information access, and expert advice.



7.                  Self-management skills:  Training programs should prepare employees to self‑manage their use of new skills and behaviors on the job.





E.                  Selecting Training Methods—Survey results indicate that classroom video­, audiovisual, and role-playing are the most frequently used training methods (Figure 7.6).



Example: U.S. companies must carefully consider whether the training methods they use in domestic facilities are appropriate for overseas operations. For example, Asian and Arab employees prefer demonstrations rather than role-playing. Johnson Wax decided not to deliver a teamwork course in Brazil because the culture strongly emphasizes teamwork and the employees would be insulted.



1.                  Presentation Methods refer to methods in which trainees are passive recipients of information.





a.                   Classroom Instruction



-         Classroom instruction typically involves having the trainer lecture a group of trainees. The lecture is often supplemented with questions and answers.

-         It is one of the least expensive, least time‑consuming ways to present information on a specific topic to a large number of trainees.

-         The more active the participation, job‑related examples, and exercises that the instructor can build into the class­room instruction, the more likely trainees will learn and transfer the training to their jobs.



b.         Distance learning involves two‑way communications between people. It includes audioconferencing, videocon­ferencing, and docuconferencing (this allows employees to collaborate on a shared document via computers).



-         Video teleconferencing is used by geographically dispersed companies to provide information to field locations; it usually includes a telephone link so that trainees can ask questions.

-         An advantage of teleconferencing is that the company can save on travel costs.

-         The major disadvantage is the potential lack of interac­tion between the trainer and the trainees. That's why a two‑way communication link is so important as well as on‑site instructors or facilitators.



c.         Audiovisual Techniques

            

-         Audiovisual instruction includes overheads slides, and video.

Video is the most popular.

-         Video has the advantages of allowing the trainer to vary the speed of the presentation, show situations that cannot be easily demonstrated, provide trainees with consistent instruction, and allow trainees to observe themselves.





2.                  Hands‑on methods include on‑the‑job training, simulations, business games and case studies, behavior modeling, and inter­active video. These methods are ideal for developing specific skills understanding how skills and behaviors can be trans­ferred to the job, experiencing all aspects of completing a task, and dealing with interpersonal issues that arise on the job.



a.                   On‑the‑Job Training (OJT)—Companies spend between $90 billion and $180 billion annually on informal OJT, compared with $30 billion on formal off‑the‑job training.



l.    OJT can be useful for training newly hired employees, upgrading the skills of experienced employees when new technology is introduced, cross‑training employees within a department or work unit, and orienting trans­ferred or promoted employees to their new jobs.



2.   The basic philosophy of OJT is that employees learn through observing peers or managers performing the job and trying to imitate their behavior.



3.   OJT takes various forms, including apprenticeships and directed training programs.



b.                  Self‑directed learning involves having employees take responsibility for all aspects of learning, including when it is conducted and who will be involved.



c.                   Apprenticeship is a work‑study training method with both on‑the‑job training and classroom training. To qualify as a registered apprenticeship program under state or federal guidelines, at least 144 hours of classroom instruction and 2,000 hours, or one year, of on‑the‑job experience are required.



1.   Apprenticeships can be sponsored by individual compa­nies or by groups of companies cooperating with a union.



2.   Apprenticeship programs are more widely used in Western European countries than in the United States.





3.   One of the major advantages of an apprenticeship program is that learners have the opportunity to earn pay while they learn.



4.   One disadvantage of apprenticeship programs is that minorities' and women's access to these programs has been restricted because of unions' end employers’ choice of men for entry‑level jobs. Also, there is no guarantee jobs will be available after the program is completed.



d.                  A simulation is a training method that repre­sents a real‑life situation with trainees' decisions resulting in outcomes that mirror what would happen if the trainee were on the job.



1.   Simulations allow trainees to see the impact of their decisions in an artificial, risk‑free environment and are used to teach production and process skills as well as management and interpersonal skills.



2.   Simulators replicate the physical equipment that employees use on the job.



3.   A recent development in simulations is the use of virtual reality technology. Virtual reality is a computer‑based technology that provides trainees with a three‑dimen­sional learning experience. Using specialized equipment or viewing the virtual model on the computer screen, trainees move through the simulated environment and interact with its components.



Example: Motorola has begun preliminary testing on using virtual reality (VR) training purposes. Programmers created a computer model replicating an assembly line and its activities. Through VR, the sights and sounds of the assembly line were recreated. Employees trained via VR technology have scored consistently higher on skills tests than those trained by traditional methods.





4.   A recent development in simulations is the use of virtual reality technology.  Virtual reality is a computer-based technology that provides trainees with a three-dimensional learning experience.  Using specialized equipment or viewing the virtual model on the computer screen, trainees move through the simulated environment and interact with its components.



e.                   Business Games and Case Studies—Situations that trainees study and discuss (case studies) and business games in which trainees must gather information, analyze it, and make decisions are primarily used for management skill development.



f.          Behavior Modeling



1.   Research suggests that behavior modeling is one of the most effective techniques for teaching interpersonal skills.



2.   Each session includes a presentation of the rationale behind the key behaviors, a videotape of a model performing the key behaviors, practice opportunities using role playing, evaluation of a model's performance in the videotape, and a planning session devoted to how to transfer the training to the job.



g.         Interactive Video—Interactive video combines the advan­tages of video and computer‑based instruction.



1.   With interactive video, instruction is provided one‑on-­one to trainees via a monitor connected to a keyboard.  Trainees use the keyboard or touch the monitor to interact with the program.



Example: The UAW‑GM Human Resource Health & Safety Training Center in Madison Heights, Michigan, uses interactive videos in its various training classes, such as the safe operation of a forklift and robotics equipment.





2.   Interactive video is used to teach technical procedures and interpersonal skills.



3.   Interaction video training is individualized, provides immediate feedback, and is convenient.



4.   The main disadvantage is the high cost of developing the courseware.



h.         E-learning refers to training that is delivered on public (Internet) or private computer networks (Intranets) and displayed by a Web browser.  Web-based training supports virtual reality, animation, interactions, communications between trainees, and real-time audio and video.  Web-based training has advantages similar to other multimedia methods (Figure 7.7).



Competing Through Technology



Technology Makes Training an Easy Sale at Cisco Systems



CEO John Chambers believes that e-learning was an important force for helping Cisco recover from its economic woes, for creating strong ties with information technology, and for demonstarting real business results. Cisco believes that the future of the Internet is the convergence of voice, video, and data network into one common network.  



1.   Repurposing refers to directly translating an instructor-led, face-to-face training program online.  



2.   Learner control refers to the ability of trainees to actively learn through self-pacing, exercises, exploring links to other material, and conversations with other trainees and experts.



i.          Blended learning – Many companies are moving to hybrid, or a blended learning approach because of: (1) the limitations of online learning related to technology, (2) trainee preference for face-to-face contact with instructors and other learners, and (3) employees’ inability to find unscheduled time during their workday to devote to learning from their desktops.





j.          Learning portals are websites or online learning centers that provide, via e-commerce transactions, access to training courses, services, and online learning communities from many sources.  



k.         Group‑building methods help trainees share ideas and experiences, build group identity, understand the dynamics of interpersonal relationships, and get to know their own strengths and weaknesses and those of their co‑workers.



l.          Adventure learning focuses on the development of team­work and leadership skills using structured outdoor activities.



1.   It is best suited for developing skills related to group effectiveness such as self‑awareness, problem solving, conflict management, and risk taking.



2.   For adventure learning to be successful, the exercises should be related to the types of skills that participants need on the job, and a skilled facilitator should lead a discussion after the exercise to discuss what happened and how it relates to the job.



3.   The physically demanding nature of adventure learning and the requirements that trainees often have to touch each other in the exercises may increase the company's risk for negligence claims due to personal injury, inten­tional infliction of emotional distress, and invasion of privacy. Also the ADA raises questions about requiring employees with disabilities to participate in physically demanding experiences.



m.        Team training involves coordinating the performance of individuals who work together to achieve a common goal.



1.   Cross-training involves having team members understand and practice each other’s skills so that members are prepared to step in and take another member’s place should he temporarily or permanently leave the team.  





2.   Coordination training involves training the team in how to share information and decision-making responsibilities to maximize team performance.



3.   Team leader training refers to training that the team manager or facilitator receives.



n.         Action learning involves giving teams or work groups an actual problem, having them work on solving it and commit to an action plan, and holding them accountable for carrying out the plan.



-         This is a widespread training practice in Europe, but it is just starting to be used in the United States.

-         Six Sigma Training provides employees with measurement and statistical tools to help reduce defects and to cut costs.



F.                  Evaluating Training Programs



1.                  Training outcomes can be classified into four broad categories: cognitive, skill‑based, affective, results, and return on investment (Table 7.9).



a.                   Cognitive, or learning, outcomes measure acquisition of knowledge through paper and pencil tests or work samples.   



Example:  Safety rules, Electrical principles, Steps in appraisal interview.



b.                  Skill‑based, or behavior, outcomes measure skills and behaviors through observation, work samples, or ratings.



Example:  Jigsaw use, Listening skills, Coaching skills, Airplane landings,



c.                   Affective outcomes measureattitudes, reaction to a program, and motivation through interviews, focus groups, and attitude surveys.





Example:  Satisfaction with training, Beliefs regarding other cultures.



d.                  Results outcomes measure company payoff through observation, and data from information system or performance records.  



Example: Absenteeism, accidents, patents.



e.                   Return on investment (ROI) measures economic value of training through identification and comparison of costs and benefits of the program.  



Example:  Dollars



2.                  Reasons for Evaluating Training



a.                   To determine whether the program is meeting objectives is enhancing learning, and is resulting in transfer of training to the job.



b.                  To determine whether trainees believe that the content and administration of the program were satisfactory.



c.                   To determine the financial benefits and costs of the program.



d.                  To compare the costs and benefits of different training programs to choose the best program.



3.         Evaluation Designs



a.                   Pretest/Posttest with Comparison Group—In this method, a group of employees who receive training and a group who do not are compared.



b.                  Pretest/Posttest—No comparison group is used.



c.                   Posttest Only—Only reining outcomes are collected.





d.                  Time Series—In the time‑series method, training outcomes are collected at periodic intervals before and after training, rather than only once before and after training.



e.                   Factors to consider in choosing an evaluation design include the size of the program, purpose, and the implications if the program does not work. Other factors include the company norms regarding evaluation, costs of the evaluation, and the need for speed in obtaining evaluation information.



4.         Determining Return on Investment—Cost‑benefit analysis is the process of determining the economic benefits of a training program using accounting methods.



a.                   Training cost information is important for several reasons, including controlling costs and comparing the costs for alternative programs.



b.                  Determining Costs



c.                   Determining Benefits—To identify the potential benefits of training, the company must review the original reasons that the training was conducted.



d.                  Making the Analysis—A cost‑benefit analysis is best explained by an example.



IV.       Special Training Issues -- Cross-cultural Preparation



-         Cross-cultural preparation involves educating employees and their families who are to be sent to a foreign country.



-         An expatriate is an employee sent by his or her company to manage operations in a different country.



A.                 Steps in Cross-Cultural Preparation



To be successful in overseas assignments, expatriates need to be:

-         Competent in their area of expertise



-         Able to communicate verbally and nonverbally in the host country.

-         Flexible, tolerant of ambiguity, and sensitive to cultural differences.

-         Motivated to succeed, able to enjoy the challenge of working in other countries, and willing to learn about the host country’s culture, language, and customs.

-         Supported by their families.



B.         Three phases of Cross-cultural preparation



1.                  In the predeparture phase, employees need to receive language training and an orientation in the new country’s culture and customs.  It is critical that the family be included in the orientation programs.



a.                   Cross-cultural training methods range from presentational techniques, such as lectures, to actual experiences in the home country in culturally diverse communities.



b.                  Hands-on and group building methods are most effective in assignments with a high level of cultural and job novelty that require a good deal of interpersonal interaction with host nationals.



2.                  On-site training involves continued orientation to the host country and its customs and cultures through formal programs or through a mentoring relationship.



3.                  Repatriation prepares expatriates for return to the parent company and country from the foreign assignment.  Expatriates and their families are likely to experience high levels of stress and anxiety when they return because of the changes that have occurred since their departure.



Competing Through Globalization



Language Training Helps World Business



Although English is the common language, failing to speak the native language can cause employees to risk being misinterpreted or fail to understand informal conversations.  As a result, many businesses offer language classes for employees during the predeparture phase.







V.        Managing Workforce Diversity

            

A.        Managing diversity involves creating an environment that allows all employees to contribute to organizational goals and experience personal growth.



B.         Managing Diversity through Adherence to Legislation – One approach to managing diversity is through affirmative action policies and by making sure that human resource practices meet standards of equal employment opportunity laws.  This approach rarely results in changes in employees’ values, stereotypes, and behaviors that inhibit productivity and personal development Figure 7.8 in the text).



C.        To successfully manage a diverse work force, companies need to ensure that:



§                     Employees understand how their values and stereotypes influence their behavior toward others of different gender, ethnic, racial, or religious backgrounds.

§                     Employees gain an appreciation of cultural differences among themselves.

§                     Behaviors that isolate or intimidate minority group member improve.



1.                  Diversity training refers to training designed to change employee attitudes about diversity and/or developing skills needed to work with a diverse work force.



2.                  Attitude awareness and change programs focus on increasing employees’ awareness of differences in cultural and ethnic backgrounds, physical characteristics, and personal characteristics that influence behavior toward others.



a.                   Many of these programs use videotapes and experiential exercises to increase employees’ awareness of the negative emotional and performance effects of stereotypes, values, and behaviors on minority group members.



b.                  Criticisms of this approach:





-         First, by focusing on group differences, the program may communicate that certain stereotypes and attitudes are valid.

-         Second, encouraging employees to share their attitudes, feelings, and stereotypes toward certain groups may cause employees to feel guilty, angry, and less likely to see the similarities among racial, ethnic, or gender groups and the advantages of working together.



3.                  Behavior based programs focus on changing the organizational policies and individual behaviors that inhibit employees’ personal growth and productivity.



a.                   One approach of these programs is to identify incidents that discourage employees from working up to their potential.



b.                  Another approach is to teach managers and employees basic rules of behavior in the workplace.



c.                   A third approach is cultural immersion which refers to the process of sending employees directly into communities where they have to interact with persons from different cultures, races, and nationalities.



D.        Characteristics of Successful Diversity Efforts



Increasing evidence shows that attitude awareness programs are ineffective and that one-time diversity training programs are unlikely to succeed.  Table 7.10 in the text shows the characteristics associated with the long-term success of diversity programs.



VI.       Socialization and Orientation



A.                 Through anticipatory socialization, expectations about the company, job, working conditions, and interpersonal relationships are developed.



1.                  A realistic job preview provides accurate information about the attractive and unattractive aspects of the job, working conditions, company, and location to ensure that employees develop appropriate expectations.





B.                 The encounter phase occurs when the employee begins a new job.  Now matter how realistic the information they were provided during interviews and site visits, individuals beginning new jobs will experience shock and surprise.



1.                  Challenging work plus cooperative and helpful managers and peers have been shown to enhance employees' learning a new job.



C.                 In the settling-in phase, employees begin to feel comfortable with their job demands and social relationships.



D.                 Orientation programs play an important role in socializing employees.  Orientation involves familiarizing new employees with company rules, policies, and procedures (text Table 7.13).





A Look Back



As the chapter opener highlighted, Nokia uses training to support the company’s business strategy.  Nokia provides extensive training, encourages continuous learning, and evaluates the value of training.



Questions



1.         Suppose a manager asked you to determine whether training was supporting a company’s business strategy.  How would you conduct this type of analysis?  What kind of information would you look for?



Students’ answers will vary, but could include that they would perform cognitive outcomes to measure what was learned during training, observe learned skills through skill-based outcomes, evaluate employee attitudes and motivation through affective outcomes, identify what employees thought was successful and what inhibited training through reaction outcomes, and use results by dtermining training’s payoff for the company. Students’ answers could include that the information that would be needed to perform the analysis would be paper-and-pencil tests’ results for cognitive outcomes, observation posts for skill-based outcomes, employees’ opinions for affective and reaction outcomes, turnover and accident rates, increased/decreased production, and quality and customer service improvements for results, and what the company standards are for each outcome.







2.         Is there a difference between a company supporting learning and a company supporting training?



            Student answers will vary.  Students may state that learning is largely based on the individuals own motivation, while training is based on company procedures and objectives.





Chapter Vocabulary



These terms are defined in the "Extended Chapter Outline" section.



Training

High‑Leverage Training

Continuous Learning

Training Design Process

Needs Assessment

Organizational Analysis

Person Analysis

Task Analysis

Strategic Training and Development Initiatives

Request for Proposal

Person Characteristics

Input

Output

Consequences

Feedback

Motivation to Learn

Self‑Efficacy

Basic Skills

Cognitive Ability

Readability

Objective

Practice

Overlearning

Communities of Practice

Training Administration

Transfer of Training

Climate for Tranfer



Action Plan

Support Network

Opportunity to Perform

Electronic Performance Support System (EPSS)

Presentation Methods

Hands‑on Techniques

Self‑Directed Learning

Apprenticeship

Simulation

Virtual Reality

E-learning

Repurposing

Learner Control

Group‑Building Methods

Adventure Learning

Cross-training

Coordination Training

Team Leader Training

Action Learning

Six Sigma Training

Training Outcomes

Cost‑benefit Analysis

Expatriate

Cross-cultural Preparation

Repatriation

Managing Diversity

Diversity Training

Attitude Awareness and Change Program

Behavior-based Programs

Cultural Immersion

Organizational Socialization

Anticipatory Socialization

Realistic Job Preview

Encounter Phase

Settling-In Phase





Discussion Questions



1.         Noetron, a retail electronic store, recently invested a large amount of money to train sales staff to improve customer service. The skills emphasized in the program include how to greet customers, how to determine their needs, and how to demonstrate product conveniences.The company wants to know whether the program is effective. What outcomes should they collect? What type of evaluation design should they use?



Students' answers will vary. Noetron could collect affective (reaction) outcomes, cognitive (learning) outcomes, skill‑based (behavior) outcomes, results outcomes, or some combination of these. Both reaction and cognitive outcomes are usually collected before trainees leave the training site. Skilled‑based and results outcomes can be used to determine transfer of training. Since the training objectives are business­-related outcomes, the results outcomes should probably be included.



The type of evaluation design options include pretest/posttest with comparison group, pretest/posttest, posttest only, and time series. Factors to consider in selecting a design are listed in the chapter. The most extensive would be a pretest/posttest with comparison group done in the time‑series method. However, this would also be the most costly.



2.         "Melinda," bellowed Toran, "I've got a problem and you've got to solve it. I can't get people in this plant to work together as a team. As if I don't have enough trouble with the competition and delinquent accounts, now I have to put up with running a zoo. It's your responsibility to see that the staff gets along with each other. I want a human relations training proposal on my desk by Monday." How would you determine the need for human relations training? How would you determine whether you actually had a training problem? What else could be responsible?



Students will probably recommend a needs assessment, assuming Melinda has time for this, to determine if the problem can be successfully addressed by training. The needs assessment would ideally include organizational, person, and task analysis. Some other reasons why the employees are not working as a team could be poor management, an inappropriate reward system that does not reward teamwork, dissatisfaction with salaries, and so on.





3.         Assume you are general manager of a small seafood company. Most of the training that takes place is unstructured and occurs on the job. Currently, senior fish cleaners are responsible for teaching new employees how to perform the job. Your company has been profitable, but recently wholesale fish dealers who buy your product have been complaining about the poor quality of your fresh fish. For example, some fillets have not had all the scales removed, and abdomen parts remain attached to the fillets. You have decided to change the on‑clue‑job training received by the fish cleaners. How will you modify the on‑the‑job training to improve the quality of the product delivered to the wholesalers?



Some options would include selecting only certain fish cleaners to act as trainers, providing an off‑the‑job session for new employees, standardizing the training conducted, and so on.



This question could be used as a brainstorming exercise for the class.



4.         A training‑needs analysis indicates that managers' productivity is inhibited because they are reluctant to delegate tasks to their subordinates. Suppose you had to decide between using adventure learning or interactive video for your training program. What are the strengths and weaknesses of each technique? Which would you choose? Why? What factors would influence your decision?



The strengths of using an interactive video are that it is individualized, it provides immediate feedback, and it is convenient. The main disadvantage is the high cost of developing the courseware.



The strength of using adventure learning is that it tends to develop skills related to group effectiveness such as self‑awareness, problem solving, conflict management, and risk taking. The disadvantages are the inconvenience, the cost, and the physical dangers.



Since delegation is not one of the skills normally emphasized in adventure learning, an interactive video that includes various scenarios of effective and ineffective delegation might be best. However, the cost may be prohibitive.



Some students might suggest role-playing and behavior modeling as better choices.





5.         To improve product quality, a company is introducing a computer­-assisted manufacturing process into one of its assembly plants. The new technology is likely to result in substantial modification of jobs. Employees will also he required to learn statistical process‑control techniques. The new technology and push for quality will require employees to attend numerous training sessions. Over 50 percent of the employees who will be affected by the new technology completed their formal education over 10 years ago. Only about 5 percent of the company's employees have used the tuition‑reimbursement benefit. What should management do to maximize employees’ readiness for training?



Students might suggest such things as providing remedial learning sessions, providing pretraining sessions to prepare employees for the changes, testing of employees to determine their actual skill levels, and so on.



6.         A training course was offered for maintenance employees, in which trainees were supposed to learn how to repair and operate a new complex electronics system. On the job, maintenance employees were typically told about a symptom experienced by the machine operator and were asked to locate the trouble. During training, the trainer would pose various problems for the maintenance employees to solve. He would point out a component on an electrical diagram and ask, "What would happen if this component was faulty?" Trainees would then trace the circuitry on a blueprint to uncover the symptoms that would appear as a result of the problem. You are receiving complaints about poor troubleshooting from maintenance foremen who supervise employees who have completed the program. The trainees are highly motivated and have the necessary prerequisites. What is the problem with the training course? What recommendations do you have for fixing this course?



One idea students might have about the problem with the training course is that the course material is not really job‑relevant. In other words, the situations provided in the class are not the same as the problems found in the field. Another possibility might be a lack of evaluation or posttest to determine if the trainees actually learned what was being presented in the class.



Recommendations would depend on what the problem(s) are; however, some students might recommend a review and change in course content to make it more relevant to the field, more involvement by the machine operators to develop the content, a pretest/posttest to determine the value of the training, and so on.





7.         What factors contribute to the effectiveness of Web training programs?

            

Advantages of Web-based training include the ability to deliver training to trainees anywhere in the world at any time, cost savings and efficiency in training administration, the use of self-directed, self-paced instruction, the ability to monitor trainees’ performance, and controllable access to training.  The program also has advantages from learning and cost perspectives.  Web-based training allows the trainee to have complete control over the delivery of training, provides links to other resources, and allows the trainee to share information and communicate with other trainees and the trainer or to make ‘deposits” into databases.  Web-based training also allows more than one person to access the training materials at the same time (asynchronous training).  In addition, Web-based training has ease of updating the training program using authoritative language (such as html), which continues to become more user-friendly.  





Manager’s Hot Seat Exercise: Working in Teams



I.  Introduction



Cross-functional teams pose unique challenges to team leaders and members.  The “built-in” diversity of cross-functional teams is both an attribute and difficulty when accomplishing group goals.  This scenario depicts a cross-functional team that suffers from lack of buy-in from its members.  This scenario is a valuable teaching tool for instructors of Management, Organizational Behavior, and Human Resources to highlight the challenges of working in diverse teams.



II.  Learning Objectives



To assess students’ understanding of a team and how it develops.  
To identify strategies for improving group cohesiveness.




III.  Scenario Description:



Overview:  The Executive Director of Operations has assigned Joe Tanney the role of Team Leader for a high priority project.  The task is to generate a proposal for streamlining the antiquated blueprint generation process.  Each office is at different stages of computerization, with different teams preferring different technologies for particular tasks.  This makes everything more complex – from collaboration to revisions to cost and time management, and is having a serious impact on the company’s bottom line.  The team includes Rosa Denson, Cheng Jing, and Simon Mahoney.



Profile:

·        Joseph Tanney is the Senior Account Manager and architect at Wolinsky & Williams, a very large international architecture firm with over 400 employees and six offices throughout the world.

·        Simon Mahoney is a Partner and works as an Architect and account Manager.  He oversees some of the firm’s largest projects and has been with the company for 11 years.

·        Cheng Jing is a Junior Architect and a member of a team of architects who specialize in designing office parks.  Jing is currently working towards his Masters degree, and upon receipt of the degree, will be transferred to the design department.

·        Rosa Denson is a Senior Account Manager.  Rosa manages up to 40 accounts at any given time, specializing in international clientele involved in the construction of corporate high rises.  Rosa has been with the company for seven years.



References:  The references included in the DVD are:

·        Types of Groups and Teams (PPT 15-3)

·        Stages of Group Development (PPT 15-4)

·        Process for Group Cohesiveness (PPT 15-7)

·        Model of Team Effectiveness (PPT 15-9)





Back History:  W & W has been expanding at leaps and bounds over the past eight years or so. The growth has been spectacular and hurried – they have not always taken the time to implement change in the most efficient fashion, nor have they opened new offices with any sense of continuity of process or corporate culture.  Business has been plateauing for about eight months now and senior management has decided to take this opportunity to assess efficiencies and practices and make the necessary improvements across the board.  Rumor has it that things may take a swift down turn in the months to come.



These four account managers have been asked to get together by senior management to generate a proposal for streamlining the antiquated blueprint generation process.  Each office is at different stages of computerization, with different teams preferring different technologies for particular tasks.  This makes everything more complex – from collaboration to revisions to cost and time management etc…



The team has been attempting to meet for a couple of weeks but have never been able to agree on a date.  They all agreed to start generating ideas and breaking down the tasks in the meantime – nobody has done it except Joe, who has created a very thorough analysis of the project.  This is their first meeting – they need to plan a course of action, assign tasks and set deadlines.  Simon is the manager with most seniority and everyone assumed he would lead the team.



Scene Set-up: The group gathers to go over Joe’s agenda regarding the team project.



Scene Location: W & W conference room



The Meeting - Summary:  Joe begins the meeting describing the project and soliciting support for the team to get the work done.  Immediately, Cheng and Rosa begin to complain, suggesting they don’t have time to devote to the project.  Joe becomes annoyed and tries to redirect the group to the task at hand.  They go through Joe’s agenda (during which Joe is interrupted by Rosa and asked to speed up the meeting) and then Joe asks for volunteers.  Cheng complains about lack of time again and Rosa says her life is too busy.  Eventually, Cheng, Rosa and Joe take on assignments and Simon announces he has to leave before he is assigned any tasks.





Afterthoughts – Summary:  Joe was annoyed that only Cheng had a copy of the agenda for the meeting.  He also wanted to have everyone commit to and understand the next steps before leaving the meeting.  In retrospect, Joe thinks he should have considered the individuals’ personalities when approaching them with this project.  He thinks he should have made a judgment as to who was most appropriate for each task and then ask them to react to those assignments.  He feels that people in a professional environment need to be able to work in teams and be accountable for getting work done.



Dossier:  The specific artifacts included in the DVD are:

1.      Tanney’s Meeting Agenda

2.      Dillon’s Email to Tanney

3.      Jing’s Email to Denson



IV.  Discussion Questions:



The References and related Discussion Questions may be found in PowerPoint slides 15-1 to 15-9 on the instructor’s side of the text’s Website.



Learning Objective #1:  To assess students’ understanding of a team and how it develops.  



What type of team is this team?  How do you know? See PPT 15-3.


This is a formal team because the senior management has assigned these individuals to the team.  It is intentionally a cross-functional because the project requires integration across areas.  The team members represent different areas within the company.



What stage of group development is this team in? See PPT 15-4.


Forming.  They are just getting together to commence work on this project.  It is clear no one is really bought-in to the project or their expected role.



´        1.  Cheng is off subject.  What should Joe do?

A.     Interrupt

B.     Hear him out

C.     Let Cheng lead





Joe needs to understand the concerns of his team members.  This will help him when allocating tasks and responsibilities.  The fact this happened during the meeting caused it to derail and caused negative affect among members.



´            2.  Joe is frustrated.  He should:

A.     Cool down

B.     Refocus group

C.     Cancel meeting



Joe, first needed to cool down and then refocus the group.  Starting with a goal they could all buy-in to is a critical first step that he didn’t accomplish.



What should Joe have done before, during, and after the meeting to ensure the commencement of the project is successful?


Joe should have, and did, send out an agenda.  He may have talked to each team member individually to understand their perspective on being assigned to this project team (he suggests this in the Afterthoughts).  For example, he could have found out what Simon knew about similar past projects and how to avoid making the same mistakes.  Planning and documenting a clear mission and vision for the project team that is directly linked to the organization’s goals may have provided more motivation for the team members.



During the meeting, he could have explained the purpose of the project and why they had each been chosen and then discussed the project goals and expectations.



After the meeting, he should send out meeting minutes that specifically indicate what each team member is going to do.



Learning Objective #2:  To identify strategies for improving group cohesiveness.



What factors of group cohesiveness were present in this team? See PPT 15-7.


Answers will vary.  Group size typically depends on the nature of the task but 4 to 6 is a good size for a working meeting.  The diversity in terms of the cross-functional nature of the team could have been managed better because the group members didn’t seem to think this project was relevant or important for them.





´        3.  Joe dismisses Rosa. He acted:

A.     Appropriately

B.     Rudely

C.     In humor



He maintained Rosa’s self-esteem by noting that he appreciated her energy towards other work but needed her to focus on this project now.  Again, if he had established a common goal that all could relate to, he would have had an easier time motivating the team.



´        4.  The team is not proactive. Joe should:

A.     Insist they focus

B.     Assign tasks

C.     Continue as is



Students’ answers will vary.  This should provide an opportunity to discuss how to handle difficult team interactions.  At this point in the meeting, he should probably continue as is.  If he assigns tasks, he may alienate the group because they won’t have participated in the decision.



What should Joe do now to ensure the team’s effectiveness and ultimate success?  Use the Model of Team Effectiveness (PPT 15-9) to support your answer.


Students should use the items in the model as a guide for suggesting specific actions Joe could take.  For example, “Reward Systems” should prompt students to suggest ways to motivate team members with relevant rewards.  “Organizational Leadership” – Joe may want the Executive Director of Operations who assigned the project to briefly meet with the project team to offer support and guidance.



´        5. Simon feels the project is futile. Joe should:

A.     Find out why

B.     Debate Simon

C.     Ignore him





Simon makes comments that seem to undermine the value of the project.  This is particularly troubling because he is the most senior member of the team, and, therefore, probably influences other team members.  This attitude could have a serious negative impact on the team so Joe should find out why as soon as possible.  Having this discussion off-line would probably provide Joe with the background information he needs without the potential of “tainting” the whole group.



´        6. Simon is leaving. Joe should:

A.     Stop him

B.     Email him tasks

C.     Say goodbye



Because Simon is a senior member of the team, it would probably not be appropriate to stop him.  He did not seem like he was willing to stay.  Joe should email him tasks, but, more importantly, discuss the project one-on-one with him to gain his perspective.





Exercising Strategy:  “Learning is Key for Smooth Flying”



Questions



1.         What training method(s) should Rockwell Collins use to reach their goal of reducing operating costs as well as improving employees’ ability to attend training?  Why?



            Student answers may vary.  Instructor training can be expensive as well as time consuming for employees.  Options that are low-cost and allow flexibility to employees are workbooks or manuals, videos/CDROM/DVD/Diskettes, and Internet/Extranet/Intranet training programs.  



2.         How would you suggest that the company evaluate training to determine if it is helping the company reach its business goal?



            Student answers may vary.  Some may suggest performing a cost-benefit analysis of the training program.  Others may suggest an internal audit of the program.  Various methods could be used, including questionnaires, surveys, interviews, and paper and pencil tests.







Managing People: From the Pages of Business Week



Look Who’s Building Online Classrooms



The case study examines how online learning is impacting organizations.  The advantages and disadvantages to e learning are discussed.   



Questions



1.                  What features are necessary for on-line learning to be effective?  Explain.



Student reponses to this question will vary.  However, informatin from both the text and the case study should be referenced.  Responses should include:  quick delivery, setting where information can be shared, and make lessons relevant.



2.                  On-line learning blurs the distinction between training and work.  Trainees are expected to be motivated to complete on-line learning during breaks.  As a manager, is this realistic?  How would you schedule on-line learning for your employees?



Student responses will vary. One possible response might be: One would make on-line learning a program that is interesting and fun and that is focused to all members of an organization.  One would have workshops on different areas and intertwine all types of employees together in groups for these workshops.  In developing the program, one would talk to employees to see what they feel are the main issues regarding program scheduling problems and find ways they feel would be the best to deal with them.  Then one would implement those opinions from all employees into my program so that everyone feels that they had a part in implementing the program.  By doing this one will be motivating those employees because they will be more apt to participate in the workshops and activities if they correlate with the opinions the employees gave.  







Additional Activities

Teaching Suggestions



There are several cases and articles that can be used as additional activities when discussing the topic of training. The cases could be assigned to individ­uals as writing assignments or could be assigned to groups to be done inside or outside of class. The articles could also be assigned and discussed in class.



1.                  Harvard Business School Case 9‑488‑016. United Parcel Service (a) by Sonnenfeld and Lazo, Teaching Note 5‑488‑011. United Parcel Service (UPS) in 1987 faced serious challenges to its long‑standing policies of on‑the‑job training and promotion from within. Increased competition in its traditional business of ground transport found UPS lagging in computerization and in need of technical expertise it could not simply cull from within its ranks. Whether, when, and how the new people were to be hired and assimilated and to what extent the UPS culture and/or the new people would have to adapt were the key questions.



Suggested questions for the United Parcel Service case:



(1)   What are the trade‑offs between hiring information services employees from within and training them versus hiring talented people from outside UPS?



(2)   Both the facilities and employees at UPS contributed to a no‑frills, egalitarian, hard‑working supportive atmosphere. Develop a set of interview questions that could be used in an interview by the hiring manager to determine if a potential outside hire would 'fit' in at UPS.



(3)   What did you learn about managing human resources from reading and analyzing this case?



2.                  'Companies That Train Best ' by Ronald Henkoff, Fortune, March 22, 1993. This article could be assigned and students could be asked to write a review of the article, or small groups of students could be asked to discuss the article. The article identifies corporations that have made a significant commitment to training. Have students discuss the similarities and differences in the corporations that are described. Why would a corporation not spend the money for training? How does a corporation decide how much money should be spent on training?





3.                  Cross‑training is an important training concept discussed and explored in "Cross‑Training: The Tactical View" by Margaret Kaerer in Training, March 1993, pp. 35‑39. What are the advantages of cross‑training? What are the disadvantages? Do you think corporations will be doing more cross‑training in the future? Why or why not?



4.                  In the August 1992 edition of HR Magazine, there are two interesting articles on training. The article entitled "Training" by Linda Thornburg describes how four successful companies (Xerox, KPMG Peat Marwick, Parisian, and Aetna) are thinking about training in today's fast‑paced environment. The article entitled "Retraining" by Stephenie Overman discusses how employers and employees can benefit from retraining, but explains that many companies are reluctant to invest. These articles could be assigned together or separately, and students could be asked to discuss the main points in each article.



5.                  The following cases on training can be used by individual students for written analysis or used by small groups. Possible discussion questions are included.



Safety Training

(Raymond Noe, Michigan State University)



Last summer, Capistrano County experienced a rash of injuries to its employees. Although most of the injuries were minor, the company fen that it was imperative to conduct a series of short training programs on safe material‑handling techniques. Bill Noe, road department supervisor, who had recently attended a safety training session sponsored by the Red Cross, conducted the training. The training was completed on a department‑by­-department basis with small groups of 10 to 15 employees attending each session.



At the first of these training sessions, Bill stood up in front of 15 employees and read his presentation in a monotone voice: The entire speech lasted about two minutes. It consisted of the following text:



Statistics show that an average of 30 persons injure their backs on the job each day in this state. None of us wants to become a "statistic. "



The first thing that should be done before lifting an object is to look it over and decide whether you can handle it alone or if help is needed. Get help if there's any doubt as to whether the load is safely within your capacity.





Next, look over the area where you're going to be carrying the object. Make sure it is clear of obstacles. You may have to do a little house­keeping before moving your load. After you have checked out the load and route you're going to travel, the following steps should be taken for your safety in lifting:



1.         Get a good footing close to the load.



2.                  Place your feet 8 to 12 inches apart.



3.                  Bend your knees to grasp the load.



4.                  Bend your knees outward, straddling the load.



5.                  Get a firm grip.



6.                  Keep the load close to your body.



7.                  Lift gradually.



Once you have lifted the load, you’ll eventually have to set it down, so bend your legs again and follow the lifting procedures in reverse.  Make sure that your fingers clear the pinch points.  And, finally, it’s a good idea to set one corner down first.



After Bill’s speech had ended, the employees immediately returned to work.



Question



1.         What do you think about the county's training program? Should it be changed? How?







Outdoor Training for IDS Managers

(Raymond Noe, Michigan State University)



The CEO of IDS believes that outdoor training can improve the managerial skills of IDS managers. As a result, he has made it mandatory that each manager attend an adventure learning session in Brainerd, Minnesota. Eight to twelve managers go through the training at one time. The program begins by having managers discuss the skills they would like to develop. The managers participate in group exercises such as climbing a 12‑foot wall. Individual exercises such as 'trust falls" and crossing a creek while suspended from a basket by a wire are also used. At the end of the program, the managers discuss with a facilitator what they have learned from each exercise.



Questions



1.                  Why would anyone spend money and time to have managers attend adventure learning. (Costs typically range from $2,000 to $4,000 per manager).



2.                  How would you evaluate this type of reining to determine if it "worked?”





The Printing Press

(Raymond Doe, Michigan State University)



You are a production manager for a daily newspaper. Since you have purchased new technology for inserting different sections (sports, variety, etc.) into the newspaper, you have been having problems. Newspapers are often "assembled" in the wrong order (e.g., classified ads first), causing delivery delays and costing you money to produce papers that are “assem­bled" in the correct order. Previously this job has been completed manually with few problems. Now the employees must monitor machines to make sure that the sections are assembled in the right order. The equipment involves reading directions appearing on the computer monitor. Managers and engineers examining the new equipment held training sessions with groups of employees to show them how the new equipment worked.



Question



1.                  What are the potential reasons for the production problems?







Speed Reading for Eligibility Technicians

(Raymond Noe, Michigan State University)



The Department of Social Services represents a large portion of the county's budget and total number of employees. The job class of eligibility technician (ET) is directly responsible for all client contact, policy interpreta­tion, and financial decisions related to several forms of public aid (e.g., ADC, Food Stamps, and General Relief). The County of San Capestrano has 1,100 budgeted ET positions and hires approximately 200 ETs each year. Once hired, they are enrolled in an extensive eight-week training program intended to familiarize them with legal statutes, procedural guidelines, and report/documentation preparation. The program costs approximately $10,000 per trainee.



ETs are faced with having to read a large amount of correspondence, including internal memos, announcements of new and revised policies and procedures, on a daily basis. ETs were complaining that they had difficulty reading and responding to the large amounts of correspondence. For an additional $100 per technician, a speed‑reading module could be added to the existing training program. You decided that the money would be well spent and added the speed‑reading module to the new technician training program.



Preliminary evaluation of the speed-reading module was that trainees liked it. Speed‑reading tests administered before and after training showed that, on average, reading speeds increased 200 percent with no loss in comprehension.



Two months after the last session, you informally asked some of the technicians you supervise who had completed the speed reading module if they were using the speed‑reading principles on the job. They said they were not using them at work, but did use it in their leisure reading at home. When you asked them about using it on the job, the typical response was "I never read those memos and policy announcements anyway!"



Questions



1.                  Was this training a waste of money? What should have been done to insure that the speed‑reading program was necessary?



2.                  How would you determine the objectives of the speed‑reading program? Write several possible objectives.



3.                  What could you do to get the technicians to use the speed‑reading principles on their jobs?





4.                  How would you evaluate the effectiveness of the speed‑reading program?





Note: Eligibility Technician Job Description



Interviews applicants or recipients to determine eligibility for public assistance; interprets and explains rules and regulations governing eligibility and grants, methods of payment, and legal rights to applicant or recipient; records and evaluates personal and financial data obtained from applicant or recipient to determine initial or continuing eligibility according to department objectives; initiates procedures to grant, modify, deny, or terminate eligibility and grants for various aid programs, such as public welfare, employment, and medical assistance; authorizes amounts of grants, based on determination of eligibility for amount of money payments, food stamps, medical care, or other general assistance; identifies need for social services and makes referrals to various agencies and community resources available; prepares regular and special reports as required and submits individual recommendations for consideration by supervisor; and prepares and keeps records of assigned cases.







"Distance Learning: Work and Training Overlap" by Bill Leonard, HRMagazine, April 1966, pp. 41‑47.



Questions



1.         What are the advantages and disadvantages of distance learning?



2.         Why does the author believe emerging economics such as Thailand and Indonesia have an advantage over the United States in adapting technology to train their work forces?

该用户从未签到

 楼主| 发表于 2010-10-3 20:11:27 | 显示全部楼层
Chapter Summary



This chapter examines a variety of approaches to performance management. The chapter begins with a model of the performance-management process that examines the system's purposes. Then, the specific approaches to performance management are discussed, including the strengths and weaknesses of each approach. The various sources of performance information are also presented. Next, the errors resulting from subjective assessments of performance are identified, as well as the means for reducing those errors. Then, the effective components to performance feedback are discussed. Finally, the components of a legally defensible performance management system are presented.



Learning Objectives



After studying this chapter, the student should be able to:



1.         Identify the major determinants of individual performance.

2.         Discuss the three general purposes of performance management.

3.         Identify the five criteria for effective performance‑management systems.

4.         Discuss the four approaches to performance management, the specific techniques used in each approach, and the way these approaches compare with the criteria for effective performance‑management systems.

5.         Choose the most effective approach to performance measurement for a given situation.

6.         Discuss the advantages and disadvantages of the different sources of performance information.

7.         Choose the most effective source(s) for performance information for any situation.

8.         Distinguish types of rating errors and explain how to minimize each in a performance evaluation.

9.         Identify the characteristics of a performance measurement system that follows legal guidelines.

10.       Conduct an effective performance feedback session.



Extended Chapter Outline



Note: Key terms appear in boldface and are listed in the "Chapter Vocabulary" section.





Opening Vignette: Performance Management ABCs: Is This the Way to Manage Human Resources?



In many U.S. companies, performance evaluation systems have generated lawsuits and negative publicity, and have caused poor employee morale.  Forced distribution systems have become popular in U.S. companies, as managers are being asked to differentiate between good, average, and poor performers.  Despite their potential merits, forced distribution systems have been difficult to implement.  



I.                    Introduction



A.        Performance management is the process through which managers ensure that employee activities and outputs are congruent with the organization's goals. Performance management is central to gaining competitive advantage. The performance management system has three parts: defining performance, measuring performance, and feeding back performance information.  (See Table 8.1 in the text)



B.         Performance Appraisal is the process through which an organization gets information on how well an employee is doing his or her job.



C.        Performance Feedback is the process of providing employees information regarding their performance effectiveness.



II.         An Organizational Model of Performance Management (Figure 8.1)



A.        Individual attributes (KSAs) are the raw materials of perfor­mance and lead directly to individual behaviors that lead to objec­tive results.



B.         Another important component of the model is the organization's strategy.



1.         The company must align organizational activities of all units with the organization's strategies and goals.





2.         Performance planning and evaluation (PPE) systems seek to tie the formal performance appraisal process to the company's strategies by specifying at the beginning of the evaluation period the types and level of performance that must be accomplished to achieve the strategy. Then at the end of the evaluation period, individuals and groups are evaluated based on how closely their actual performance met the performance plan.



C.        Situational constraints are another important component of the model. Examples are the organizational culture, work‑group norms, and economic conditions.



Example: Dave is not motivated to exhibit the right behaviors because his peer group is pressuring him to conform to their standards.



III.       Purposes of Performance Management



A.        Strategic Purpose



1.         First and foremost, a performance‑management system should link employee activities with the organization's goals.



Example: If an organization has a goal to promote affirmative action, the performance‑appraisal system should reward managers that hire and promote women and minorities.



2.         Most systems do not achieve this purpose. A 1985 survey found that only 13 percent of companies used their performance­ appraisal system to communicate company objectives.



B.         Administrative Purpose



1.         Performance‑management information is used for such admin­istrative decisions as pay raises, promotions, retention/termi­nation, layoffs, and recognition of individual performance.



2.         Many managers see the performance‑appraisal process as a necessary evil, and they feel uncomfortable feeding back evalu­ations to the employees; thus, they tend to rate everyone high, or at least rate them the same.





C.            





Competing Through Globalization


The Transition from Expatriates to Local Talent

In Western business techniques, the goal of many foreign operations is to turn over control to local managers.  Performance management plays an important role in keeping expatriate performance expectations realistic and providing goals against which their performance can be evaluated.  Performance management is also important for determining whether the expatriate has successfully developed and prepared local staff members to take over the operations.



IV.       Performance Measures Criteria



A.        Strategic congruence is the extent to which the performance-management system elicits job performance that is congruent with the organization's strategy, goals, and culture.



Example: If a regional bank decides to become known as the hallmark of customer service, then the branch managers and tellers should have performance measurements focused on customer relations.



B.         Validity is the extent to which the performance measure assesses all the relevant, and only the relevant, aspects of performance. It is also called "content validity."



1.         Validity is concerned with maximizing the overlap between actual job performance and the measure of job performance (Figure 8.2).



2.         A performance measure is deficient if it does not measure all aspects of performance.





Example: A company's performance measure for managers is deficient because it does not measure such aspects of managerial performance as developing others or social responsibility.



3.         A contaminated measure evaluates irrelevant aspects of perfor­mance or aspects that are not job related.



Example: A company's performance measure would be contaminated if it evaluated its managerial employees based on how physically attractive they were.



C.        Reliability refers to the consistency of the performance measure.



1.         Interrater reliability is the consistency among the individuals who evaluate the employee's performance.



Example: Professor Wagner's teaching evaluations have inter­rater reliability since both her students and her peers who visited her classes rated her above average.



2.         With some measures, internal consistency reliability is impor­tant. This is the extent to which all of the items rated are inter­nally consistent.



3.         The measure should be reliable over time (test‑retest reliability).



D.        Acceptability refers to whether the people who use the performance measure accept it. It is affected by the extent to which employees believe the performance management system is fair. (Table 8.3)



E.         Specificity is the extent to which the performance measure gives specific guidance to employees about what is expected of them and how they can meet these expectations.



Example: Paula, a sales representative for a brokerage firm, is expected to record 25 cold calls per day and call each client on her books every two weeks. She is also expected to make sales of at least $30,000 per month to remain in her position.





Competing Through Sustainability:



Team Performance Helps Get the Bills Paid



To improve cash flow and shorten the billing cycle, Children’s Hospital in Boston designed an incentive program that showed employees the relationship between quarterly cash flow and the number of days a bill spent in the hospital.  The teamwork netted very positive results.  Not only did the hospital improve its cash flow and billing cycle issues after one year, but it also was able to recruit more employees and reduce turnover.



V.        Approaches to Measuring Performance



A.        The Comparative Approach—The comparative approach to perfor­mance management consists of techniques that require the rater to compare an individual's performance with that of others.



1.         Ranking is one of the techniques that arrive at an overall assessment of the individual's performance.



a.         Simple ranking requires managers to rank employees within their departments from highest to lowest.



b.         Alternation ranking consists of a manager looking at a list of employees, deciding who the best is and then who is the worst and so forth.



c.         In the courts, the ranking system has not been looked at favorably.



2.         Forced Distribution—The forced distribution method requires the managers to put certain percentages of employees into predetermined categories. (Table 8.4 in the text)



3.         Paired Comparison—The paired‑comparison method requires managers to compare every employee with every other employee in the work group, giving an employee a score of one every time he or she is considered the higher performer. Employees are ranked by how many points they receive.





4.         Evaluating the Comparative Approach



a.         The comparative approach provides an effective tool when the major purpose is to differentiate employee performance; it virtually eliminates problems of leniency, central tendency, and strictness; it is relatively easy to develop and use; and it is often considered acceptable by the users.



b.         Comparative‑approach techniques are not linked to the strategic goals of the organization; the validity and reliability depend on the raters themselves due to the subjective nature of the ratings, and they lack specificity for feedback purposes.



B.         The Attribute Approach—The attribute approach to performance management focuses on the extent to which individuals have certain attributes (characteristics or traits) believed to be desirable for the company's success.



1.         Graphic Rating Scales



a.         Graphic rating scales can provide the rater with a number of different points (a "discrete" scale) or with a continuum along which the rater simply places a check mark (a "continuous" scale) (Table 8.5 in the text)



b.         Graphic rating scales are easy to develop, but some graphic rating scale systems have been criticized by the courts due to their subjective nature.



2.         Mixed Standard Scales



a.         Mixed standard scales are developed by defining the rele­vant performance dimensions with statements representing good, average, and poor performance. These statements are then mixed together (text Table 8.6).



b.         Raters indicate whether the employee's performance is above, at, or below the statements. A special scoring key is then used to score the employee's performance for each dimension.





3.         Evaluating the Attribute Approach



a.         Attribute approach methods are the most popular methods, are easy to develop, and are generalizable across a variety of jobs and organizations. They can be as reliable and valid as more elaborate techniques if attention is devoted to attrib­utes that are relevant to job performance.



b.         There is usually little strategic congruence between the attribute approach methods and the company's strategy; they usually have very vague standards that result in low validity and reliability; and when raters give feedback, these techniques tend to elicit defensiveness from employees.



C.        The Behavioral Approach—The behavioral approach to performance management attempts to define the behaviors an employee must exhibit to be effective in the job.



1.         Critical Incidents—The critical incident approach requires managers to keep a record of specific examples of effective and ineffective performance for each employee.



Example: On May 5, the marketing manager did not attend the executive committee meeting and did not send a replacement or notify the chairman that he would not be attending.



a.         This approach provides specific feedback to the employee and can be tied to corporate strategy by focusing on inci­dents that support that strategy.



b.         Managers may resent having to keep a daily or weekly log of their employees' behavior, and it is often difficult to compare employees, since each incident is specific to an individual.



2.         Behaviorally anchored rating scales (BARS) specifically define performance dimensions by developing behavioral anchors associated with different levels of performance (text Figure 8.3).





a.         BARS can increase interrater reliability by more specifically defining the performance dimension. They are somewhat easy to use and often readily accepted by managers and employees. They also provide specific performance feed­back, and the performance dimensions can be tied to the corporate strategy.



b.         BARS can bias information recall (behavior that is similar to the anchors is more easily recalled), and they tend to be expensive and time‑consuming. Also, research has demon­strated that managers and subordinates do not make much of a distinction between BARS and trait scales.



3.         Behavioral observation scales (BOS) are a variation of BARS. They are developed from critical incidents but use a larger number of behavioral anchors and require managers to rate the frequency with which the employee has exhibited each behavior during the rating period (text Table 8.7).



a.         BOS consume more of the manager's time than BARS and may require more information than most managers can process or remember.



b.         BOS can be linked to the corporate strategy and were preferred by managers and employees when compared to BARS and graphic rating scales.



4.         Organizational behavior modification (OBM) entails managing the behavior of employees through a formal system of behav­ioral feedback and reinforcement.



a.         The techniques vary, but most have four major components: define key behaviors necessary for job performance, use a measurement system to assess whether these behaviors are exhibited, inform employees of those behaviors, and provide feedback and reinforcement to employees.





b.         OBM techniques have been used in a variety of settings, and some studies show significant increases in performance (text Figure 8.4). They tend to increase the frequency of desired behaviors, provide specific guidance to employees, and can be tied to the company strategy.



Example: OBM techniques have been successfully used in several organizational settings to improve safety behaviors (wearing safety equipment, observing safety procedures, etc.). Performing correct safety behaviors was rewarded and not performing correct safety behaviors was punished.



5.         Assessment centers can be used for measuring managerial performance. During an assessment, individuals usually perform a number of simulated tasks, and assessors observe and evaluate the individual's skill or potential as a manager.



a.             Assessment centers tend to provide a somewhat objective measure

of an individual's performance at managerial tasks; they tend to be reliable, valid, and highly accepted; and they also provide specific feedback to the individual being assessed.     



Example: The California Highway Patrol Academy uses a three‑and‑one‑half‑day assessment center to help individ­uals develop a career plan.



b.         Assessment centers are expensive to conduct and do not assess how managers actually perform on a daily basis.

6.         Evaluating the Behavioral Approach





a.         Evaluating the Behavioral Approach

The behavioral approach can link the company's strategy to specific behaviors; it provides specific feedback; its dimen­sions are usually job relevant, which leads to validity. Since people using the system are involved in developing the measures, the acceptability is high, and with appropriate training the techniques are reasonably reliable. The major weaknesses have to do with the organizational context of the system. The behaviors must be constantly monitored to ensure they are linked to the corporate strategy, and the approach is not suited for complex jobs where there are multiple ways to achieve success.



D.        The Results Approach—The results approach to performance management focuses on managing the objective, measurable results of a job or work group. This approach assumes that subjectivity can be eliminated from the measurement process and that results are the closest indicator of one's contribution to organizational effectiveness.

            

1.         Management by Objectives (MBO)



a.         Management by objectives (MBO) is a joint goal‑setting process in which goals are agreed upon between the managers and each subordinate. These goals then become standards used to evaluate the individual's performance. This goal‑setting process cascades down the organization so that all managers are setting goals that help the company achieve its goals.



b.         MBO systems have three common components: (1) they require specific, difficult, objective goals; (2) the goals are set jointly; (3) the manager gives objective feedback throughout the rating period to monitor progress toward the goals (text Table 8.8).



c.         MBO usually increases productivity and is most effective when there is substantial commitment from top manage­ment. MBO systems effectively link employee performance with the firm's strategic goals.





2.         Productivity Measurement and Evaluation System (ProMES)



a.         Productivity measurement and evaluation system (ProMES) consists of four steps: (1) identify the objectives the organization expects to accomplish; (2) the staff defines indicators of performance; (3) the staff establishes the contingencies between the indicators and the level of perfor­mance; (4) a feedback system is developed that provides employees with information about their specific level of performance on each of the indicators.



b.         ProMES is somewhat new, but some research has shown increased productivity (text Figure 8.5). ProMES also tends to be an effective feedback system.



c.             ProMES, however, is time‑consuming to develop.



3.         Evaluating the Results Approach



a.         The results approach minimizes subjectivity, is highly acceptable to both managers and employees, and links an individual's results with the organization's goals.



b.                  Objective measurements used in the results approach can be both contaminated and deficient; individuals may focus only on aspects of their performance that are being measured; and the feedback may not help employees learn how they need to change.



E.         The Quality Approach—Two fundamental characteristics of the quality approach are a customer orientation and a prevention approach to errors. The quality manage­ment approach to performance management suggests that the major focus should be to provide employees with feedback about areas in which they can improve (areas over which employees have complete control).  



1.         Total quality management approach to performance management is a cooperative form of doing business that relies on the talents and capabilities of both labor and management to continually improve quality and productivity.





2.         A performance management system designed with a strong quality orientation can be expected to:



a.         Emphasize an assessment of both person and system factors in the measurement system.

b.         Emphasize that managers and employees work together to solve performance problems.

c.         Involve both internal and external customers in setting standards and measuring performance.

d.         Use multiple sources to evaluate person and system factors.



3.         Subjective Performance Evaluation feedback should be based on such dimensions as cooperation, attitude, initiative, and communication skills. Performance evaluations should include a discussion of the employee's career path, should avoid providing an overall evaluation, and should not be linked to compensation. Compensation rates should be based on prevailing market rates of pay, seniority, and business results, which are distributed equitably to all employees.



4.         Objective Performance Evaluation feedback--Statistical process quality control techniques are used to provide employees with an objective tool to identify causes of problems and potential solutions.



a.                   Process‑flow analysis involves identifying each action and decision necessary to complete the work.



b.                  Cause‑and‑effect diagrams identify events or causes that result in undesirable outcomes.



c.                   A Pareto chart is used to highlight the most important cause of a problem. Causes are listed in decreasing order of importance (text Figure 8.6).



d.                  Control charts involve collecting data at multiple points in time so that employees can identify what factors contribute to an outcome and when they tend to occur. (Text Figure 8.7).



e.                   Histograms are used for displaying distributions of large sets of data and allow data to be grouped into a smaller number of categories (text Figure 8.8).





f.                    Scattergrams show the relationship between two variables, events, or different pieces of data.



3.         Evaluation of the Quality Approach



a.                   The quality approach relies primarily on a combination of the attribute and results approaches to performance measurement. The quality approach adopts a systems‑oriented focus in which traditional performance systems focus on individual employee performance.



b.                  Some companies would not like the idea that quality perfor­mance appraisals are not tied to compensation. The quality approach advo­cates evaluation of personal traits that are difficult to relate to performance unless the company uses a work‑team structure.



Example: Ensoniq Corporation is a privately held company that manufactures electronic musical instruments. It employs approximately 200 people. Under its quality approach to performance measurement, Ensoniq believes that performance reviews and compensation should not be linked. The wage and salary guide states that the "purpose of the performance reviews is to provide a dialogue between the employee and the supervisor, whereby the employee's performance can be enhanced."



4.         Evaluation of Approaches to Performance Measurement—Table 8.9 summarizes the various approaches to measuring performance based on the criteria set forth earlier (strategic congru­ence, validity, reliability, acceptability, and specificity) and illustrates that each approach has strengths and weaknesses.

The Comparative Approach   The Attribute Approach The Behavioral Approach The Quality Approach     The Results Approach   

VI.       Choosing a Source for Performance Information



A.        Managers are the most frequently used source.



1.         Supervisors usually have extensive knowledge of the job and have had the opportunity to observe employees.





2.         Because supervisors have something to gain from employees' high performance and something to lose by low performance, they have the motivation to make accurate ratings. Also, feed­back from supervisors is strongly related to performance.



3.         However, in some instances, supervisors do not observe employees, and supervisors may be biased against a particular employee.



B.         Peers



1.         Peers, or coworkers, are excellent sources when supervisors don't have the opportunity to observe employees.



2.         Peers may be biased, however, and may be uncomfortable with being the rater when the ratings are used for administrative decisions.



Example: Digital Equipment Company uses team appraisals. A performance appraisal committee is established, which includes the person being evaluated, the chairperson chosen by the person being evaluated, an internal management consultant, and two randomly selected team members.



C.        Subordinates



1.         Subordinates are a valuable source of performance informa­tion for managers. They often have the best opportunity to evaluate how well a manager treats employees.



2.         Allowing subordinates to rate managers may put managers in a position where they emphasize employee satisfaction over productivity. Also, subordinates may fear retribution from their managers if the evaluations are not anonymous.



3.         It is best to use subordinate evaluations only for developmental purposes.



D.        Self



1.                  Self‑ratings can be valuable but are not usually used as the sole source of information.





2.                  One problem with self‑ratings is a tendency toward inflated assessments, especially if ratings are used for administrative decisions.



3.                  The best use of self‑ratings is as a prelude to the performance feedback session to make employees think about their perfor­mance and to focus discussion on areas of disagreement.



E.         Customers



1.                  In some instances, the customer is often the only person present to observe the employee's performance.



2.                  The main weakness of customer surveys is that they are some­what expensive.



F.         The best source of performance information



1.         One should choose the sources that provide the best opportu­nity to observe employee behavior and results. (Table 8.10 summarizes this information.)



2.         One popular trend in organizations is called 360‑degree appraisals.  This technique consists of having multiple raters (boss, peers, subordinates, and customers) provide input into a manager’s evaluation. (This technique is discussed in Chapter 9.)



Competing Through Technology


Online Goals Provide Clear Line of Site



This case scenario discusses how several companies are using technology to ensure that employees understand the companies’ priorities and that their efforts are aligned with the companies’ business objectives.  Some companies like Seagate use annual performance reviews, and other companies like Baxter Healthcare have an automated goal-alignment system.  Employees tend to appreciate these performance systems.





VII.      Rater Errors in Performance Measurement



A.        Rater Errors



1.         “Similar to Me” is the error we make when we judge those who are similar to us more highly than those who are not.



Example: Diana tends to rate Beth higher than Paul because Beth is also a single mother of a toddler and Diana and Beth are about the same age. Paul is much older than Diana; he has no children and has never been married.



2.         Contrast errors occur when we compare individuals with one another instead of with an objective standard.



Example: Jason and Angela both do above‑average work, but Jason’s boss tends to rate him as excellent because he performs better than Angela.



3.         Distributional errors are the result of a rater’s tendency to use only one part of the rating scale



a.         Leniency occurs when a rater assigns high ratings to all employees.

b.         Strictness occurs when a manager gives low ratings to all employees.

c.         Central tendency reflects that a manager rates all employees in the

middle of the scale.      



4.         Halo/Horns



a.         Halo errors occur when one positive performance aspect causes

the rater to rate all other aspects of performance positively.   



Example: Nicole tends to get high ratings in quantity and quality of work because she is so friendly and cooperative with everyone and never misses work.





b.         Horns error occurs when one negative aspect results in the rater

assigning low ratings to all the other aspects.   



B.         Reducing Rater Errors



1.                  Rater error training attempts to make managers aware of rating errors and helps them develop strategies for minimizing those errors.



2.                  Rater accuracy training, or frame‑of‑reference training, attempts to emphasize the multidimensional nature of perfor­mance and thoroughly familiarizes raters with the actual content of various performance dimensions.



C.        Appraisal Politics



1.                  Appraisal politics refers to evaluators purposefully distorting ratings to achieve personal or company goals.



2.                  To reduce appraisal politics, companies can do many things to keep evaluators from distorting results. See Table 8.3 in the Text.



VIII.     Performance Feedback

            

A.                 Performance Feedback‑The performance feedback process is complex and

provokes anxiety for both the manager and the employee.



IX.       The Manager’s Role in an Effective Performance Feedback Process   



A.        Ways to Improve Performance Feedback Process


1.         Feedback should be given every day, not once a year.



a.         Managers have a responsibility to correct performance defi­ciencies immediately upon becoming aware of them.



b.         A major determinant of how effectively a feedback session goes is the extent to which the subordinate is not surprised by the evaluation.





2.         Create the Right Context for Discussion



a.         Managers should create a neutral location for feedback.



b.         Managers should describe the meeting as an opportunity for the employee.



c.         Manager should also encourage open dialogue.



3.         Ask the employee to rate his or her performance before the session.



a.         Self‑ratings require employees to think about their perfor­mance, including their weaknesses.



b.         The feedback session can go more smoothly by focusing discussion on areas of disagreement.



c.         Employees will be more likely to participate fully in the feedback session.



4.         Encourage the subordinate to participate in the session.



a.         Even though the problem‑solving approach is the superior approach, most managers still use the tell‑and‑sell or the tell‑and‑listen approach.



b.         When employees participate in the feedback session, they are consistently satisfied with the process.



5.         Recognize effective performance through praise.



6.                  Focus on solving problems.



7.                  Focus feedback on behavior or results, not on the person.



8.                  Minimize criticism.



9.         Agree to specific goals and set a date to review progress.





X.        What Managers Can Do to Manage the Performance of Marginal Employees


Marginal employees are those employees who are performing at a bare minimum level due to a lack of ability and/or motivation to perform well. Table 8.12 identifies four types of employees (solid performers, misdirected efforts, underutilizers, and deadwood) and actions for the managers to take.



XI.       Developing and Implementing a System That Follows Legal Guidelines


A.                 In discrimination suits, the plaintiff often claims that the perfor­mance ratings were subjective and that the rater was biased and influenced by gender or racial stereotypes



1.         Research has shown that both white and black raters give higher ratings to members of their own race, even after rater training.



2.         Discriminatory biases in ratings are worse when one group makes up a small percentage of the work group.



B.                 In unjust dismissal suits, the plaintiff often claims that the dismissal was for reasons other than those the employer claims. The court case will likely focus on the performance system that was used as the basis for claiming the employee's performance was poor.



There are certain characteristics of a system that will withstand legal scrutiny.



1.                  A valid job analysis that ascertains the important aspects of job performance.



2.                  A system based on either behaviors or results.



3.                  Raters trained in how to use the system.



4.                  Some form of review by upper‑level managers and a system for employee appeal.



5.                  Some form of performance counseling or corrective guidance to help poor performers.   



6.                  Multiple raters should be used.





XII.      Use of Technology for Performance Management:  Electronic Monitoring



1.                  Employees’ performance ratings, disciplinary actions, and work-rule violations can be stored in electronic databases.



2.                  Legislation regarding computer monitoring may occur in the future.



A.        Using performance management applications for decision making



1.                  Performance management applications are available to help managers tailor performance problems.



A Look Back


The chapter opener on forced distribution highlighted the problems that companies are having with forced ranking appraisal systems that are used to differentiate above-average, average, and below-average employees. Companies remain challenged to find a performance management system that is acceptable to employees and managers alike yet allows managers to make important administrative decisions.



Questions



1.         Based on what was covered in the chapter, what type of performance system would you recommend that Goodyear, Ford, or Dow Chemical adopt to improve managers’ performance and ability to deal with future business challenges?



Students’ answers will vary, but could include: ProMES would be recommended for these companies because it is a motivator and it is a way to measure and feed back productivity information to the employees.  If using a ProMES, the companies’ objectives/activities, indicators of the products, contingencies between the amount of the indicators and level of evaluation associated with that amount, and the feedback system are already set in the beginning, so managers should be able to deal with future business challenges better.





2.         What advantages will your system have over their current systems?



Students’ answer will vary, but may include the following information.  ProMES would be more effective than their current system because it increases productivity and is an effective feedback mechanism.  The current system only gives feedback in the for of “Top Achiever,” “Achiever,” and “Improvement Required,” which doesn’t give constructive feedback until after the grades are given and when it may be too late to correct any problems.


Chapter Vocabulary


These terms are defined in the "Extended Chapter Outline" section.



Performance Management

Performance Appraisal

Performance Feedback

Performance Planning and Evaluation (PPE) Systems

Strategic Congruence

Validity

Reliability

Acceptability

Specificity

Assessment Centers

360‑Degree Appraisals

Appraisal politics

Marginal Employee



Discussion Questions



1.         What are examples of administrative decisions that might be made in managing the performance of professors? Developmental decisions?



Examples of administrative decisions for professors include pay raises, promotions, tenure decisions, layoffs, and recognition of individual performance. Developmental decisions include working with professors who tend to be having problems with teaching, dealing with students, research productivity, or dealing with colleagues.





2.         What would you consider the strategy of your university (e.g., research, undergraduate teaching, graduate teaching, combination)? How might the performance‑management system for faculty members fulfill its strategic purpose of eliciting the types of behaviors and results required by this strategy?



Students' answers will vary depending on the university, and some students may not be aware of what the university's strategy is; however, the performance‑management system should emphasize the university's strategy. For instance, a research institution should emphasize research in the performance system for professors.



3.         If you were developing a performance‑measurement system for faculty members, what type of attributes would you seek to measure? Behaviors? Results?



Some examples of desirable attributes for professors might be ability to get along with others, responsibility, maturity, self‑confidence, dependability, and so forth.

Some examples of desirable behaviors for a professor might be that he or she keeps office hours, starts and ends classes on time, listens to students, speaks clearly in front of class, and so forth.

Some examples of desirable results for professors might be the publication of research papers, above‑average student ratings, and so forth.



4.         What sources of performance information would you use to evaluate faculty members' performance?



Sources of performance information for professors could include students, advisees, colleagues, department heads, and self‑ratings.



5.         The performance of students is usually evaluated with an overall results measure of grade‑point average. How is this measurement contaminated? How is it deficient? What other measures might you use to more adequately evaluate student performance?



A contaminated measure evaluates irrelevant aspects of performance or aspects that are not job related. A student's overall grade point average will normally include grades for courses that are not job relevant.



A deficient measure does not measure all aspect of performance. A student's overall grade‑point average does not reflect the student's leadership experiences or social experiences with various clubs and organizations on campus. The grade‑point average also does not include experience gained from internships and various class projects.





Other measures that might be used to evaluate student performance include written evaluations from professors and an analysis of courses taken that are particularly relevant to a specific job.



6.         Think of the last time that you had a conflict with another person, either at work or school. Using the guidelines for performance feedback, how would you provide feedback to that person regarding his or her performance in a way that is effective?



Students might suggest focusing on problem solving and trying a problem‑solving approach instead of a tell‑and­ sell or tell‑and‑listen approach, focusing feedback on the behavior and not the person, minimizing criticism, and recognizing effective performance through praise.



7.         Explain what fairness has to do with performance management?



The three categories of fairness are procedural fairness, interpersonal fairness, and outcome fairness. In procedural fairness, managers should give employees the opportunity to participate in the development of the system, ensure consistent standards, and minimize rating error and biases. In interpersonal fairness, managers should have timely and complete feedback, allow employees to challenge the evaluation, and provide feedback in an atmosphere of respect and courtesy. In outcome fairness, managers should communicate expectations regarding performance evaluations and standards and communicate expectations regarding rewards.



8.         Why might a manager intentionally distort appraisal results? What would you recommend to minimize this problem?



Managers might purposefully distort appraisal because managers are accountable for the employees being rated, there might be competing goals within the organization, and if there is a direct link between performance appraisals and highly desirable rewards. To minimize this, managers can adhere by Table 8.2 in the book as well as:



·                    Train managers on the appropriate use of the system

·                    Build top management support

·                    Recognize employee accomplishments that are not self promoted

·                    Have a open culture within the organization





9.         Can computer monitoring of performance ever be accepted to employees? Explain.



Student answers will vary. Explore with students both sides of the issue; Company side—monitoring performance to weed out poor performers and reward top performers; affects the productivity and overall competitiveness; employee side—trust and privacy issue.


Manager’s Hot Seat Exercise: Project Management: Steering the Committee



I.  Introduction



Conflict among work groups is a common problem in organizations.  This scenario depicts an organizational conflict and highlights different conflict management styles.  A discussion of these styles and their effectiveness would be valuable for students in Management or Organizational Behavior courses studying organizational conflict.



II.  Learning Objectives



To assess students’ understanding of the sources of conflict and the conflict process.
To analyze and evaluate different conflict management styles.
To identify ways to ensure functional, rather than dysfunctional, conflict occurs.


III.  Scenario Description:



Overview:  Three months ago, Patrick was given goal of implementing a computerized tracking system on the factory floor with the expectations that it would raise quality control and workflow by a significant percentage.  The team spends an enormous amount of time revising/fixing chips after they fail quality control testing – mostly due to lack of information.  There would be a serious review of the project after the first phase is complete – that is now.  



Patrick is called to a meeting with three top executives.  The tracking system project is very costly and has not been proven, although the company was gung-ho, because their competitors were using all sorts of computerized systems in their production facilities.  Patrick has spent a little time writing a report, but is feeling generally confident.





Profile:

·        Patrick Bennett is the Managing Supervisor of Production at TechBox, a desktop design software house, who oversees production and supervises a floor of 50 employees.

·        Morgan Baines is the Vice President of Distribution.  He works at corporate headquarters and builds relationships with potential clients in expanding markets.

·        Lucinda Bergen is the Director of Product Management.  She oversees all production at TechBox’s manufacturing factories.  She works directly with the factory managers and reports to corporate headquarters.

·        Sam Adelson is the VP of Operations.  His responsibilities include overseeing department managers at the manufacturing factories and corporate headquarters.  He also acts as a liaison between his company and the officers of a number of high-end computer companies.



References:  The references included in the DVD are:

·        Sources of Conflict (PPT 9-3)

·        The Conflict Process (PPT 9-4)

·        Conflict Management Styles (PPT 9-7)

·        Structural Approaches to Conflict Management (PPT 9-9)



Back History: TechBox produces a specialized computer chip for a large client base of high-end computer companies such as Dell, IBM, and Compaq.  Within the TechBox compound are the manufacturing factories and the corporate headquarters.      



Patrick Bennett is the new hotshot manager overseeing the production of the main component of the chip.  He has started to implement a computerized tracking system to increase quality control and workflow.  He supervises a floor of 50 workers.  Patrick has been working at TechBox for 2 years and has been widely praised for numerous successes and improvements on the factory floor – improvements with very positive financial results.  He is young and motivated, possessing a unique comprehension of production/engineering knowledge and management sensibilities.  He is greatly admired by his team and generally received well by top senior management.   



Sam Adelson has worked his way up from the mailroom.  He does not embrace change and is skeptical of young hot shots and young people in general.  He also does not embrace computerized system enhancements, which is odd for a man in high tech.  In a nutshell, he’s stodgy, grumpy, and difficult.





Lucinda Bergen is the only female executive at TechBox.  She is generally compassionate, understanding, and easy going, but is quick to defend her position within the company and over assert herself when she feels confronted.  She is very comfortable with young, talented people and has instituted many technological systems for workflow improvement within her own department.  



Morgan Baines is the class clown and jokester of the company.  He speaks without thinking – supports, attacks, and withdraws without rhyme or reason.  Everyone always has the impression that he’s on their side; the kind of guy who agrees with the last person who spoke.  He’s been around very many years, successfully overseeing product distribution – between wisecracks.



Scene Set-up: A meeting is held with key stakeholders of Patrick’s project to discuss the status of it in terms of time and budget.



Scene Location: TechBox conference room  



The Meeting - Summary:  Stakeholders in the project note their concern with the project going over budget and missing deadlines.  Patrick, the project manager, defends himself and assures them that everything will work out because the longest phase of the project is complete.  He cites his past track record and says the budget may go over just a little but it will be ok.  The other team members say that going over budget is not an option.  He concedes and says he will take a look at exactly how he can improve the progress of the project.  They agree to meet again in a week for a status check.



One Week Later - Patrick assures the group that they are now firmly under budget and plan to be done on time.  Lucinda mentions that Frank, one of Patrick’s employees, said he thought the project was going to be late.  Patrick defends himself by saying that Frank’s role would not allow him to be privy to all the information that Patrick knows and that he has this job for a reason – his talent and prior track record.  Sam suggests supporting Patrick with an additional staff member.  Patrick doesn’t think that would be necessary.  Instead he suggests a daily email to the group updating the project in terms of budget and deadlines.  They compromise by having the “1st deputy” be responsible for sending those daily emails and keeping track of all the relevant numbers.





Afterthoughts – Summary:  Patrick doesn’t think the meeting went very well.  He says he didn’t feel listened to and was upset that the group was more persuaded by people indirectly involved in the project than by him.  He felt he did ok and tried to work with them on an individual basis.  In the second meeting, he said if he hadn’t stood his ground they would have “squished” him.



Dossier:  The specific artifacts included in the DVD are:

1.      Bennett’s Progress Report 5/6/02

2.      Bennett’s Progress Report 5/31/02

3.      Voice message from Frank Jambey to Lucinda Bergen



IV.  Discussion Questions:



The References and related Discussion Questions may be found in PowerPoint slides 9-1 to 9-10 on the instructor’s side of the text’s Website.



Learning Objective #1:  To assess students’ understanding of the sources of conflict and the conflict process.



1.      What sources of conflict were present in this scenario?  Refer to PPT 9-3 to construct your answer.



Many sources of conflict come into to play here.  Ambiguous rules may have been a source of conflict because Patrick didn’t seem to think that going over budget a little was going to be a big problem.  He found out that that would be completely unacceptable which caused him to reevaluate his costs.  Related to the budget – scarce resources may also be a factor contributing to this conflict.  The other project members continually stress the need for the project to be on time and under budget.  Communications are also contributing to this conflict because people were not actively listening to each other.  Also, other communication channels (e.g., Frank) had an impact on the discussion and caused frustration for Patrick.





2.      On PPT 9-4, The Conflict Process, overt behaviors are depicted as indicators of manifest conflict.  What overt behaviors did you identify during this scenario?



Lucinda repeatedly rolled her eyes and displayed negative affect towards Patrick.  She was also very impatient and insulted Patrick by saying that interrupting him would get him off track. Patrick went on the defensive almost immediately.  Adelson maintained composure but insinuated a lack of confidence in Patrick.



´    1.  Patrick’s strategy should be to:

A.     Emphasize progress

B.     Defend himself

C.     Impress superiors



Patrick should emphasize his progress in terms that the rest of the group will understand and be persuaded by.  Defending himself based on his reputation will only go so far to influencing the group on this particular issue.  Emphasizing his progress will impress his superiors.  In actuality, Patrick defended himself based on his past reputation.



´    2.  What does Sam Adelson insinuate about Patrick [when he calls him a
         hot- shot]?

A.     He’s arrogant

B.     He’s incompetent

C.     He’s inexperienced



Adelson insinuates that Patrick is inexperienced, though apparently competent.  The comment falls short of actually signifying confidence in Patrick.



´        3.  How was this [first] meeting?

A.     A success

B.     A failure

C.     A good start



Patrick thinks it was a failure because he did not feel supported or listened to.



Learning Objective #2:  To analyze and evaluate different conflict management styles.



Various conflict management styles are depicted on PPT 9-7.  For each of the participants in the scenario, describe which style(s) they used and cite examples to support your answer.




Lucinda – Competing – she doesn’t seem to want to listen to Patrick’s position or reasoning and puts roadblocks up to any suggestion that is not her own.

Sam – Compromises – he tries to give Patrick more support which is initially rejected but then reinforces Patrick’s daily email idea with a person to perform that function

Patrick – attempts Competing in that he rejects any suggestions for changing the current situation.  He eventually attempts to compromise with the daily email idea.  In the end he seems to accommodate to the rest of the group in accepting additional help.

Morgan – Avoidance – he really stays out of any substantive discussion on this project.



´        4. How should Patrick respond [when Lucinda says “Frank says you will need an additional 3 weeks’]?



A.     Defame Frank

B.     Provide facts

C.     Admit fault



Patrick indirectly defames Frank by saying he is in a lower-level position for a reason.  A more persuasive technique would have been to provide facts to refute Frank’s claim.  If Frank was correct, than Patrick should have admitted fault.



Learning Objective #3: To identify ways to ensure functional, rather than dysfunctional, conflict occurs.



Review the Structural Approaches to Conflict Management (PPT 9-9).  Several approaches to minimizing dysfunctional conflict are provided.  Identify at least two specific actions taken by members of the group to minimize dysfunctional conflict.


Clarify rules and procedures – Patrick suggests a daily email to keep the group posted on the project.  This will minimize conflict by continually monitoring the situation so there are no surprises.

Increase resources – Sam offered to provide Patrick with a #2 deputy to assist him in managing the project.



Choose two approaches (on PPT 9-9) and provide specific examples of what could have been done in this scenario to reduce dysfunctional conflict.


The superordinate goals that all members share should have been discussed up front.  This guides the discussion and ensures that information provided is maximally influential to other group members.





Improve communication and understanding – group members should have tried to take the others’ perspectives.  Many behaviors indicated that people were not actively listening and had pre-determined ideas on the best course of action and were not willing to change.



´        5. Confidence is low. Patrick should:

A.     Fight back

B.     Accept help

C.     Clarify position



It’s unclear why Patrick is resistant to accept help other than to preserve his ego.  Generally resources are difficult to acquire and he is being offered another staff person to help him.   Ultimately he does accept the help but at first he refuses.



´        6. How should Patrick respond?

A.     Refuse offer

B.     Change approach

C.     Accept offer



Once it is established that he will maintain control over the project, he agrees to the offer.





Exercising Strategy:  Are All Employees Above Average?



Questions



1.         How does this type of performance management system potentially affect training and development and compensation?

            

            Student answers may vary.  The company can use top performers to help in training and development of new employees and those who are not performing well.  It could also offer raises and promotions first to the top performers in the company.





2.         Why might this type of ranking system work in comparison to those systems discussed in the chapter opener?



            Student answers may vary.  This system is more flexible because it does not mandate the exact proportion of employees to be placed in different categories.  This process is better for smaller companies, where there can be a fluctuation in talent from one measurement period to the next.  



3.         What recommendations would you give to First Consulting Group to make this system even more effective?



            Student answers will vary.  Some may suggest offering prizes or vacations to top performers, as a reward and incentive to those that are not performing at the top.  Some may say that the system is too stringent and they should alter it/get rid of it.





Managing People: From the Pages of Business week


Focusing on the Softer Side of Managing


This case scenario examines how one organization, Granite Construction, is striving to improve its relations with employees through an improved feedback system.  Their new ideas have helped the firm’s managers improve their own “people skills” which is proving to be beneficial to the organization.



1.         Having direct reports, peers, and customers evaluate managers’ people skills is a new idea for some companies.  What steps would you recommend a company take if they are going to introduce an evaluation system to help insure that managers accept this type of evaluation system?



Student responses may vary but should identify such items as communication, tying compensation to non-financial objectives, and periodic automated phone surveys,



2.         Advocates of the quality approach argue that “systems factors” (factors not under the control of the person being evaluated) need to be taken into account in performance management systems. What “systems factors” might affect how managers’ direct reports evaluate their people skills?  If you were asked to design an evaluation system that focused on managers’ “people skills” what features would you include to either eliminate the influence of or take account of systems factors?







Systems factors that could affect performance could include weather conditions and other outside influences that employees cannot control. If the students recommended the comparative approach, then they could say that management could observe similar employees in benchmarked organizations.   




Additional Activities
Teaching Suggestions


The topic of performance management is an excellent one for additional activities. Several cases are given that can be used as small group activities or that can be assigned to individual students or discussed in class. There are also some role-plays, a library assignment, a reading assignment, and an outside assignment.



1.         Assign the following article: "Team Appraisals‑Team Approach" by Norman and Zawacki, Personnel Journal, September 1991, pp. 101­104. Ask students to discuss the advantages and disadvantages of the team approach to performance management.



2.         The following short cases can be used as written assignments, in‑class group cases, or role-plays.



Case 1: Don Leslie
(Raymond Noe, Michigan State University)



Don Leslie, age 50, closed his eyes, propped his feet up on his desk, and contemplated his future with Wexler Foods. He could hardly believe that he had just completed his 23rd year marketing various Wexler products. Don entered the military service after college. After four years in the service, Don worked in retail sales and attended the evening M.B.A. program at the University of Minnesota. He joined Wexler after graduation. His first posi­tion was in marketing research at corporate headquarters. He moved up to assistant brand manager and later to brand manager for a successful line of packaged dried fruits. Don's last promotion six years ago was to brand manager for Wexler's popular jams and jellies. The market share for the products had grown slowly but steadily while he was brand manager. Don was comfortable in his position and satisfied with his low six‑figure salary.





This morning, Don learned that one of his colleagues, Carol Marshall, was promoted to marketing vice‑president. Don and several other brand managers were candidates for the position. It was clear that he had been passed over in favor of Carolyn. This was the second time that Don missed a promotion. Perhaps his career had reached a plateau with Wexler Foods. Don tried to picture how this would affect his career aspirations and motivation.



Question


1.         You are Don's manager. What do you propose to do with Don so that he will not become a motivation and performance problem?



Case 2: Martina Jones
(Raymond Noe, Michigan State University)



Martina Jones works for 3M. She is 58 years old, but feels like she might as well be 68. She has had extensive management experience, including over­seas experience. A couple of years ago, the division in which she worked had a major reorganization, and Martina ended up with less responsibility than she used to have. Her performance has not been a problem. Employees in other departments no longer call on her to share her exper­tise. She realizes she's not likely to be promoted again before retirement. In a recent performance feedback session, Martina revealed to you that she is angry and frustrated. After all her years of service and the numerous contributions she's made, she feels like the company has let her down.

Question



1.         You are Martina's manager. What will you do to avoid having Martina become a motivation and performance problem?





Case 3: The Problem Employee
(Raymond Noe, Michigan State University)



One of the employees you manage has been working with the company for three years. The employee's performance has been average to below­ average. You have consistently mentioned to the employee during perfor­mance appraisals that you feel the employee lacks motivation and interest in the job, but it does not seem to have had much of an effect. Recently, the employee's performance has almost ruined three major projects. You had established two goals with the employee: (1) to complete a survey of customer needs within six months, and (2) to increase sales by 30 percent in one year. The employee has made little progress toward these goals during this appraisal period.



Question


1.         You are preparing once again to conduct a performance review with this employee. What will you do during the meeting to try to improve the employee's performance?



3.         Outside assignment: Have students interview a human resources manager about the performance‑appraisal system that he or she uses. Have the students give a short report to the class on their findings.



4.         Library assignment: Have students write a research paper on MBO or BARS. There are numerous sources on both subjects.



5.         An interesting article to assign to students on the subject of discrimination in performance appraisals is "Some Neglected Variables in Research on Discrimination in Appraisals" by Robert L. Dipboye, Academy of Management Review (1985) pp. 116‑127.



6.         The following are role plays dealing with defining the job expectations, giving positive feedback, improving poor performance, and taking disciplinary action. It is often useful to perform these role-plays in front of the class and get students to make comments or set up triads where one person is the observer and provides feedback to the players. Role-plays can also be videotaped and played back for feedback to the players. (The source for these role-plays is Patrick Wright, Cornell University.) Encourage students to change the names to be gender correct for the players.





Defining the Job


Role for James Haas



Craig Teft is a new recruit who just joined your force today. He is coming in to meet with you before you assign him to his partner during his probationary period.



You need to discuss with him exactly what your expectations are for his performance. This includes the fact that he will work with an experienced partner for six months, during which time you expect him to demonstrate his ability as a law enforcement officer. Also during this time, he will be working for substantially lower pay, which will be raised as soon as he is off probation. If he has not demonstrated this ability at the end of the six months, he can either be put on another three‑month probation or he can be terminated immediately.



Role for Craig Teft



You are about to meet with James Haas, the sergeant that you will work under for the next two years. You just graduated second in your class from the law enforcement academy. Because you demon­strated far‑above‑average ability as a police officer, you simply do not feel that you will need to serve out the full six months of your probationary period. You feel that the lower pay you receive during this time is not exactly fair, since you will be paid the same as those who graduated far below you; thus, you would like to serve only three months on probation, rather than the required six.



Giving Positive Feedback


Role for James Haas



You are about to meet with Mary Hill, an officer who has been with your department for just over a year. Over the weekend, she was at the local shopping mall with her husband and apprehended a purse-snatcher in the act while she was off duty. The elderly victim called you this morning to thank the department for having such a fine officer who would risk her safety when she was not on the job. She mentioned that she felt it was great to have female role models for some of the little girls that observed the arrest. This obviously will give the department a lot of positive public image.





Role for Mary Hill



You are about to meet with James Haas, your sergeant. You think that he's going to talk to you about what happened this weekend while you were shopping with your husband at the mall. You just happened to be walking towards an elderly woman about the time a juve­nile snatched her purse. Before you even knew what happened, you had taken the juvenile down and were able to apprehend him. You really didn't think much of it because it all took place so fast



Improving Poor Performance



Role for James Haas



You are about to meet with John McMahon, an officer who has been on the job for two years. In the last few months, there have been some reports that he was spending a large number of on‑duty hours in very nonproductive ways. His car is often seen in front of his home while on duty, and he's on a first‑name basis with the employees of every fast food joint in the town. In many ways, he has done an outstanding job (in terms of arrests, tickets, and meticulous paperwork), but you think that he can do a lot better if he spends a little more time working and a little less time in other somewhat nonwork activities.



Role for John McMahon



You have been working for the New Homestead Police Department for the last two years. You are going to meet with James Haas, your sergeant, in just a few minutes. You assume that he is probably going to commend you on the fine arrest you made last week. Your arrest record is much better than those of most of the other officers you work with, not to mention your excellent record for giving tickets. The arrests you have made and the tickets you have written all stand up in court, and that's more than can be said of a lot of your co­workers. Because you've performed so well, you feel entitled to spend a little extra rest time on the job. You have a good rapport with a lot of the employees at the local fast food restaurants (it's good public relations), and occasionally you are able to slip home to see your family. Although you may do this more than some of the other officers, as long as you are more productive than they are, you don't really see the problem with this.





Taking Disciplinary Action



Role for James Haas



You are preparing to meet with Peter Wemberly, an officer who has worked on your shift for three years. During this time, he has made quite a few very tough arrests, and there are few other officers that his peers would like to have back them up in violent situations. However, yesterday he pulled over a driver who he suspected was driving under the influence of alcohol. The driver refused a Breathalyzer test and became quite belligerent. It was not perfectly clear who actually threw the first punch (although most of the witnesses claim it was the officer). It is quite clear from the witnesses that he used more force than was apparently necessary to subdue the driver. The driver has filed a complaint with your department, claiming that he was physically assaulted by the officer. This is not the first time that a complaint has been filed against this officer, although the other two were dismissed. Your policy is that if there seems to be enough evidence to warrant an investigation, you can (but are not required to) suspend the officer without pay until the investigation is complete. After reviewing the matter, you think that it is in the best interest of the department to suspend him until the investigation has ended.



Role for Peter Wemberly



You are about to meet with James Haas regarding a complaint that was filed against you by some drunk driver. You had pulled the driver over yesterday, and he became quite belligerent. When he began to take a swing at you, you quickly reacted and subdued him quite forcefully. While it may have been more than was enough, you just never really know how much force is necessary with drunk drivers. You have had a few similar complaints against you in the past, but these have always been dismissed. You think you get more of these complaints than other officers because they always seem to send you out on the tough/violent calls. You know that although the policy is that you can be suspended without pay until the investigation is complete, this is not required. You don't think that you will be suspended this time because you feel that you were justified in your actions.



3.                  The article, "Six Strategies for Success in Measuring Performance" by R. Campbell and L. M. Garfinkel, HRMagazine, June 1996, pp. 98‑104, makes the point that no one formula leads to the best performance‑management system. Companies with successful systems look at their business, how work gets done, and the important capabilities required of employees.  





Question


What are the costs and possible disadvantages of the six top‑rated approaches to performance management? Why wouldn't a company use these approaches?

该用户从未签到

 楼主| 发表于 2010-10-3 20:11:45 | 显示全部楼层
Chapter Summary


This chapter begins by discussing the relationship between development, training, and careers. Then, four approaches (education, assessment, job experiences, and interpersonal relationships) used to develop employees, managers, and executives are presented. The third section of the chapter provides an overview of the steps of the development planning process.  The chapter concludes with a discussion of special issues, including succession planning, dealing with dysfunctional managers, and melting the glass ceiling.



Learning Objectives


After studying this chapter, the student should be able to:



1.                  Discuss the current trend in using formal education for development.

2.                  Relate how assessment of personality type, work behaviors, and job performance can be used for employee development.

3.                  Develop successful mentoring programs.

4.                  Explain how job experience can be used for skill development.

5.                  Tell how to train managers to coach employees.

6.                  Discuss the steps in the development planning process.

7.                  Explain the employees’ and company’s responsibilities in the development planning process.

8.                  Discuss what companies are doing for management development issues including succession planning, melting the glass ceiling, and helping dysfunctional managers.

9.                  Explain how employee development contributes to strategies related to employee retention, developing intellectual capital, and business growth.



Extended Chapter Outline



Note: Key terms appear in boldface and are listed in the "Chapter Vocabulary" section.



Opening Vignette: Developing Employees Increases Intellectual Capital at Booz Allen Hamilton


Booz Allen Hamilton is a strategy and technology consulting company.  It uses a program called the Development Framework to help managers and employees choose the right combination of development activities and to strengthen competencies and manage their careers.  The development framework consists of four sections: Development roles, Performance expectations, Development needs, and Development road map.  The framework is used to help employees better understand how to develop themselves.





I.          Introduction



II.         The Relationship between Development, Training, and Careers (See Table 9.1 in text)

            

A.        Development refers to formal education, job experiences, relationships and assessment of personality and abilities that help employees prepare for the future. It involves learning what is not necessarily related to the employee's current job.



B.         While development involves learning what is not necessarily related to one’s current job, training is focused on helping employees’ performance in their current job. Development helps prepare employees for other jobs within the organization.



Example: Southwest Industries, an aerospace engineering company, uses teams to identify specific customer needs, and the team is responsible for determining how those needs can be best met.



C.        Development and Careers- Careers have been described as the sequence of positions held within an occupation. Each employee’s career consists of different jobs, positions, and experiences.



1.                  A new concept of the career is often called a protean career. A protean career is a career that is frequently changing based on both changes in the person’s interests, abilities, values, and changes in the work environment. Employees are taking more responsibility for managing their careers compared to the traditional career.



a.                   Evaluating the psychological contract between employees and the company has influenced the development of the protean career. A psychological contract is the expectations that employees and employers have about each other.



b.                  The goal of a new career for an employee is psychological success or the feeling of pride and accomplishment that comes from achieving life goals that are not limited to achievements at work.





2.                   The types of knowledge that an employee needs to be successful have changed.  In the traditional career, “knowing how” was critical.  Now employees also need to “know why” and “know whom.”



3.                  The emphasis on continuous learning and learning beyond knowing how as well as changes in the psychological contract are altering the direction and frequency of movement within careers.



a.                   Traditional career patterns consisted of a series of steps arranged in a linear hierarchy, with higher steps in the hierarchy related to increased authority, responsibility, and compensation.



b.                  Career patterns involving movement across specializations or disciplines (a spiral career pattern) will become more prevalent.



c.                   The most appropriate view of a career is that it is “boundaryless.”  It may include movement across several employers or even different occupations.



4.                  To retain and motivate employees companies need to provide a system to identify and meet employees’ development needs.  This system is often known as career management or development planning system.



III.       Approaches to Employee Development


A.        Formal Education



1.                  Formal education programs include off‑site and on‑site programs designed specifically for the company's employees, short courses offered by consultants or universities, executive MBA programs, and university programs in which participants actually live at the university while taking classes.



Example: The University of Tennessee at Chattanooga has an executive MBA Program that includes a week of study abroad in the final year of the program.





2.                  Table 9.3 lists the five largest institutions for executive educa­tion. Many companies and universities are increasingly using distance learning to reach executive audiences.



3.                  Another trend in executive education is for companies and the education provider to create short, custom courses, with the content designed specifically to meet the needs of the audience.



4.                  The final important trend in executive education is to supplement formal courses from consultants or university faculty with other types of development activities.  For example, Avon Products’ “Passport Program.”



5.                  Most companies consider the primary purpose of education programs to be providing the employee with job‑specific skills.



Competing through Technology


E-learning Helps Build Management Talent



To be successful in the new economy, companies need to identify employees with managerial talent and help managers develop skills needed to be more effective.  To attract and retain talented employees who are in short-supply, companies need to offer training and development opportunities on the web that meet the needs of a geographically dispersed workforce dealing with many work demands.



This case scenario looks at how IBM is attempting to encourage and develop its managers.   



B.         Assessment involves collecting information and providing feedback to employees about their behavior, communication style, or skills.



1.                  Myers‑Briggs Type Indicator (MBTI)



a.                   MBTI, the most popular psychological test for employee development, consists of more than 100 questions about how the person feels or prefers to behave in different situations.





b.                  The MBTI identifies individuals' preferences for energy (introversion versus extroversion), information gathering (sensing versus intuition), decision-making (thinking versus feeling), and lifestyle (judging versus perceiving).



c.                   Sixteen unique personality types result from the four MBTI preferences (Text Table 9.4).



d.                  MBTI is used for understanding such things as communica­tion, motivation, teamwork, work styles, and leadership.



e.                   MBTI scores appear to be related to one's occupation but are not necessarily stable over time.



2.                   Assessment Center



a.                   The assessment center is a process in which multiple raters evaluate employees' performance on a number of exer­cises



b.                  Role-plays are exercises in which a participant takes the part or role of a manager or other employee.



c.         In-basket is a simulation of the administrative tasks of a manager’s job.  



d.         A leaderless group discussion is a process in which a team of 5-7 employees are assigned a problem and must work together to solve it within a certain time period.



Example: Allied Domecq Spirits and Wine, North America (formerly Hiriam \Yialker), arranges a yearly regional assess­ment center, called their Management Development Center, at Eastern Michigan University's Corporate Education Center in Ypsilanti, Michigan. The center's purpose is to assist upper‑level managers achieve their full professional and career potential through specific actions derived from feedback and reports generated as a result of testing and evaluation over a number of days at an off‑site location.





e.         From 6 to 12 employees usually participate at one time, and the assessment center is usually held at an off‑site location.



f.          The evaluators are normally managers from the organization who have been trained to identify behaviors related to the skills assessed.



g.         Assessment center exercises are designed to measure employees’ administrative and interpersonal skills.



h.         Assessment centers are useful for development purposes because employees who participate in the process receive feedback regarding their attitudes, skill strengths, and weaknesses.



3.                   Benchmarks



a.                   Benchmarks are an instrument designed to measure the factors that are important to being a successful manager.



b.                  There are 16 skills and perspectives believed to be impor­tant for becoming a successful manager that are measured by benchmarks (Text Table 9.6).



c.                   The managers’ supervisors, peers, and the managers themselves complete the instrument, and a summary report includes self‑ratings and ratings by others, along with comparative information with other managers. A development guide is also available.



4.                   Performance Appraisals and 360-degree Feedback Systems



a.                   To be useful for development, performance appraisals must provide specific information to employees about their performance problems and ways they can improve their performance.



b.                  Upward feedback involves subordinates’ evaluation of managers’ behaviors or skills.





c.                   The 360‑degree feedback system is a case where managers' behaviors and skills are evaluated by subordinates, peers, customers, their bosses, and themselves.



1).        The benefits of 360‑degree feedback include collecting multiple perspectives of managers' performance, allowing the employees to compare their own personal evaluation with the view of others, and formalizing communications between employees and internal and external customers.



2).        Some of the potential limitations of 360‑degree feed­back include the time demands placed on the raters to complete the evaluation, managers seeking to identify and punish raters who provide negative information, the need to have a facilitator to help interpret results, and companies' failure to provide ways that managers can act on the feedback they receive.



A.                 Job Experiences—Most employee development occurs through job experiences,

and development is most likely to occur when employees must stretch their skills (Text Table 9.9).



Example: At Andersen Consulting, during the first five years as a consultant, the individual receives nearly 800 hours of formal training supplemented with carefully chosen on‑the‑job experiences. Early job assignments often cross business and technical areas to allow employees to discover interests and abilities.



1.                  Enlarging the Current Job – Job enlargement refers to adding challenges

and new responsibilities to employees’ current jobs.



Example: Joanne has recently had her management job enlarged by being assigned an additional employee to supervise.





2.                   Job Rotation



a.                   Job rotation involves providing employees with a series of job assignments in various functional areas of the company or movement among jobs in a single functional area or department.



b.                  Job rotation helps employees gain an overall appreciation of the company's goals, increases their understanding of different company functions, develops a network of contacts, and improves their problem solving and decision­ making skills.



3.         Transfers, Promotions, and Downward Moves



a.                   A transfer is usually a lateral move in which an employee is given a different job assignment in a different area of the company.



b.                  Promotions are advancements into positions with greater challenges, more responsibility, and more authority than in the previous job; they usually include a pay increase.



c.         Since transfers requiring relocations can be anxiety producing, many companies have difficulty getting employees to accept them.



(1)               Employee characteristics associated with a willingness to accept transfers include high career ambitions, a belief that one's future with the company is promising, and a belief that accepting a transfer is necessary for success in the firm.



(2)               Among married employees, the spouse's willingness to move is the most important influence on whether employees will accept a transfer.



c.                   A downward move occurs when an employee is given a reduced level of responsibility and authority. Temporary cross‑functional moves to lower‑level jobs, which give employees experience working in different functional areas, are most frequently used for employee development.





d.                  To ensure that employees accept transfers, promotions, and downward moves as development opportunities, compa­nies can provide such things as an orientation program for the new location and job or employee involvement in the transfer decision.



4.         Temporary Assignments with Other Organizations



a.                   Many companies use temporary assignments in nonprofit organizations as a method to help employees apply their current skills to new problems and to develop new skills.



b.                  First Chicago National Bank and Kodak participated in an employee-exchange program so that the two companies could better understand each other’s business and discover ways to improve the services provided.



c.                   Externship refers to a company allowing employees to take a full-time operational role at another company.


d.                  Temporary assignments can include a sabattical (a leave of absence from the company to renew or develop skills).



D.        Interpersonal Relationships


1.                  Mentoring



a.                   A mentor is an experienced, productive senior employee who helps develop a less‑experienced employee (protege).



Example: NCR, headquartered in Dayton, Ohio, has spon­sored mentoring for five years and uses it as an early iden­tification program for high‑potential employees who are selected by their mentors to be advised. Mentoring at NCR is viewed as an educational tool to train and track "green" talent (new college graduates).





b.                  Most mentoring relationships develop informally; however, companies can have formal mentoring programs.



(1)               One of the limitations of formal mentoring programs is that the mentors may not be able to provide counseling and coaching in a relationship that has been artificially created.



(2)               One of the major advantages of formalized mentoring programs is that they ensure access to mentors for all employees, regardless of gender or race.



(3)               Characteristics of a successful formal mentoring program include such things as voluntary participation and the length of the program being specified (Text Table 9.11).



2.         Benefits of Mentoring Relationships



a.                   Mentors provide career support, which is coaching, protection, sponsorship, and providing challenging assignments, exposure, and visibility.



b.                  Mentors offer psychological support, which is serving as a friend and role model, providing positive regard and acceptance, and creating an outlet for a protégé to talk about anxieties and fears.



c.                   Proteges normally receive higher rates of promotion, higher salaries, and greater organizational influence.



d.                  Mentoring relationships provide opportunities for mentors to develop their interpersonal skills and increase their feel­ings of self‑esteem and worth to the organization.





3.         Purposes of Mentoring Programs



a.                   Mentor programs are used to socialize new employees, to increase the likelihood of skill transfer from training to the work setting, and to provide opportunities for women and minorities to gain the exposure and skills needed to move into managerial positions.



b.                  As part of their approach to managing a diverse work force, many companies are using mentoring programs in which women and minorities gain the skills and visibility needed to be promoted. Pacific Bell's mentoring program received an award from the Department of Labor for its commit­ment to equal employment opportunity.



c.                   In group mentoring programs, a successful senior employee is paired with a group of four to six less experienced proteges.  One potential advantage of the mentoring group is that proteges can learn from each other as well as from a more experienced senior employee.



4.         Coaching



a.                   A coach is a peer or manager who works with an employee to motivate him, help him develop skills, and provide reinforcement and feedback.



b.                  There are three roles that a coach can play.  Part of coaching may be one-on-one with an employee.  Another role is to help employees learn for themselves.  Third, coaching may involve providing resources such as mentors, courses, or job experiences that the employee may not be able to gain access to without the coach’s help.





c.                   To develop coaching skills, training programs need to focus on four issues related to mangers’ reluctance to provide coaching.  First, managers may be reluctant to discuss performance issues even with a competent employee because they want to avoid confrontation.  Second, managers may be better able to identify performance problems than to help employees solve them.  Third, managers may also feel that the employee interprets coaching as criticism.  Fourth, as companies downsize and operate with fewer employees, managers may feel that there is not enough time for coaching.





Competing through Sustainability


Mentoring Programs Have Potential to Pay Individual, Corporate, and Societal Dividends



A company’s sustainability is determined by the extent to which it satisfies the needs of shareholders, customers, employees, community, and society.  KLA-Tencor, a supplier of process control solutions for the semiconductor industry, uses mentoring to improve senior managers’ skills.  Fannie Mae uses a mentoring program that is designed to encourage the advancement of high-potential employees, especially women and minorities.  Cardinal Health’s mentoring program is designed to expose mentors and protégés to the company’s different business units.





IV.       Career Management and Development Planning Systems


A.                 Steps and Responsibilities in the Career Management Systems (Figure 9.2)



1.         Self‑Assessment



a.                   Self‑assessment helps employees determine their interests, values, aptitudes, and behavioral tendencies.



b.                  Self‑assessment often involves the use of psychological tests such as the Strong‑Campbell Interest Inventory and the Self-Directed Search.





2.         Reality Check



a.                   Employees receive information about how the company evaluates their skills and knowledge and where they fit into the company's plans.



b.                  Usually, employees receive this information from their managers as part of the performance appraisal process.



3.         Goal Setting



a.                   Employees determine their short‑term and long‑term career goals during this phase of the career planning process.



b.                  These goals are often discussed with the manager and written into a development plan (Text Figure 9.3).



4.         Action Planning



a.                   During this phase, employees determine how they will achieve their goals.



b.                  Action plans may involve enrolling in training courses and seminars, conducting informational interviews, or applying for job openings within the company.



Competing Through Globalization



Career and Development Planning Needed for Successful Reentry



Development planning is an important issue in successful repatriation of expatriates.  Employees should be encouraged to self-manage the repatriation process.  Royal Dutch Shell gets involved with expatriates and their careers to avoid the problem of expatriates feeling undervalued and ultimately leaving the company.





V.        Special Issues in Employee Development


A.        Melting the Glass Ceiling



(1)               The glass ceiling is a barrier to advancement to the higher levels of the organization. This barrier may be due to stereotypes or company systems that adversely affect the development of women or minorities.



(2)               Many companies are using mentoring programs to ensure that women and minorities gain the skills and visibility needed to move into managerial positions.



Example: Mattel has been one of the companies shat­tering the glass ceiling. Women have ascended into senior management positions with significant responsi­bilities, including Jill Barad, CEO of Mattel Toys.





B.         Succession Planning



1.         Succession planning primarily involves the identification and tracking of "high‑potential employees." High‑poten­tial employees are those that the company believes are capable of being successful in higher-level positions. These employees typically participate in fast-track development programs that involve education, executive mentoring and coaching, and rotation through job assignments.



2.         Research suggests that the development of high‑potential employees involve three stages: In Stage I, high‑potential employees are selected. In Stage II, high‑potential employees receive development experiences. In Stage III, high‑potential employees usually have to be seen by top management as fitting into the company's culture and having the personality characteristics needed to successfully represent the company.



C.        Helping Managers with Dysfunctional Behaviors


1.         Some development programs are designed to develop managers with dysfunctional behaviors such as insensitivity to others, inability to be a team player, arrogance, and poor conflict‑management skills.





2.         Typically, a combination of assessment, training, and coun­seling is used to help managers change the dysfunctional behavior.



3.         One program designed specifically to develop managers with dysfunctional behavior is the Individual Coaching for Effectiveness (ICE) program. The ICE program includes diagnosis, coaching, and support activities. Clinical, coun­seling, or industrial/organizational psychologists are involved in all phases of the ICE program.


A Look Back


The chapter opener described Booz Allen Hamilton’s development program designed to help employees develop and take charge of their careers.  



Questions


1.         How might job experiences be useful for helping employees develop?



Student responses will vary, but may include that by mismatching an employee’s skills and past experiences and the skills needed for his or her job will require the employee to expand his or her skills to fit the job skills.



2.         To be effective, what should a development plan include that an employee at Booz Allen Hamilton complete as part of the Development Framework?



Students’ answers will vary.  They may include that the employees need to perform self-assessments to see what they need to improve upon and what opportunities that already have.  Then, employees need to get information about how the company evaluates their skills and knowledge and where they stand in the company’s plans.  Next, employees need to decide where they want to advance to, regarding job positions, skills, work setting, and/or skill acquisition.  Finally, the employees need to determine paths to take to achieve their goals.





Chapter Vocabulary


These terms are defined in the "Extended Chapter Outline" section.



Development
Protean Career

Psychological Contract

Psychological Success

Career Management System

Formal Education Programs

Assessment

Myers-Briggs Type Indicator (MBTI)

Assessment Center

Leaderless Group Discussion

Interview

Role Play

In-Basket

Benchmarks

Performance Appraisal

Upward Feedback

360-Degree Feedback Systems

Job Experience

Job Enlargement

Job Rotation

Transfer

Promotions

Downward Move

Externship

Sabbatical

Mentor

Career Support

Psychological Support

Group Mentoring Programs

Coach

Glass Ceiling

Succession Planning

High-potential employees





Discussion Questions


1.                  How could assessment be used to create a productive work team?



Assessment could be used to create a productive work team by selecting individuals that would work well together.  Also, assessment would help individuals understand themselves and their work style so that they can better understand their reactions to others.  MBTI is an especially useful tool to help teams understand the different personality types and how they work together.  Assessment can also help match teams with assignments that allow them to capitalize on their preferences and strengths.



2.                  List and explain the characteristics of effective 360-degree feedback systems.



An effective 360-degree feedback system is where employees’ behaviors and skills are evaluated not only by subordinates, but also by peers, customers, their bosses, and themselves.  The raters are given a questionnaire to complete asking them to rate the person on a number of different dimensions.  Raters are often asked to rate the degree to which each particular item is considered a strength or needs development.  Some benefits of the system include collecting multiple perspectives of mangers’ performance, allowing the employee to compare his own personal evaluation with the views of others, and formalizing communications about behaviors and skills rated between employees and their internal and external customers.  By performing this system, studies have shown that performance improvement and behavior change occur as a result.   Reliable or consistent ratings are provided, raters’ confidentiality is maintained, the behaviors or skills assessed are job-related (valid), the system is easy to use, and mangers receive and act on the feedback.  





3.                  Why do companies develop formal mentoring programs?  What are the potential benefits for the mentor?  For the protégé?





Companies develop mentoring programs because it allows employees to increase their knowledge and develop skills by interacting with a more experienced employee.  A mentor is an experienced, productive senior employee who helps develop a less experienced employee.  One major reason why companies form formal mentoring programs is because they ensure access to mentors for all employees, regardless of race or gender.  Also, participants in the mentoring relationship know exactly what is expected of them. Mentor programs are used to socialize new employees, increase the likelihood of skill transfer from training to the work setting, and to provide opportunities for women and minorities to gain the exposure and skills needed to evolve into managerial positions.  



Mentoring relationships provide opportunities for mentors to develop their interpersonal skills and increase their feel­ings of self‑esteem and worth to the organization.  Mentors provide career support and psychological support to their protégés.



Protégés normally receive higher rates of promotion, higher salaries, and greater organizational influence.



4.                  Your boss is interested in hiring a consultant to help identify potential managers among current employees of a fast food restaurant. The manager’s job is to help wait on customers and prepare food during busy times, oversee all aspects of restaurant operations (including scheduling, maintenance, on-the-job training, and food purchase), and help motivate employees to provide high-quality service. The manager is also responsible for resolving disputes that might occur between employees. The position involves working under stress and coordinating several activities at a time. She asks you to outline the type of job assessment program you believe would do the best job of identifying employees who will be successful managers. What will you tell her?





The best type of assessment program that should be used is the MBTI.  One reason for this is because to become a manager in a type of stressful and active atmosphere as a restaurant or fast food restaurant you need to have a certain type of personality.  The MBTI will let a consultant know what each employee’s personality is and which one will work best under this high-pressure job.  A person taking this type of position needs to be flexible, able to work with others, agreeable and stern, approachable, etc. in order for the employees’ and customers to feel comfortable to approach him/her when things go wrong.  Another assessment program could be benchmarks or assessment center.



5.                  Many employees are unwilling to relocate because they like their current community, and spouses and children prefer not to move. Yet employees need to develop new skills, strengthen skill weaknesses, and be exposed to new aspects of the business to prepare for management positions. How could an employee’s current job be changed to develop management skills?



Some ways that an employee’s current job could be changed to develop his or her leadership skills include job rotation assignments, enlarging the current job by adding challenges or new responsibilities, and transfers or promotions.  Also, the current employees could be sent to workshops updating them on the various management skills where they have to interact and participate to gain a feeling for the different management skills when in practice.  



6.                   What is coaching? Is there one type of coaching? Explain.



Coaching is when a peer or manager works with an employee to motivate him, help him develop skills, and provide reinforcement and feedback.  Coaching can take on three different roles.  One role is the one-on-one with an employee, which includes giving them feedback.  The second role is to help employees learn for themselves which involves helping them find experts who can assist them with their concerns and teaching them how to obtain feedback from others.  The third role includes providing resources such as mentors, courses, or job experiences that the employee may not be able to gain access to without the coach’s help.  





7.                  Why are many managers reluctant to coach their employees?



Managers are reluctant to coach their employees because they would rather avoid confrontation so they are reluctant to discuss performance issues with even a competent employee.  This occurs usually when the manager is less of an expert than the employee; for instance, when a new manager is trying to discuss performance with a long term employee.  Second, managers may be better able to identify performance problems than to help employees solve them.  Third, managers may also feel that the employee interprets coaching as criticism.  Fourth, as companies downsize and operate with fewer employees, managers may feel that there is not enough time for coaching.



8.                  Why should companies be interested in helping employees plan their careers?  What benefits can companies gain? What are the risks?



This question could be used as a brainstorming exercise in which students try to think of as many benefits as they can.  Some benefits would include improved employee morale if employees feel that the company cares about their careers, better career planning for the company, a better fit with the company and the employees, and so on.  The risks are that employees may decide to change careers and leave the company after the company has spent time and money training them.



9.                   What are the manager’s roles in a career management system? Which role do you think is most difficult for the typical manager? Which is the easiest role?  List the reasons why managers might resist involvement in career management.



The manager’s roles in a career management system include self-assessment, reality check, goal setting, and action planning.  In self-assessment managers’ provide assessment information to identify strengths, weaknesses, interests, and values.  In the reality check role, managers’ communicate performance evaluation to show where employee fits into long range plans of the company.  In the goal setting role, managers’ ensure that the goal is specific, challenging, and attainable as well as committing to help the employees reach the goal.  In the action-planning role, managers’ identify resources employees need to reach goal including courses, work experience, and relationships.  





The most difficult role is the reality check role because here managers’ have to communicate with their employees about their performance and evaluations.  This may be very hard for some managers because not many are comfortable with commenting on employees’ progress because they don’t want to create conflict, may not be as knowledgeable as the employee being evaluated, and the manager may feel that the employee will take it as criticism and become defensive.  The easiest role for managers is the goal setting role because almost every manager has an idea on what the goals for the company are to be and in this role they have to make those goals clear and attainable to the employee.



Managers may resist involvement in the management career program because they may not feel comfortable with such a structured program.  Many managers are not comfortable in being involved in all the steps.



10.       What are the characteristics of the most effective company development strategies?  Which characteristics do you believe is most important?  Why?



            Student answers will vary.  Successful companies use a wide array of development strategies.  Certainly, many use mentoring and coaching as a method for developing new employees.  In these programs, the most important characteristic is fairness and flexibility.  Companies should not discriminate in terms of who is given development opportunities.  Diversity should be encouraged



11.       Nationwide Financial, a 5,000 employee life insurance company based in Columbus Ohio, found that their management development program contained four types of managers.  One type, unknown leaders, have the right skills but their talents are unknown to top managers in the company.  Another group, arrogant leaders believe they have all the skills they need.  What types of development program would you recommend for these managers?



            Student answers will vary.  Mentoring programs would work better for unknown leaders, while transfers may be suggested for arrogant leaders.







Manager’s Hot Seat Exercise: Personal Disclosure: Confession Coincidence?



I.  Introduction



This scenario demonstrates how disclosure of personal information and how friendships between managers and employees can create difficult situations.  Management and Organizational Behavior students will enjoy analyzing this case and discussing the pros and cons of various actions.  This case could also be used in a Human Resources course when discussing the Americans with Disabilities act, as the situation deals with a recovering alcoholic and potential discrimination.  



II.  Learning Objectives



To assess students’ understanding of the self-disclosure process in the workplace.
To analyze and evaluate the “friend/manager” role in the workplace.
To analyze organizational downsizing strategies.  


III.  Scenario Description:



Overview:  Kathleen Doerder has set up a meeting with an employee, Janeen Winthrop, who has a recent record of poor performance.  Although Kathleen has met with Janeen on two prior occasions to discuss the issue, the poor performance and frequent absences and tardiness, have continued.  Janeen’s behavior has disrupted workflow, negatively affecting a number of colleagues within the department.  



Profile:

·        Kathleen Doerder is the Manager of a 65-employee marketing department at Gen-Y.net, an internet travel agency selling customized surprise adventure trips around the world.

·        Janeen Winthrop works with the creative marketing department to produce radio and print advertisements.  She works within a team of 15, generating concepts and assisting in the implementation and layout of designs.



References:  The references included in the DVD are:

·        Components of Self-Disclosure (PPT 11-3)

·        Benefits of Self-Disclosure (PPT 11-4)

·        Elements of Trust (PPT 11-7)

·        Tips for Developing Trust (PPT 11-8)





Back History:  Janeen has never been known for her punctuality, which has never been a big deal – it’s actually become one of the running jokes in the department.  But now it is becoming a big deal – very big.  Janeen has been showing up later and later, more and more frequently over the course of six weeks.  It is becoming problematic – Kathleen is quite torn:  she feels she should be lenient, knowing Janeen is truly devoted to maintaining an enthusiastic and content staff and is always persistent in her efforts to make things work out.   



Janeen is a recovered alcoholic and has been sober for eight years.  Over the past six weeks, she’s had a few relapses.  Janeen has been very close-mouthed about her alcoholism and has shared it with no one – on social occasions, she claims to never have had any interest in alcohol and simply prefers to not partake.  As of two weeks ago she’s been going to AA twice a day.  She’s not drinking and is over the hump.



Scene Set-up: Kathleen meets with Janeen to discuss her tardiness and performance problems.



Scene Location: Kathleen’s Office



The Meeting - Summary:  Kathleen explains to Janeen that she has noted increased tardiness and poor performance.  Kathleen tries to get Janeen to divulge what is going on in her life personally that could explain this sudden downturn in performance.  Janeen breaks down in tears and reveals that she is a recovering alcoholic who had a relapse in the past six weeks but she is returning to AA and is getting back on track.  Kathleen is sympathetic to this issue and suggests that she take the rest of the week off and then return on Monday.  Kathleen indicates that if Janeen continues to be late, she’ll have to report the issue to HR, otherwise, she will not share this information with anyone.



Two months later – The company has suffered financially and needs to lay off employees.  Kathleen explains that Janeen is being laid off because of her seniority, not her performance.  Janeen is outraged and accuses Kathleen of using the personal information she divulged against her.  She threatens to sue and storms out.





Afterthoughts – Summary:  Kathleen says that it was hard not to get emotionally involved with Janeen when she broke down and cried but she had to stay even-keeled.  Kathleen says that being friends with her employees is a strategy she uses to improve relations among co-workers.  Being Janeen’s friend made it hard to tell her she was being laid off.  If she didn’t lay her off, others would think it was because of nepotism.  When she did lay her off Janeen thought it was because of the personal information Janeen shared.



Dossier:  The specific artifacts included in the DVD are:

1.      J. Winthrop Performance Notes:  4/8/02

2.      J. Winthrop Performance Notes:  6/7/02

3.      Company-wide Notice on Layoffs



IV.  Discussion Questions:



The References and related Discussion Questions may be found in PowerPoint slides 11-1 to 11-10 on the instructor’s side of the text’s Website.



Learning Objective #1:  To assess students’ understanding of the self-disclosure process in the workplace.



1.        Review the Components of Self-Disclosure (PPT 11-3).  Based on Janeen’s behavior in the scenario, what basic steps and tips did she exemplify or violate?



Janeen did share her feelings and focused mainly on the present by indicating what she was doing now to solve the problem.  It’s unclear what level of disclosure she had provided to Kathleen in the past so it’s hard to tell if it was gradual or not.  From the scenario, it appears that she did not maintain a reciprocal level with her manager, but, again, we don’t know what prior personal conversations may have taken place.



Student’s answers will vary as to the appropriateness of the disclosure at this point in time.  Discuss the level of disclosure Janeen provided and what other approach she may have taken.





2.        What benefits of self-disclosure (see PPT 11-4) did Janeen realize?  What were the disadvantages of her self-disclosure?



Janeen probably enjoyed many of the benefits listed on the slide.  Specifically, she was given time off to continue to make progress on her recovery.  Janeen perceives that her disclosure was a major disadvantage because it caused her to be fired.  Again, it may be useful to discuss more moderate levels of disclosure that Janeen could have taken to prevent this situation.



´        1.  What is Kathleen’s strategy [in the initial meeting]?

A.     Identify problem

B.     Threaten Janeen

C.     Be compassionate



Janeen wants to understand Janeen’s problem because her work performance is suddenly different.



´        2. Janeen’s disclosure [about her relapse] is:

A.     Inappropriate

B.     Appropriate

C.     Irrelevant



Because she was pressed by Kathleen, Janeen’s disclosure was appropriate.  It appeared that that was the type of information Kathleen was looking for.  However, it may not have been the best course of action for Janeen.



Learning Objective #2:  To analyze and evaluate the “friend/manager” role in the workplace.  



What tips for building trust did Kathleen employ in this scenario?  Provide examples from the scenario to support your answer.  (see PPT 11-7 and PPT 11-8)


She kept the lines of communication open by soliciting information with Janeen and by having repeated meetings with her regarding her performance.  She says she kept the information about Janeen’s alcoholism confidential.  





How did the friendship with Janeen affect Kathleen when the company decided to downsize?


Kathleen was stuck in a very difficult situation because she had used her friendship with Janeen to get her to reveal information that may have affected her decision on who to lay-off.  Even if the information did not affect her decision, the appearance that it did is hard to defend.  On the one hand, Kathleen wants to be open and honest with her employees, but she also wants to maintain complete professionalism and make business-related decisions.  This scenario depicts the fact that this is a hard line to walk.



´        3.  Kathleen’s response [that she will have to write Janeen up and tell human resources if she doesn’t improve] is:



A.     Too strict

B.     Too gentle

C.     Irresponsible



Student’s answers will vary and this should generate a good discussion about how the friendship/management role differentiation is challenging.





Learning Objective #3:  To analyze organizational downsizing strategies.  



After reviewing the “Company-wide Notice on Layoffs”, do you believe Kathleen made the right decision in laying off Janeen?  Why or why not?


The memo says that the first round of lay-offs will be based on performance, not seniority.  Then, seniority will be the determining factor if more layoffs are required.   During her conversation with Janeen, Kathleen refers to her seniority as the reason she is being laid off.  Janeen indicates that she is the first one to be laid off and her performance has been excellent.  Kathleen agrees and “J. Winthrop Performance Notes:  6/7/02” describe Janeen’s excellent recent performance.  Therefore, it doesn’t make sense that Janeen should be the one laid off first.  Perhaps, her performance when she suffered her relapse was taken into consideration?  Discuss the students’ perspectives on this.





´        4. Kathleen’s delivery [regarding Janeen’s layoff] is:

A.     Compassionate

B.     Cold

C.     Professional



Student’s answers will vary.  It seems that she may be a little cold and also doesn’t really provide an adequate explanation or effectively refute Janeen’s accusations.



´        5. Janeen’s argument [that she is being discriminated against] is:

A.     Valid

B.     Over-emotional

C.     Off target



She seems to jump to conclusions without asking for more information which indicates that she is a little over-emotional.  However, her suspicions could be valid if Kathleen’s choice was biased and based on seniority rather than overall performance to determine her layoff.



´        6. Is there a discrimination case?

A.     Absolutely

B.     Definitely not

C.     Possibly



It’s definitely possible because individual’s who are recovering from a drug or alcohol addiction and actively seeking help are protected under the Americans with Disabilities Act.  Therefore, for the reasons mentioned in the answer to #1 above, Janeen may have a case.







Exercising Strategy: Employees in Motion at PPG Industries



Questions



1.         PPG is relying on moving employees to new positions as a development strategy.  What things should PPG do to ensure this is a successful strategy?  Are other development activities necessary?  Why?



            Student answers may vary.  PPG may be moving employees around in order to cross-train them in various functions.  When leaders need to be replaced, there will be a wider pool to select from because of these lateral moves.  Other development activities may be necessary to ensure that moved employees are not discouraged by the changes.  Mentoring would be a great way for more experienced employees to transfer knowledge to younger employees before they leave.  



2.         What data on outcomes should be collected to monitor the effectiveness of PPG Industries development program?



            Student answers may vary.  The company could perform cost-benefit analyses to see if this strategy is working.  Also, is should study sales to see if losing experienced salespersons to different positions is hurting the company.  In addition, the company could interview employees or submit questionnaires.





Managing People: From the Pages of Business Week



How to Groom the Next Boss


Questions


1.                  What development activities did Kenneth Freeman use to strengthen the skills of Surya Mohapatra?  List the activities and the skills they were designed to improve.  What other development activities could he have used?  Identify the development activities and the skills they would be targeted to improve.



Student answers may vary.  Freeman used case assignments to see candidates’ leadership skills in action.  He used trial by fire to improve Mohapatra’s speaking skills.  Weekly phone conversations were used to help Mohapatra become a faster thinker.  Freeman asked Mohapatra to give board presentations to make him more participatory on the Board of Directors.  





Other recommendations may include mentoring or coaching with other supervisors to give Mohapatra an understanding of every facet of the organization.  



2.         Could a coach help Mohapatra develop the skills needed to be an effective CEO?  Explain.



            Student answers may vary.  A coach would probably be effective because having someone by his side most of the day, talking to him constantly will improve his communication skills and ability to think quickly.  It would also show Mohapatra how others apply their leadership skills in the organization.  If the coach is on the Board, he or she could show Mohapatra how to be a more assertive member.



3.         What recommendations do you have fore identifying and preparing managers for CEO positions?  Indicate the succession planning process as well as the development activities you would recommend.



            Student answers will vary.





Additional Activities



Teaching Suggestions



Several interesting topics in this chapter lend themselves to additional activ­ities, such as diversity training, mentoring, and so on. A couple of in‑class activities are suggested as well as a possible library assignment.



1.        The following statements can be given to student groups (three to five students per group), and they can be asked to agree or disagree and prepare the rationale for their decision.



Diversity
(Raymond Noe, Michigan State University)





"For diversity training to work, it requires employees to explore values and stereotypes they hold about different ethnic, racial, and gender groups. This may actually result in even more alienation between groups. Why? One of the potential negative aspects of this approach is that it highlights the difficulty that employees will have in trying to determine the 'right' way to communicate and work with others from ethnic, racial, or religious backgrounds with whom they are not accustomed to dealing. Also, discussing stereotypes may expose the company to potential lawsuits due to the fact that the training is very personal in nature and may expose discriminatory practices and libelous and slanderous information."



"Our company attempts to manage a diverse work force, not through training, but through affirmative action and strict adherence to the Civil Rights Act of 1991. We ensure that employees are treated fairly in terms of compensation, selection, development opportunities, and promotions."



"Because diversity training is such a sensitive issue, we will use it only in response to incidents that occur that warrant attention."



2.                  Mentoring is a subject that can be used for class discussion. An interesting article on mentoring is "Mentoring Takes a New Twist" by Beth Rogers, HR Magazine, August 1992, pp. 48‑51. What are the advantages and problems with formal mentoring programs? What are the advantages and problems with informal mentoring programs? What are the advantages and problems with the person serving as a mentoring? As the person being mentored?



3.                  Part of dealing with diversity for most organizations includes dealing with an increasing number of women managers. Students could take a test called "Women‑as‑Managers Scale" to determine their stereotypes and possible misconceptions about women managers. This self‑test can be found in Terborg, Peters, llgen, and Smith, "Organizational and Personal Correlates of Attitude toward Women as Managers." Academy of Management Journal, March 1977, p. 93.



4.                  Library Assignment: Have students research examples of successful corporate mentoring programs, diversity training programs, and assessment centers. Students could write up their results and share them with the class.





5.                  Research Assignment: Have students research the "glass ceiling" issue. Two available sources include "A Solid Investment: Making Full Use of the Nation's Capital" and "Good for Business: Making Full Use of the Nation's Human Capital." These are the two reports from the federal Glass Ceiling Commission created by the Civil Rights Act of 1991. Copies of these reports are available on the Internet at http://www.ilr.cornell.edu/library.

该用户从未签到

 楼主| 发表于 2010-10-3 20:12:02 | 显示全部楼层
Chapter Summary



The purpose of this chapter is to focus on employee separation and retention.  The chapter is divided in two sections.  The first examines involuntary turnover, that is, turnover initiated by the organization.  The topics covered include principles of justice, progressive discipline, alternative dispute resolution, employee assistance programs, and outplacement counseling.  The second part deals with voluntary turnover, that is, turnover, initiated by employees.  The topics covered include job withdrawal and job satisfaction and how survey feedback interventions can be used to retain high performers.



Learning Objectives



After studying this chapter, the student should be able to:



1.                  Distinguish between involuntary and voluntary turnover, and discuss how each of these forms of turnover can be leveraged for competitive advantage.

2.                  List the major elements that contribute to perception of justice and how to apply these in organizational contexts involving discipline and dismissal.

3.                  Specify the relationship between job satisfaction and various forms of job withdrawal, and identify the major sources of job satisfaction in work contexts.

4.                  Design a survey feedback intervention program and use this to promote retention of key organizational personnel.



Extended Chapter Outline



Note: Key terms appear in boldface and are listed in the "Chapter Vocabulary" section.



Opening Vignette: When Family-Friendly Policies Attack



Employers are more and more using family-friendly policies to help recruit and retain valued workers.  These policies include referral services, corporate sponsored community child care centers, corporate-owned on-site child-care centers, on-site backup emergency care centers, and after-school care.  These centers reduce absenteeism of employees.  However, some of these programs are being criticized by family-free workers.  Responding to the needs of the increasingly diverse workforce is a challenge for employers.





I.          Introduction



To compete effectively, organizations must take steps to ensure that good performers are motivated to stay with the organization, whereas chronically low performers are allowed, encouraged, or if necessary, forced to leave.  Retaining top performers is not always easy, however.  Similarly, the increased willingness of people to sue their employer, combined with an unprecedented level of violence in the workplace, has made discharging employees legally complicated and personally dangerous.



A.        Involuntary turnover—turnover initiated by the organization (often among people who would prefer to stay)



B.         Voluntary turnover—turnover initiated by employees



II.         Managing Involuntary Turnover



Despite a company’s best efforts, some employees will occasionally fail to meet performance requirements or will violate company policies while on the job.  The company must then utilize its discipline policy that may lead to discharge.  



·                    The employment-at-will doctrine is a policy that allows for termination of an employee with or without a “good or just cause.”  Today, however, this policy is not always upheld and employees sometimes sue their employers for wrongful discharge.



·                    In addition to the financial risks associated with a dismissal, there are issues related to personal safety.  Violence in the workplace has become a major organizational problem in recent years.



·                    A standardized, systematic approach to discipline and discharge is therefore necessary.



A.        Principles of Justice



1.                  Outcome fairness refers to the judgement that people make with respect to the outcomes received relative to the outcomes received by other people with whom they identify (i.e., referent others).





2.                  Procedural justice focuses specifically on the methods used to determine the outcomes received (Table 10.1).



3.                  Interactional justice refers to the interpersonal nature of how the outcomes were implemented (Table 10.2).



Competing Through Sustainability



You’re Fired!  What You Should (and Shouldn’t) Learn from “The Donald”



“The Apprentice” was a highly successful business survival reality show.  As supervisor, Trump executed terminations that are both positive and negative approaches.   



B.         Progressive Discipline



Effective discipline programs have two central components—documentation and progressive punitive measures (Table 10.3). This may start with an unofficial warning for the first offense, followed by a written reprimand for additional offenses. At some point, later offenses may lead to a temporary suspension and eventually termination.



C.        Alternative dispute resolution is a method of resolving disputes that does not rely on the legal system.  In general, this form of resolution proceeds through four stages: open door policy, peer reviews, mediation, and arbitration (Table 10.4).



D.        Employee assistance programs (EAPs) attempt to ameliorate problems encountered by workers who are drug dependent, alcoholic, or psychologically troubled. They are basically referral services that supervisors and employees can utilize in order to seek professional treatment. Companies may use carve‑out plans—where they incor­porate mental health benefits into the EAP, which is a single vendor in charge of all mental health benefits.



1.        EAPs are usually identified in official documents published by the employer (employee handbook). Both supervisors and employees may be trained in using the referral system. Evaluation of their success is important because they are just evolving.





2.         There are several issues in controversy regarding EAP: whether various treatments (such as inpatient alcoholism treatment) are effective and whether to use an in‑house EAP or whether to contract with an outside agency.



E.         Outplacement counseling tries to help dismissed employees manage the transition from one job to another.



III.       Managing Voluntary Turnover



A.                 Process of Job Withdrawal—Progression of withdrawal is a theory that dissatisfied individuals enact a set of behaviors in succession to avoid their work situation (see Figure 10.1).



1.         Behavior Change



a.                   An employee's first response to dissatisfaction would be to try to change the conditions that generate the dissatisfaction. This could lead to supervisor‑subordinate confrontation, perhaps even conflict, as dissatisfied workers try to bring about changes in policy or upper‑level personnel.



Example: Ray, who is dissatisfied with his new job assignment, has scheduled a meeting with his supervisor to let her know how he feels about the changes.



b.                  When employees are unionized, dissatisfaction leads to an increased number of grievances. When employees are not unionized, dissatisfaction can lead to the formation of a union.



c.                   Employees sometimes initiate change through whistle blowing, making grievances public by going to the media.



Example: Marjorie discovered that her firm was charging the government for services that were not performed. She blew the whistle and reported this action to the appropriate government officials.





d.                  Employees can sue their employers when the disputed policies relate to an aspect of employment that is covered by legislation ­(race, sex, safe working conditions, etc.)



2.         Physical Job Withdrawal



a.                   A dissatisfied worker may be able to solve his or her problem by leaving the job. This could take the form of an internal transfer.



b.                  If the source of dissatisfaction relates to organization wide poli­cies, organizational turnover is likely.



c.                   In addition to the overall turnover rate, companies should also look at who is leaving. Turnover rates among minorities and women at the managerial levels are sometimes two to three times that of white males.



d.                  Another way of physically removing oneself from the dissatis­fying work is to be absent. Like turnover, absenteeism is disrup­tive and costly. It has been estimated that absenteeism costs organizations an average of $505 a day for large employers and $662 a day for employers with fewer than 100 employees. Absenteeism problems in organizations have been increasing lately, especially in the manufacturing sector.



e.                   Tardiness is another way to physically remove oneself from work. Tardiness can also be disruptive and costly, especially when others depend on the tardy individual.



3.         Psychological Withdrawal—Employees that are unable to physically remove themselves from the situation may "psychologically disengage" themselves from their jobs.



a.                   If the primary dissatisfaction has to do with the job itself, the employee may display a very low level of job involvement. Job involvement is the degree to which people identify themselves with their jobs. People who are uninvolved with their jobs consider their work an unimportant aspect of their lives, which makes it more difficult to motivate them.





b.                  If the dissatisfaction is with the employer as a whole, the employee may display a low level of organizational commit­ment. Organizational commitment is the degree to which an employee identifies with the organization and is willing to put forth effort on its behalf. Individuals who have low organiza­tional commitment are often just waiting for the first opportu­nity to quit their jobs, and they are also very difficult to motivate.



B.                 Job Satisfaction and Job Withdrawal



1.                  Job satisfaction is a pleasurable feeling that results from the perception that one's job fulfills or allows for the fulfillment of one's important job values.



2.         Three important aspects of job satisfaction are values, perceptions, and importance.



a.                   Job satisfaction is a function of values.



b.                  Different employees have different views of which values are important, and this is critical in determining the nature and degree of their job satisfaction.



c.                   It is an employee's perception of the situation relative to the employee's values that matter.



d.                  People's perceptions are strongly influenced by their frames of reference, a standard point that serves as a comparison for other points and thus provides meaning.



3.         Frame of Reference is a standard point that serves as a comparison for other points and thus provides meaning.



a.         It usually reflects average past experiences.



b.         It can also reflect perceptions or other peoples’ experiences.





C.                 Sources of Job Dissatisfaction—This section focuses on the four cate­gories of organizational life that can cause dissatisfaction among employees.



1.                  Personal Disposition—Since emotions and dissatisfaction ulti­mately reside with the person, individual differences play a role in these outcomes.



a.                   Negative affectivity is a term used to describe a dispositional dimension that reflects pervasive individual differences in satisfaction with any and all aspects of life.



b.                  Employees high in negative affectivity report higher levels of aversive mood states, including anger, contempt, disgust, guilt, fear, and nervousness across all contexts.



c.                   People who are high in negative affectivity tend to focus exten­sively on the negative aspects of themselves and others and may be relatively dissatisfied regardless of what steps the organiza­tion or the manager takes.



d.                  Research has shown that negative affectivity in early adoles­cence is predictive of overall job dissatisfaction in adulthood. Also, research on identical twins who were raised apart suggests that there may be a genetic component. Thus, there is evidence that these people may be relatively dissatisfied regardless of what steps the organization takes.



e.                   Core self-evaluations are basic positive or negative bottom-line opinions that individuals hold about themselves.



1.         Positive core evaluation—person’s self-image on a number of more specific traits



a.         Includes: high self-esteem, high self-efficiency, internal locus of control, and emotional stability





2.         Negative core evaluation—person’s tendancy to attribute dissatisfying features of their lives or work to the acts of other people, who they blame for all their problems.



a.         These people are less likely to work toward change, instead either doing nothing or acting agressively twoeard those they blame for their misfortunes.



2.         Tasks and Roles



a.         Three primary aspects of tasks affect job satisfaction: job complexity, degree of physical strain and exertion on the job, and the value the employee puts on the task.



1.         There is a strong, positive relationship between task complexity and job satisfaction.



2.         Boredom generated by simple, repetitive jobs and boredom generated by jobs that monitor critical events that rarely occur can lead to a lack of concentration.



3.         Research suggests that in simple jobs with minimal customer contact, the use of personal‑stereo headsets can improve performance.



4.         When the job is valued, people, such as volunteers, will often perform that job for low pay or for no pay and feel a great deal of job satisfaction.



b.                  Job enrichment, referring to a specific way to add complexity and meaningfulness to a person's work, is one intervention that explicitly focuses on the task as a source of dissatisfaction. Job enrichment is based on the job characteristics theory.



c.         Job rotation, the process of systematically moving a single indi­vidual from one job to another over the course of time, is another task‑based intervention.





d.         The person's role in the organization can be defined as the set of expected behaviors that both the person and other people who make up the social environ­ment have for the person in that job.



1)                  Role ambiguity refers to the level of uncertainty about what the organization expects from the employee in terms of what to do or how to do it.



Example: DeArmond just started his job as a management trainee, but he has not been given a job description and he is not sure what he is supposed to do.



2)                  Role conflict is the recognition of incompatible or contradictory demands by the person who occupies the role.



Example: Etta's boss told her to cut costs by 20 percent, and he also told her to expand services to customers. Etta is not sure how to expand services and also cut costs at the same time.



3)                  Role overload is a state in which too many expectations or demands are placed on the person, and role underload refers to the opposite problem.



Example: One department of a bank that was recently down­sized is trying to provide the same set of services with a staff of 6 instead of 70 people. This would be an extreme case of role overload.



4)                  Role‑analysis technique is a method that enables a role occupant and other members of the role occupant’s role set to specify and examine their expectations for the role occupant (text Figure 10.2).





Competing Through Globalization



How Do You Say Role Overload in French?  You Don’t



A four-year old law in France reduced the national workweek from 40 hours to 35 hours.  While Americans are experiencing crushing expansion of role overload, the French have enjoyed low levels of role overload.  However, these moves do little to help lower unemployment rates in France, as had been hoped.





3.         Supervisors and co-workers—The two primary sets of people in an organization who affect job satisfaction are co‑workers and supervisors.



a.         A person may be satisfied with his or her supervisor and co­workers for one of three reasons:



1)      The person may have many of the same values, attitudes, and philosophies that the co‑workers and supervisors have.



2)      The person may be satisfied with his or her supervisor and co‑workers because they provide social support, the degree to which the person is surrounded by other people who are sympathetic and caring.



3)      One's supervisor or co‑worker may help the person attain some valued outcome.



b.         Because a supportive environment reduces dissatisfaction, many firms foster team building both on and off the job (softball and bowling leagues).



4.         Pay and Benefits—For many people, pay is a reflection of self worth, so pay satisfaction takes on critical significance when it comes to retention.



a.                   Pay level—the absolute amount of income associated with the job





b.                  Pay raises—pay increments over time



c.                   Pay structure—satisfaction with the pay structure deals with how happy the person is with the manner in which pay within the organization is rank ordered across different job categories.



d.                  Benefits



D.        Measuring and Monitoring Job Satisfaction



1.                  Most attempts to measure job satisfaction rely on workers' self-­reports. For example, the Job Descriptive Index (JDI) empha­sizes various facets of work: pay, the work itself, supervision, and co‑workers (text Table 10.5).



E.         Survey Feedback Interventions



1.         Reasons for routinely surveying employee attitudes include the following:



a.         It allows the company to monitor trends over time.



b.         It provides a means of empirically assessing the impact of changes in policy.



c.        If a company uses a standardized scale, it can compare itself with others in the same industry.



d.         If a company provides feedback to employees and provides a corresponding action plan to deal with specific problems, what was once dissatisfaction can become a plus.



2.         Some scales emphasize overall satisfaction (text Figure 10.3).



3.         Engaging in an ongoing program of employee satisfaction provides a means of empirically assessing the impact of changes in policy (text Figure 10.4).





4.         When these surveys incorporate standardized scales, they often allow the company to compare itself with others in the same industry along these dimensions (Figure 10.5).



5.         A systematic survey program also allows the company to check for differences between units and hence benchmark “best practices” that might be generalized across units (Figure 10.6).



6.         Yearly surveys and periodic question and answer sessions give employees a constructive outlet for voicing their concerns and frustrations. Voicing is a formal opportunity to complain about one’s work situation.



Competing Through Technology



Taking the Pulse of Your Organization: Don’t Wait for Your Annual Checkup



Assessing employee attitudes is a critical process, however most organizations treat it more like an annual checkup rather than a day-to-day activity.  Increasingly, organizations are conducting surveys on the Web, using automated software to help perform the analysis and reporting process.  The problem with many annual surveys is that by the time problems are identified, it is often too late to act upon them.  Overall, electronic surveys are superior to conventional surveys because they are cheaper, faster, more frequent, and process richer data.





A Look Back



The opening vignette shows how a family-friendly policy is perceived by various groups of workers, and how this in turn affected work attitudes and retention.  



Questions



1.         Based upon our discussion of outcome fairness, procedural fairness, and interactional fairness, do you think that family-friendly policies are unfair to single employees?  If so, what could be done to reduce this unfairness without significantly increasing costs?





Student answers will vary.  Some may state that family-friendly policies are fair, because employees with families have significantly more responsibilities than do single employees.  Some may state that these policies are unfair because single employees receive less compensation because they don’t have families.  To reduce this unfairness, possible suggestions include paying singles more, offering them more annual paid leave, or maybe sponsoring a singles night out on weekends.



2.         What specific facets of job satisfaction or role problems are being targeted by these kinds of policies, and what facets or problems do they ignore?  What facets of job satisfaction or role problems might be more of a concern for single people, and what steps can organizations take to address such issues?



            Student answers will vary.  Family-friendly measures aim to reduce turnover and lower tardiness and absenteeism.  They ignore the added financial responsibilities employees with families incur.  Single people may be focusing more time and energy to their jobs because they don’t have many outside responsibilities.  Therefore, organizations could offer singles more paid vacation time or paid leave.   



3.         Given the increased diversity of workers and their personal situations, how can a systematic employee survey process be used to quickly identify problems and solutions for different subgroups?



            Student answers will vary.  The surveys should be confidential, and should determine whether respondents are single or have families.  Surveys should be handled electronically to reduce expenses and time.  They should be administered frequently; not just annually.







Chapter Vocabulary



These terms are defined in the "Extended Chapter Outline" section.



Involuntary Turnover

Voluntary Turnover

Employment-at-will doctrine

Outcome Fairness

Procedural Justice

Interactional Justice

Alternative Dispute Resolution

Employee Assistance Programs

Outplacement Counseling

Progression of Withdrawal

Whistle-blowing

Job Involvement

Organizational Commitment

Job Satisfaction

Frame of Reference

Negative Affectivity

Job Enrichment

Job Rotation

Role

Role Ambiguity

Role Conflict

Role Overload

Role Analysis Technique

Voicing





Discussion Questions



1.         The discipline and discharge procedures described in this chapter are systematic but rather slow. In your opinion, are there some offenses that should lead to immediate dismissal, and if so, how would you justify this to a court if you were sued for "wrongful discharge"?



Offenses that are violent in nature or that endanger other members of the work force should most likely lead to immediate discharge. Additionally, large‑scale thefts or significant criminal activity might be treated harshly. To justify immediate dismissal to a court, the magnitude of the offense should be carefully described, and evidence should be presented that shows the guilt of the employee. Additionally, if the offense endangered other employees, this should be shown to the court.



2.                  Organizational turnover is generally considered a negative outcome, and many organizations spend a great deal of time and money trying to reduce it. Can you think of some situations in which an increase in turnover might be just what an organization needs? Given the difficulty of terminating employees, can you think of any organizational policies that might promote the retention of high performing workers but promote voluntary turnover among low performers?



Situations in which an increase in turnover might be just what an organization needs are situations where "new blood" can bring in new ideas and enthusiasm lacking in current employees.



One organizational policy that might promote the retention of high ­performing workers and promote voluntary turnover among low performers is a policy that rewards only high performers. Low performers would tend to leave because they would not get rewarded, and high performers would stay because they were getting rewarded.





3.         Three popular interventions for enhancing worker satisfaction are job enrichment, job rotation, and role analysis. What are the critical differences between these interventions, and under what conditions might one be preferable to the others?



Job enrichment involves redesigning the job so it is more complex and has more responsibility, whereas job rotation simply rotates employees to different jobs, usually at the same level of responsibility. Role analysis looks at the role expectation for a particular job by involving the jobholder as well as supervisors, co‑workers, subordinates, and so on. With role analysis, the job does not change, but the job changes with job enrichment. With job rotation, other jobs are involved.



Job enrichment is more for situations in which employees may desire more responsibility or complexity in their jobs. Job rotation, which can be used for cross‑training, also increases work complexity. Role analysis is mainly for clarifying what the expectations are for a particular job.



4.         If off‑the‑job stress and dissatisfaction begin to spill over and create on‑the‑job problems, what are the rights and responsibilities of the human resource manager in helping the employee overcome these problems? Are intrusions into such areas an invasion of privacy a benevolent and altruistic employer practice or simply a prudent financial step taken to protect the firm's investment?



Students' answers will vary. Some students may think that managers' helping the employee with personal problems is inappropriate, whereas other students might think it is a humane thing to do, and still others may think that it makes good business sense.



5.         Discuss the advantages of using published, standardized measures in employee attitude

surveys. Are there any conditions in which employers need to develop their own measures for such surveys? Where would one turn to learn about how to do this?      



Advantages of using published, standardized measures in employee attitude surveys is that they are readily available and quick to assess and use, they allow for possible comparisons with other companies, they have often been tested for validity, and so forth.

Employers need to develop their own measures if they want to measure some aspect of work that is specific to their organization, such as a new health plan.

To develop your own attitude survey, you could review standardized surveys that are on the market and also work with someone who has developed these surveys before.





Manager’s Hot Seat Exercise: Whistleblowing: Code Red or Red Ink?



I.  Introduction



This scenario depicts a perceived ethical dilemma for an employee who has discovered information about a client that could be putting human lives in danger.  His management wants to delay any action on the matter until future investor funding is secured.  This scenario may be used for instructors teaching Management because it presents a challenging problem for a middle manager in an ethical bind and caught between two opposing points of view from the levels above and below her.



II.  Learning Objectives



To assess student’ understanding of the role of the middle manager when faced with difficult ethical dilemmas.  
To analyze Whistleblower statutes and understand their purpose and value for employees and managers.


III.  Scenario Description:



Overview:  An employee at a hospital management company files a report with his supervisor on patient injury and death caused by staff errors.  There is an investor meeting scheduled for one month following receipt of the report and our manager has been told by her superiors not to release the report until after that meeting. The employee who created the report disagrees with this decision, puts up a fight, and threatens to leak the report directly to the media.



Profile:

·        Susan Novrotsky is a Senior Account Executive managing up to 15 accounts per quarter and supervising a team of six people.  Susan’s main functions include new account development, client management, and managing the planning processes for hospital restructuring.

·        Daniel Yoshi has been working with Woodland Management Services for five years within Susan’s department.  His functions include research and analysis of hospital structure, general competency, cost-efficiency, human resource assessment, and liability issues.  Daniel’s research reports are used as the basis for the Woodland’s restructuring plan and capital allocation.





References:  The references included in the DVD are:

·        Concepts in Whistleblowing (PPT 5-5)

·        The Whistleblowers’ Protection Act (PPT 5-7)

·        Whistleblower Provisions of Sarbanes-Oxley (PPT 5-8 to 5-10)

·        United States Protection Policies (PPT 5-11)



Back History:  Novrosky and Yoshi have been working at Woodland for over five years.  Novrosky is managing a new account for Hillshire Hospital.  The current objectives are to analyze the efficiency of the staff and the organization as a whole and generate a restructuring plan. Yoshi was directed to research these issues and generate a status report.



Yoshi sent Novrosky a progress email with his findings, highlighting his concern over the exorbitant number of accidents, injuries and unnecessary deaths that have occurred at the hospital over the past four years.  The numbers greatly exceed the national rate of accidental harm to patients. Yoshi strongly believes that immediate action is necessary.  [Artifact 1 – Yoshi email]



Novrosky forwards the email to her supervisor, Walsh.  Novrosky has never come across such severe findings and is unsure how to proceed.  [Artifact 2- Novrosky email]



Walsh responds via email, instructing Novrosky and Yoshi to hold off on sharing the research.  Hillshire Hospital has an investor meeting in exactly one month, and if the meeting goes poorly, which it would should this report circulate, there would be no funding for a restructuring project.  Walsh agrees that this is a serious issue, but insists it would be foolish and more detrimental to act on it immediately. [Artifact3- president email]



Novrosky leaves a voicemail with Yoshi relaying Walsh’s instruction –Artifact 4 –Novrosky’s voicemail recording]



Scene Set-up: Yoshi is agitated by the voicemail and barges into Novrosky’s office.



Scene Location: Novrosky’s Office, Woodland Management Services; lunch time





The Meeting - Summary:  Daniel Yoshi approaches Susan in her office clearly agitated about the email and voicemails he received indicating that no action will be taken on his research findings until after the investor’s meeting in one month.  Susan tries to convince Daniel that no action should be taken at this point because that would prevent further funding that is required to restructure the hospital.  Daniel contends that taking action now would save lives.  Susan reminds Daniel of what the business goals are and to stay focused on that.  Daniel’s conscience will not let him accept Susan’s position and he threatens to go to the media if Woodland Management Services does not notify the public and shareholders of Hillshire Hospital’s problem.  The scenario ends with Daniel’s ultimatum.



Afterthoughts – Summary:  Susan believes that Daniel is so emotionally involved with his work that he can’t see the bigger picture and what consequences his actions would really have.  She indicates that Daniel needs to feel valued and that his work is important so that he doesn’t go to the media.  Although Susan admits she can’t prevent Daniel from going to the media, she let others in the organization know about his intentions.  She feels she needs to balance her obligation to Daniel to reach his career goals with the needs of the organization which is to get the funding.



Dossier:  The specific artifacts included in the DVD are:



1.      Yoshi’s email to Novrosky detailing the research findings, and calling for immediate action.

2.      Novrosky’s email to Walsh forwarding Yoshi’s email, and asking for advice on managing the situation.

3.      Walsh’s email to Novrosky replying with instructions to hold back the info.

4.      Novrosky’s voicemail to Yoshi instructing him to hold  the report until after the investor meeting.



IV.  Discussion Questions:



The References and related Discussion Questions may be found in PowerPoint slides 5-1 to 5-11 on the instructor’s side of the text’s Website.





Learning Objective #1: To assess student’ understanding of the role of the middle manager when faced with difficult ethical dilemmas



What competing organizational goals are present in this scenario?


The organization clearly has a goal of receiving funding so it can continue operations and ultimately reach its goal of restructuring the hospital.  Another goal is to research hospitals in order to have quality information on which to make decisions about its management.  In this scenario, the research called for an action in conflict with receiving funding.



What could the organization do to guide managers and employees in handling situations like this one?


The organization should have clearly defined mission and vision statements.  In addition, well-communicated values would serve to guide employee behavior when dealing with situations such as these.  Missions, visions, and values are an important part of an organization’s culture and can be used during conversations among employees regarding difficult decisions.



´        1.  Susan’s strategy seems to be to:

A.     Minimize problem

B.     Re-assign research

C.     Commiserate



Susan seems to minimize the problem by suggesting that Daniel’s research is incomplete.  She suggests that Daniel research the situation further as a way to stall Daniel from taking action before the investor meeting.





Daniel was clearly not satisfied with Susan’s explanation.  What could Susan have done differently to convince Daniel to wait?


Susan should probably have used a strategy based on reasons that Daniel would appreciate and understand.  For example, she could have pointed to aspects of the research that do not show a clear link between his findings and accidental deaths.  She could have provided alternative explanations and provided a plan for testing those before jumping to conclusions.  While Daniel was emotional and passionate about his work, his stated repeatedly that his primary concern was the health of the hospital patients.  Susan seemed to think Daniel was upset because he didn’t feel his work was valued.  Therefore, Susan didn’t address Daniel’s exact concern and how her plan would also serve to protect patients.  She seemed to be very focused on the business objective of making money rather than the endangerment of people in the hospital.  Thus, not correctly interpreting Daniel’s concern caused her to use an ineffective influence strategy.



´        2.  Primary reason for communication failure:

a.     Danny’s not listening

b.     Susan’s not listening

c.     A misunderstanding



Student’s answers will vary.  Discussion could focus on behaviors that indicated where active listening broke down.  Susan did not seem to address Daniel’s primary concern directly (see answer to #3 above).



´        3.  What is Susan’s key reason for “holding off’?

a.     Incomplete research

b.     Funding at risk

c.     No need for disclosure



Susan’s motives are unclear.  From her email, she initially seemed to agree with Daniel’s findings and interpretation.  She says she met with Walsh and then supported his position with Daniel.  It’s difficult to ascertain whether she truly believes the research is incomplete or whether she is more motivated by the fear of lack of funding.





Learning Objective #2: To analyze Whistleblower statutes and understand their purpose and value for employees and managers.



If Daniel went to authorities with his information, would he be making a “good faith allegation”?  Refer to “Concepts in Whistleblowing” (PPT 5-5) for your answer.  


Yes, Assuming his research is thorough and accurate.



Susan states that there is nothing she could do to stop Daniel from “blowing the whistle.” Do you agree there is nothing she can do to stop Daniel?  Why or why not?  
While she can’t physically restrain him or threaten him in any way, Susan could make an attempt at a compromise solution that would appease Daniel’s concerns and meet the needs of the organization.



´        4. Why does Susan focus on goals?

A.     Establish “big picture”

B.     Distract Daniel

C.     Outline Daniel’s job



Susan focuses on goals because she is trying to get Daniel to see the bigger picture in that taking action now would actually be detrimental to his ultimate goal of saving lives.  However, she doesn’t make that point very convincingly.



Susan does indicate that she would tell others in the organization of Daniel’s intentions to blow the whistle.  What are the pro’s and con’s of this action?


Pro’s - others should know that bad press or a public relations disaster could occur and be prepared for it.

Con’s - someone could take an adverse action toward Daniel in retaliation for his intended or committed actions.  He may be protected under the Whistleblowers’ Protection Act and could file suit which would cause even more bad press.



´        5. Susan’s response should be:

A.     Acquiesce to demand

B.     Fire Daniel

C.     Call Walsh





Susan should probably call Walsh to discuss the issue further.  She and Walsh may be able to come up with a compromise action that would meet both Daniel’s and the organization’s needs.



Given what you saw in this scenario, if an adverse action was taken toward Daniel in retaliation, would Daniel be protected under the Whistleblowers’ Protection Act (see PPT 5-7)?  Why or why not?


It would be illegal for the company to take an adverse action against Daniel because he alleges a problem that could affect one’ health and safety.  However, it is unclear whether he is protected by this law.  While he did report this to his supervisor he wants action taken immediately which may or may not be perceived as a reasonable amount of time.  Further, the organization does seem to want to correct the problem, they are just concerned that taking immediate action now will prevent real improvements in the future due to lack of funding.





Exercising Strategy: Feeling Insecure about Airline Security



Questions



1.         Many now look back at the way airport security was selected and managed prior to the 9/11 attacks with disbelief.  What was the relationship between the airlines, the airports, the government, and the security agents prior to the attacks?  How did the nature of these relationships explain why airport security agents were managed so poorly?



            Student answers will vary.  Prior to 9/11, airlines and airports were earning profits, but little was being done to increase job satisfaction.   According to the article, the security agents were isolated from other airport workers. There was also a sense of resentment, because security workers were being paid less than many other airport workers, and the job was less satisfying.  This explains the high turnover rate for security agents in airports.  





2.         How has the relationship between the airlines, airports, government, and security agents changed since 9/11, and in what ways has the new Transportation Safety Administration changed the nature of airport security work?



            Student answers will vary.  After 9/11, many security measures were imported from Europe and Israel.  Today, airline screening is treated as a police function.  In addition, there is a close emotional attachment between the security personnel and the airlines.  The task of security agents has become more important, even patriotic.



3.         Much of the task in keeping air travel safe requires teamwork between the airlines, the airports, the government, and the security agents.  In what way does high interdependence between people working together heighten the negative impact of employee turnover and performance?



            Student answers will vary.  Employee turnover and performance can be negatively impacted by high interdependence because employees may feel that they are helpless when situations surface.  Even if problems are reported in the proper fashion, change comes slowly and laboriously.  As a result, more airling employees may leave the field or become disgruntled.   



Managing People: From the Pages of BusinessWeek



The Costco Way



Questions



1.         Many organizations that skimp on pay, benefits, and working conditions attribute this to the intense competition they face.  Does Costco face competitive pressures, and if so, how does their “pro-employee attitude” help them overcome these pressures?



            Student answers will vary.  Like most organizations, Costco does face threats from competitors, particularly Walmart and Sam’s Club.  To overcome these pressures, Costco’s pro-employee attitude helps reduce turnover and improve employee performance.  





2.         In what ways are there hidden costs associated with the “cheap labor model” epitomized by Costco’s major competitor, Wal-Mart?



            Student answers will vary.  Although Wal-mart may benefit financially from cheap labor, employees there may not be happy to be earning such low wages.  As a result, turnover is higher at Wal-mart, and the company has to spend more annually on training new workers and compensating for mistakes made by new workers.



3.         In the article, one investment analyst states “At Costco, it is better to be an employee or a customer than a shareholder.”  From a strategic perspective, when are the goals of these three different groups of stakeholders (customers, employees, shareholders) the same or different?  If you are a manager, how do you balance the needs of these three constituencies?



            Student answers will vary.  The goals for these three groups will be the same when high employee wages reduce costs for the company, resulting in low prices for customers, and high profitability for shareholders to due high volume.  However, generally high employee wages and low costs for customers yield low shareholder growth.  As a manager, one could balance the needs of these three constituencies by continuing to make Costco a great place to work, and continually finding ways to lower costs.  This would eventually improve shareholder value.   





Additional Activities



Teaching Suggestions



There are many interesting topics related to job satisfaction and job with­drawal. Some of these topics are suggested for class debate, discussion, and brainstorming. Several articles that are suggested provide additional information on the specific topics. Also, a case is given that can be used as a role-play dealing with absenteeism.



1.         The results of several studies examining the attitudes of workers are included in the following article. Students could be assigned to read the article and comment on their agreement or disagreement with the results. "What Do Workers Want?” INC, November 1992' pp. 101‑102. Some of the other results reported in the article are as follows: How important is each of the following characteristics to you? How satisfied are you with it in your current job?



Percent of Workers Who



Ranked It as           Said They

                                                                                            Very Important       Were Satisfied



Good health insurance and other benefits                                   81%                             27%

Interesting work                                                                        -8                                 41

Job security                                                                              -8                                 35

Opportunity to learn new skills                                      68                                31

Having a week or more of vacation                                           66                                35

Being able to work independently                                              64                                42

Recognition from co‑workers                                                    62                                24

Regular hours (no weekends, no nights)                         58                                40

Having a job in which you can help others                                 58                                34

Limiting Job stress                                                                    58                                17

High income                                                                             56                                13

Working close to home                                                 55                                46

Work that is important to society                                               53                                35

Chances for promotion                                                 53                                20

Contact with a lot of people                                                      52                                45

Flexible hours                                                                           49                                39



Source: Gallup Poll, 1991.





Which one of the following would most give you the feeling of success in your life?



Percent Choosing Each Factor



Happy family life                                                                                   62

Ability to do some good in the world                                                     15        

Earning lots of money                                                                            10

Position and prestige in your work                                                         6

Involvement in some creative activity                                         4

Fame                                                                                                    1

Don't know/no answer                                                              2



Source: The Chivas Regal Report on Working Americans, 1989.



2.                                                                               Stressed Out at work: Chin Up, You Can Blame It on the Boss.

3.                                                                       

If you are stressed out at work, you may well have a lousy boss. Sure, you've known this all along, but now there is a study that shows it: Poor supervisors are one of the chief sources of workplace stress, according to a recent survey by St. Paul Fire and Marine Insurance Company, the nation's largest medical liability insurer.

Friction with supervisors can lower productivity, diminish quality, and increase absenteeism, according to Stacey Kohler, the psychologist who supervised the survey of 215 companies. Of the employees who said they had a bad supervisor, 76 percent said co­workers talk about leaving the company, 68 percent said they are tired during the workday, and 65 percent said their productivity could be improved.

The survey revealed that one in three workers has neutral or negative feelings about his or her boss. The survey also showed that problems with supervisors are likely to carry over into employees' home lives, straining their relationships and driving them to substance abuse problems. The study showed that problems at work are more likely than personal problems to have a negative effect both at work and at home.



The survey showed that employees are looking for a few basic qualities in a manager: someone who sets clear and reasonable goals, someone who is not overly critical, and someone who listens to employees and involves them in decisions that affect them.

VIC Inc. of Bloomington, one of the companies that participated in the study, found that its 220 employees largely were satisfied with their managers. But half the employees at the semiconductor manufacturer indicated that their performance evaluations did not help them improve.





How should performance evaluations be conducted in order to actually help employees improve? The performance evaluation process consists of measures of performance as well as the process of communication of performance information to employees.



3.         There are several written stress tests that you could give to students. One test is known as the Glazer Stresscontrol Life‑Style Questionnaire, which helps students determine if they are Type A or Type B personalities. This test can be found in Business Week's Executive Health, (McGraw‑Hill, Inc., 1987). Another written test is called the Social Readjustment Rating Scale, which measures the amount of change in one's life and equates that to stress. This scale can be found in T. H. Holmes and R. H. Rahne, "The Social Readjustment Rating Scale," Journal of Psychosomatic Research ll, (1967) pp. 213‑18.



4.         One interesting article dealing with whistleblowing is "Blowing the Whistle without Paying the Piper," Business Week, June 3, 1991, pp. 138‑139. Students could be asked to read this article and also find other articles on the topic and write a report on the topic.



5.         One classroom activity is to have students list or brainstorm the characteristics that they want an employer to have. In other words, what makes a company a good place to work? To supplement this discussion compare the class's list with the six criteria (pay/benefits, opportunities, job security. pride in work/company, openness/fairness, and camaraderie/friendliness) listed in the article “The Work Place 100,” USA Weekend, January 22‑24, 1993, pp. 4‑6. This article also lists the 100 best companies to work for in America.



6.         Do you have to be a workaholic to succeed in the business world? Students could debate this issue. One article that sheds some interesting ideas on the subject is "You Don't Have to Be a Workaholic, Fortune, August 9. 1993, pp. 65‑69.



7.         Humor is often suggested as a great stress reliever. Have students discuss and brainstorm how a company can use humor in the workplace and identify the issues/problems involved with humor in the workplace. An interesting article on the subject is "Building Fun in Your Organization" by David J. Abramis, Personnel Administrator, October 1989, pp. 68‑72.



8.         The following case can also be used as a role-play:





The Absenteeism Problem



Melissa has been an absenteeism problem ever since she was hired two years ago. She has been on probation twice and has been able to improve her attendance enough to get off probation, but the problems seem to be recur­ring. As Melissa's supervisor, you are aware that Melissa has health prob­lems, and she is a single parent with a young child that also has health problems. You believe her absences to be related to these health problems; however, co‑workers are complaining about having to cover for Melissa when she is out, and one customer has complained about not receiving information that Melissa was supposed to have sent last week. Melissa has just returned from being absent for five days due to a recurring illness. She has missed over 12 days in the last three months. You are getting pressure from your boss to terminate Melissa if her attendance does not dramatically improve. You have scheduled a meeting with Melissa for this afternoon.



9.         Library Assignment: Have students report on the Job Absence and Turnover rate for U.S. companies and the trend of this data. The Bureau of National Affairs publishes this data on a monthly basis by industry. From the BNA report on the first quarter of 1993, absence rates rose from a year earlier in almost all size, industry, and geographic categories, with the sharpest increases occurring among larger companies. The BNA also reported that the long period of decline in employee turnover may be at an end, but separation rates remain at the low levels of the past two years. Also, have students research how job absence rates and turnover rates are calculated.



10.       “It Finally Happened: The Cyberfiring, Brought to You by AT&T.” In September, 1995, 8,500 AT&T employees were told they had been made redundant through a company-wide e‑mail message. (Source: People Trends, September 1995, p. 10). Have students discuss the advantages and disadvantages of using e‑mail for such announcements, from the perspective of employers and of employees.

该用户从未签到

 楼主| 发表于 2010-10-3 20:12:29 | 显示全部楼层
CHAPTER 10


Pay Structure Decisions


Chapter Summary


This chapter discusses the nature of the pay structure and its component parts, the pay level, and the job structure. Equity theory is discussed relative to internal and external comparisons that employees make about their pay. Such comparisons may have consequences for employee attitudes and retention. Such comparisons also play an important role in the contro­versy over executive pay. Two administrative tools used in managing pay­ level and job structure components of the pay structure are pay benchmarking surveys and job evaluation. A discussion of globalization suggests why benchmarking is increasingly critical to meet the competition. A theme that runs through this chapter is the need to consider how the organization's strategy matches the compensation plan. For example, skill­-based pay is discussed in terms of the trend toward reducing bureaucracy and communication of compensation being critical to reinforce participa­tion and commitment that is increasingly expected from employees.



Learning Objectives


After studying this chapter, the student should be able to:



1.                  List the major decision areas and concepts in employee compensation management.

2.                  Describe the major administrative tools used to manage employee compensation.

3.                  Explain the importance of competitive labor‑market and product-­market forces in compensation decisions.

4.                  Discuss the significance of process issues such as communication in compensation management.

5.                  Describe new developments in the design of pay structures.

6.                  Explain where the United States stands from an international perspective on pay issues.

7.                  Explain the reasons for the controversy over executive pay.

8.                  Describe the regulatory framework for employee compensation.



Extended Chapter Outline


Note: Key terms appear in boldface and are listed in the "Chapter Vocabulary" section.



Opening Vignette: AMR Unions Express Fury over Management Benefits



This opening scenario discusses how American Airlines workers felt betrayed after retention bonuses and creation of a trust for executive pensions were disclosed. This came shortly after three unions wrapped up voting on whether to accept deep cuts in pay and benefits.  While AMR’s board agreed to protect pension benefits and created retention bonuses for top executives, the company was asking workers to take pay cuts of between 15.6 percent and 23 percent.  Union leaders demanded that executives give up the bonuses and the pension trust.  A senior executive at American said that these measures were taken to retain key employees who are needed to save the company.   



I.                    Introduction


A.           From the employer's point of view, pay is critical in attaining strategic goals.   



1.         Pay has a major impact on employee attitudes and behaviors.



2.         Employee compensation is typically a significant organizational cost. (See Table 11.1 for industry companies of Total Compensation as a Percentage of Revenues.)



B.         From the employees’ point of view, policies having to do with wages, salaries, and other earnings affect their overall income and thus their standard of living.



C.        Pay decisions can be broken into two major areas: Pay Structure and
Individual Pay.   



1.         Pay structure refers to the relative pay of different jobs (job structure) and how much they are paid (pay structure).



2.         Pay level is the average pay in organizations, including wages, salaries, and bonuses.



3.         Job structure is the relative pay of jobs in organizations (i.e., the range of pay often expressed by salary grades).



4.         Pay policies are attached to jobs, not individuals, in order to make the process more manageable and equitable.



II.                 Equity Theory and Fairness


A.                 Equity theory describes a process in which people evaluate the fair­ness of their pay by comparing their pay to that of other people.



1.         A person will compare his or her ratio of perceived outcomes (e.g.,       pay, benefits, etc.) to perceived inputs (e.g., education, effort, experience) to the ratio of a comparison other.



2.         If the person's ratio is higher, research suggests that rationaliza­tion will occur to account for the perceived overpayment. If the comparison other's ratio is higher, the person may attempt to restore equity by reducing one's inputs (e.g., working less), increasing one's outcomes (e.g., asking for a raise), or leaving the company.



3.         Two types of employee social comparisons of pay are espe­cially relevant in making pay‑level and job structure decisions. (See Table 11.2)



a.         External equity pay comparisons focus on what other organizations pay for roughly the same job. These comparisons influence decisions to join and remain in the organization. A market‑pay survey is used by organiza­tions to examine the level of other organizations' pay.



b.         Internal equity pay comparisons focus on what employees within the same organization, but in different jobs, are paid.  In addition, employees make internal equity pay comparisons with others performing the same job.


III.       Developing Pay Levels


A.        An organization faces two important competitive market challenges in deciding what to pay its employees:



1.         Product‑market competition – the challenge to sell goods and services at a quantity and price that will bring a return on investment. An organization that has higher labor costs than its competitors will have to charge higher prices for products of similar quality. This will likely lead to a loss of sales to the competition. Product‑market competition places an upper bound on labor costs and compensation, although productivity differences are also important. Labor costs are the average cost per employee (direct pay and indirect compensa­tion, such as benefits) and staffing level (number of employees). Financially troubled organizations tend to focus cuts on one or both of these.



2.         Labor‑market competition – the amount an organization must pay to compete against other organizations that hire similar employees. Usually these are companies with similar products and those that hire similar employees. If an organization is not competitive, it will fail to attract and retain sufficient numbers of high‑quality employees. Labor‑market competition places a lower bound on pay levels.


B.         Employees as a Resource

1.         This is a philosophy that considers employees to be an investment that will yield valuable returns. Controlling costs through  noncompetitive pay can result in low employee produc­tivity and quality.



2.         Pay policies and programs are one of the most important human resource tools for encouraging desired employee behaviors and discouraging undesired behaviors.


C.        Deciding what to Pay



1.         Deciding pay is discretionary, with a broad range. The organization has to decide whether to pay at, below, or above the market average.



2.         Efficiency wage theory states that wages influence worker productivity.  The benefits of higher wages may outweigh higher costs when the organization's tech­nology or structure depends on highly skilled employees or when the organization has difficulty observing and monitoring employee performance.



            D.        Market Pay Surveys



1.         Benchmarking is a procedure by which an organization compares its own practices against those of the competition. Benchmarking in this situation is done by pay surveys that provide information on the going rates of pay of the competi­tion.



2.         Benchmarking is typically accomplished through use of pay surveys, which provide information on going rates of pay among competing organizations.  The following issues must be determined before pay surveys are used:



a.         Which employers should be included in the survey?



b.         Which jobs are included in the survey?



c.         If multiple surveys are used, how are all the rates of pay weighted and combined?



3.         Product‑market comparisons will be more important when:



a.         Labor costs represent a large share of total costs.



b.         Product demand is elastic.



c.         The supply of labor is inelastic.



d.         Employee skills are specific to the product market.



4.         Labor‑market comparisons will be more important when:



a.         Attracting and retaining employees is difficult.



b.         The costs of recruiting are high.



5.         As well as knowing what other organizations are paying, it is necessary to know what those organizations are getting in return for their investment in employees. Ratios such as revenues to employees or revenues to labor cost could be used.



            E.         Rate Ranges



1.         Use of rate ranges permit a company to recognize differences in employee performance.  Different employees in the same job may have different pay rates.



F.         Key jobs and nonkey jobs



1.         Key jobs are benchmark jobs that have relatively stable content and are common to many organizations so that market‑pay survey data can be obtained.



2.         Nonkey jobs are unique to organizations and cannot be directly valued or compared through the use of market surveys.



Example: Typical key jobs would include administrative assistant, department manager, controls engineer, and so on.



IV.       Developing a Job Structure



A.        A job structure refers to the relative worth of various jobs in the organization, based on internal comparisons.



B.         Job evaluation is an administrative procedure that measures a job's worth to the organization.



1.         The evaluation process is composed of compensable factors, which are the characteristics of jobs that an organization values and chooses to pay for.



C.        Job evaluators often apply a weighting scheme to account for the differing importance of the compensable factors to the organization. Either a priority weight can be assigned using expert judgment about the importance of each factor, or weights can be derived statistically by how important each factor seems to be in determining labor‑market pay. (Table 11.3 indicates an example of a three-factor job evaluation system).  



Example: One of the most popular commercial point‑factor systems on the market is The Hay Plan, which includes three compensable factors: know‑how, problem solving, and account­ability.



D.        Developing a Pay Structure



1.         Includes examining and balancing both internal and external comparisons. There are significant differences in employers in terms of whether they place priority on internal or external comparisons (Table 11.4 indicates an example of both types of data for 15 jobs).



2.         Three pay‑setting approaches include:



a.         Market survey approach -  This approach has the greatest emphasis on external compar­isons.  It bases pay on market surveys that cover as many key jobs as possible (for an example, see Table 11.5).



-  A pay‑policy line is a mathematical expression that describes the relationship between a job’s pay and its job evaluation points. (Figure 11.1).



b.         Pay policy line - The second pay‑setting approach combines information from internal and external comparisons to derive pay rates for both key and nonkey jobs.  However, actual market rates are not used for key jobs.



c.         Pay grades – Group jobs into a smaller number of pay grades.  (Table 11.6 and the last column of Table 11.5 demonstrate a five‑grade structure).



-         A pay grade is grouping jobs of similar worth or content together for pay administration purposes.



-         The range spread is the distance between the minimum and maximum amounts in a pay grade



-         Disadvantages of using grades are that some jobs may be underpaid while others may be overpaid.



E.         Conflicts Between Market Pay Surveys and Job Evaluation



1.         The relative worth of jobs is quite similar overall, whether based on job evaluation or pay survey data.  However, some inconsistencies do arise.  



2.         In resolving the conflict, emphasizing the internal data would create a situation of overpaying jobs that are paid less in the market, which drives up labor costs (and creates product-­market problems).



3.         If external market data are emphasized and a job is paid lower internally, the comparisons that employees make internally would result in dissatisfaction.



4.         There are no right answers. An organization should consider its strategy and what jobs and/or functions will be critical for success.



F.         Monitoring Compensation Costs



1.         The pay structure represents policy, but practice may not always coincide.



2.         One way to examine the difference between policy and practice is to compute a compa-­ratio, which is the actual average pay for grade/midpoint pay for grade (Table 11.7).



G.        Globalization, Geographic Region, and Pay Structures



1.         Pay structures can differ substantially across countries both in terms of their level and in terms of the relative worth of jobs (See Figure 11.3).



2.         Typically, expatriate pay and benefits continue to be linked more closely to the home country. However, this link appears to be slowly weak­ening and now depends more on the nature and length of the assignment.



V.        The Importance of Process: Communication and Participation



A.        Since pay decisions might be viewed in different ways by different groups in the organization and technology rarely provides a "right" answer, how decisions are made and communicated is critical.



1.         Participation should involve both those who will manage the process and those who will be affected by it (HR staff and line managers). Usually, participation comes in recommending, designing, and communicating a pay program. Typically, pay‑level decisions are only made by top management.



2.         Communication



a.         The consequences of not communicating effec­tively are described in a study in which a group that received pay cuts, but to which was communicated adequate information and remorse, engaged in significantly less theft after the pay cut than a group to whom communication was inadequate.



b.         The effect of communication is likely to be an impact on employees' perceptions of equity. Managers must be prepared to explain to employees why the pay structure is designed the way it is and to judge whether employee concerns about the structure need to be addressed with changes to the structure.



Competing Through Technology



A Chill East Wind



This case discusses how Eastern Europe is losing jobs to Asia.  Hungary, Poland, the Czech Republic, and other European countries are realizing that they are in a rat race with Asian countries in terms of growth and development.  



VI.       Current Challenges



A.        Although most commonly used, job‑based pay structures can create the following problems:



1.         They encourage bureaucracy.



2.         They reinforce top‑down decision making as well as status differentials.



3.         The bureaucracy, time, and cost required to generate and update job descriptions can become a barrier to change.



4.         The job‑based structure may not reward desired behaviors, where the knowledge, skills, and abilities needed yesterday may not be helpful today and tomorrow.



5.         The system encourages promotion‑seeking behavior, but discourages lateral movement.



B.         Responses to problems with job‑based pay structures include the following:



1.         Delayering is reducing the number of job levels. This provides more flexibility in job assignments and assigning merit increases. These broader groups are also called broad bands. (Table 11.8 and Figure 11.4 provide an example of pay bands).



Example: At the beginning of 1996, Metropolitan Property and Casualty Ins. Co. (MetP&C) unveiled "CareerBanding" to its approximately 3,500 employees. A multidisciplined team of 21 individuals studied, analyzed, and designed the new process. Source: "Switching to Broadbanding: One Company's Experience" by W. DeCaporale, American Compensation Association News, April 1996, pp. 8‑12.



2.         A second response to job‑based pay structure problems has been to move away from linking pay to jobs and toward building structures on skill, knowledge, and competency. Competency‑based pay is similar but usually refers to a plan that covers exempt employees (e.g., managers).



a.         Skill‑based pay typically pays individuals for the skills they are capable of using rather than for the job they are performing at a point in time.



b.         Advantages are that flexibility helps promote lower staffing levels and aids in situations where the manufacturing process demands adaptable and flexible responses (e.g., flexible manufacturing, just‑in‑time systems). It has also been suggested that skill‑based plans contribute to a climate of learning and adaptability and give employees a broader view of how the organization functions.



c.         Potential disadvantages:



-         The organization may find it difficult to use all skills effectively (i.e., work design must also change).



-         Employees may acquire skills quickly and compensation tops out.



-         Skill‑based plans may require a larger bureaucracy (related to skills definition and measure­ment, training, and certification).



-         Lastly, there is almost no market information available on how to price skills.



Example: Alliant Health Systems, Inc., a hospital and health‑care provider in Louisville, Kentucky, began testing a skills‑based pay (SBP) system in 1993. Today, only the finance department has retained the system. It was decided that SBP was too complex to design and administer to each work group. Source: "Lessons Learned in Skill‑Based Pay" by J. Morris, HRMagazine, June 1996, pp. 136‑142.





Competing Through Sustainability


Can the German Labor Force Compete?



This case scenario discusses a cost-cutting deal that DaimlerChrysler reached with its German workers in return for job security guarantees.  The deal specifies that some workers will have to work longer hours or take pay cuts.  Other big German industrial firms have been pressing for similar concessions.



VII.      Can the U.S. Labor Force Compete?



A.        The costs for labor are high in the United States, particularly in comparison to newly industrialized and developing countries.



B.         There are several factors to consider in shifting production to other countries:



1.         Relative labor costs are very unstable over time because of fluctuations in currency exchange rates.



2.         The quality and productivity of national labor forces can vary dramatically. Lower labor costs may reflect the lower average skill level of the labor force.



3.         In terms of comparative productivity, unit labor costs, and gross domestic product per person, the United States is the highest in the world.



4.         Nonlabor Considerations—Operating costs may be high enough to compensate for lower labor costs. Also, product development and customer response may be faster when manufacturing is closer to staff groups. The discussion below provides some other consid­erations.



VIII.     Executive Pay



A.        Executive pay has been given widespread attention in the press. However, executive pay accounts for a small proportion of the labor costs of an organization, and executives have a disproportionate ability to influence organizational performance. They also help set the culture, so if their pay seems unrelated to organizational perfor­mance, employees may not understand why their pay should be at risk depending on the organization's performance. Table 11.9 in the text provides data on CEO compensation.



B.         There is much criticism of executive pay, however, because:



1.         Some executives are very highly paid, such as the CEO of Walt Disney over $600 million. However, these figures reflect participation in a stock plan (Table 11.10).



2.         Executives in the United States are the best paid in the world (see Table 11.11 for total remuneration of CEOs in selected countries).



3.         Often, the ratio of executive pay to average worker pay is cited as creating a "trust gap" in which workers do not trust executives' intentions and resent their pay. The issue becomes even more salient when companies are engaging in layoffs but are not cutting execu­tive pay.



IX.       Government Regulation of Employee Compensation



A.        Equal Employment Opportunity (EEO) (Title VII) prohibits discrimination in all employment outcomes, including pay, unless business necessity can be proven. Two trends related to EEO are the increasing participation of women and nonwhites in the labor force.



1.         The proportion of wages that women earn compared to men was 76 percent in 2000, and black to white earnings was 79 percent. There are legitimate explanations such as education, experience, and occupation, although even when these are controlled, differences remain.



2.         Although the rates have risen in recent years, in the case of women, the gap may be related to the fact that "women's" work may be undervalued. Another hypothesis is related to "crowding," which argues that women have been restricted to entering only a few (low‑paid) occupations.



3.         Comparable worth (or pay equity) is a public policy that advo­cates remedies for any under evaluation of women's jobs. (See Table 11.12).



a.         Typically, comparable worth becomes an issue when comparisons between internal (job evaluation) and external (market surveys) data suggest that there is conflict in which jobs predominantly occupied by women are evaluated more highly internally than in terms of the market data.



b.         One problem is that job evaluation is most often used to help apply market‑pay policy and not replace the market. There is also concern that EEO regulations will attempt to replace market forces (although there are no regulations related to comparable worth) and that using only internal comparisons would result in overpayment and underpay­ment in several instances in relationship to the market, which would create a market disadvantage.



c.         Despite potential problems with market shares, the courts have consistently ruled that using the going market rates of pay is an acceptable defense in comparable worth litigation suits.



d.         Another approach has been to suggest that organizations should examine, when women's pay falls behind that of men, entry and access to promotions, and so on. Programs such as mentoring might improve the ability of women to access higher‑level jobs.



B.         Minimum Wage, Overtime, and Prevailing Wage Laws


1.         The Fair Labor Standards Act (FLSA) of 1938 established a minimum wage and overtime pay rate.  



2.         Minimum wage is the lowest amount that employers are legally allowed to pay.  Minimum wage now stands at $5.15 an hour.



3.         Executive, professional, adminis­trative, and outside sales are exempt from FLSA coverage (the estimate is that about 20 percent of jobs fall in this category). Exempt means that these employees are not covered by the FLSA, and they are not eligible for overtime pay.  



4.         The Davis‑Bacon Act (1931) and Walsh‑Healy Public Contracts Act (1936) require federal contractors to pay employees no less than the prevailing wages in the area. The prevailing wage is set by the Secretary of Labor and is greatly influenced by relevant union contracts in the area.



A Look Back


The opening case study showed how one company encountered serious employee relations problems because of the inequity perceptions created by its pay structure decisions, as well as the apparent attempt to keep these decisions from being known by employees.



Questions


1.         What types of changes have the companies discussed in this chapter made to their pay structures to support execution of their business strategies?



For example, IBM decided to delayer and band from 5,000 job titles and 24 salary grades to 1,200 job titles and 10 bands.  This allowed managers to give more discretion to reward high performers and to choose pay levels that were competitive in the market for talent.



2.         Would other companies seeking to better align their pay structures with their business strategies benefit from imitating the changes made at these companies?



This would be a good exercise for students to be broken into groups to develop opinions about imitating other companies.



Chapter Vocabulary



These terms are defined in the "Extended Chapter Outline" section.



Pay Structure

Pay Level

Job Structure

Efficiency Wage Theory

Benchmarking

Rate Ranges

Key Jobs

Nonkey Jobs

Job Evaluation

Compensable Factors

Pay&#8209olicy Line

Pay Grades

Range Spread

Compa‑Ratio

Delayering

Skill‑based Pay

Fair Labor Standards Act (FLSA)

Minimum Wage

Exempt



            

Discussion Questions



1.            You have been asked to evaluate whether your organization's current pay structure makes

sense in view of what competing organizations are paying. How would you determine what organizations to compare your organization to? Why might your organization's pay structure differ from those in competing organizations? What are the potential consequences of having a pay structure that is "out of line" relative to your competitors'?



You would determine what organizations to compare an organization to by looking primarily for organizations in a similar business and located in a similar geographical region. Trade associations or employer associations can frequently help by providing information.



Your organization's pay structure may differ from those in competing organizations if decisions are made to use pay as a primary reward strategy or perhaps emphasize other issues. For example, your organization might offer good training opportunities for new college graduates, and therefore, your pay‑level decision is to pay typically below market.



A consequence of having a pay structure out of line is that if you are overpaying, you will find it increasingly difficult to compete, since your labor costs are too high. If you are underpaying, it will be difficult to attract and retain employees. There are trade‑offs, however, that make a considerable difference in the consequences of over‑ or underpaying in relationship to the market, for example, as mentioned in the text.



2.            Top management has decided that the organization is too bureaucratic and has too many

layers of jobs to compete effectively. You have been asked to suggest innovative alternatives to the traditional "job‑based" approach to employee compensation and to list the advantages and disadvantages of these new approaches.



The most innovative alternative discussed in the text is skill‑based pay. The advantages and disadvantages discussed in the text are as follows:



§         Advantages are that flexibility helps promote lower staffing levels and aids in situations where the manufacturing process demands adaptable, flexible responses (e.g., flexible manufacturing, just‑in-­time systems).



§         Potential disadvantages are that the organization may find it diffi­cult to use all skills effectively (i.e., work design must also change), employees may acquire skills quickly and compensation tops out, and skill‑based plans may require a larger bureaucracy (related to skills definition and measurement, training, and certification). Lastly, there is almost no market information available on how to price skills.



3.            If major changes of the type mentioned in question 2 are to be made, what types of

so‑called process issues need to be considered? Of what relevance is equity theory in helping understand how employees might react to changes in the pay structure?



Communication and participation would both be critical if skill‑based pay was implemented. Communication would undoubtedly be complicated since, as mentioned in the last question, employees must understand what skills are, how they are defined, what has to be learned to be certified as acquiring a new skill, and so forth. Participation is also important since skill‑based pay is logically connected with the need for participation (e.g., technology that is advanced and complex). It also makes sense to ensure participation is built into the process so that skill‑based pay will be cost‑efficient: that is, given the existence of skills, there should be a system that ensures the skills are employed.



4.         Are executive pay levels unreasonable? Why or why not?



There is evidence that may be presented on both sides for this question. The text discusses that pay levels may not be unreasonable, since executives are critical in determining the success of an organization. Data are also presented that suggest, on the average, most executives are not paid that much differently from several other professionals. In addition, only salaries themselves tend to be compared without benefits hectored into the equation. This presents a different picture in comparing executive compensation across countries.



On the other hand, there are some executives who tend to be very highly compensated (often on the basis of stock options). This presents some problems relative to social comparisons of workers. Ben & Jerry's policy of allowing the executives to earn no more than seven times the salary of the lowest paid worker illustrates the social responsibility issue related to executive pay. An interesting question would be whether, with more profit sharing occurring, fewer perceptions of inequity would exist.



5.         Your company plans to build a new manufacturing plant but is undecided where to locate it. What factors would you consider in making a decision about which country (or state) to build the plant in?



For most organizations, the cost of labor is the highest operating expense; therefore labor with the appropriate skills must be available. Also, the area should be assessed to determine prevailing wage and benefit rates. Some other issues to consider would be the availability of an appropriate infrastructure to ensure your raw materials can arrive and that you can ship your product with few problems, costs of building and running a plant, supportiveness of local government, and tax structure. As the example of BMW suggested, the organization must also consider whether customers will perceive that the plant can produce the product at the appropriate levels of quality and quantity.



6.         You have been asked to evaluate whether the pay structure is fair to women and minorities. How would you go about answering this question?



First, you would have to examine whether any policies exist that are unfair to women and minorities. For example, although it is likely that seniority plays a strong role in rewarding employees, this is considered a legitimate factor. If there are, however, separate seniority systems across your organization and women and minorities are located in lower‑paying departments, this is not legal.



Next, you would want to examine possible clusters of minorities and women, what their jobs and skills are, and their average pay in comparison to white males in similar jobs and with similar skills. A statistic, multiple regression, might be useful in determining what predicts pay in the organization. For example, one would expect that experience, education, certification of skills, and other job‑related factors would predict pay while race or sex should not be predictive.





Manager’s Hot Seat Exercise: Negotiation: Thawing the Salary Freeze



I.  Introduction



This scenario depicts a negotiation between labor and management.  For an Organizational Behavior or Management course, this vignette will stimulate a discussion on negotiation skills and, potentially, ethics.  Instructors of HR-related courses will find this vignette useful in discussing the labor-management negotiations process.



II.  Learning Objectives



To assess students’ understanding of the negotiation planning process and effective execution.  
To analyze a negotiation and identify effective and ineffective strategies and behaviors that occurred during the negotiation.
To have students identify key issues related to labor-management negotiations and the legal and managerial implications, therein.


III.  Scenario Description:



Overview:  An upper management executive of a magazine publishing company, JBL Publishing, is negotiating with a Union representative regarding production workers’ contract.  The main issues are salary increases, health benefits, and flexible work schedules.  The Union representative is irate, because she just found out, prior to this meeting, that the executive board has received enormous bonuses for the year, at the same time they are refusing to budge on salary increases for the production workers.



Profile:

·        Katherine Knudsen has a degree in Marketing and Management and a law degree.  At JBL Publishing she is Vice President of Production, managing operations and production.  In addition to overseeing these departments, Katherine handles all contract negotiation and labor relations.

·        Alisa Jackson holds a Bachelor of Arts degree in Sociology.  She has been a union organizer and representative for over eight years, with efforts focused on re-negotiation at the local level.  Alisa has a renegotiation success rate of 95%, winning better terms in one or more categories per contract.  The renegotiation with JBL publishing is her first in the printing industry.  



References:  The references included in the DVD are:

·        Bargaining Zone for Negotiation (PPT 3-3)

·        Concepts in Negotiating (PPT 3-5)

·        Integrative Bargaining Strategy (PPT 3-6)

·        Key Negotiator Behaviors (PPT 3-11)



Back History:  Knudsen and Jackson have been meeting and talking frequently regarding the contract renegotiation for the unionized labor force working in the printing press/production building.  Jackson is a new representative for Local 1087, coming from an airline workers union where there were always very tough negotiation meetings.  Knudsen has been the VP of Production at JBL for 10 years and has negotiated all the contracts over that time.



The renegotiation thus far has addressed the following issues:



·        A 7% wage increase for workers with seniority

·        An improved benefits package across the board

·        More personal days and schedule flexibility to accommodate family needs



Knudsen has said flat out “no” to the wage increase – the company has a salary freeze for all employees – labor and management included.  Knudsen has slightly improved the health package and is considering the schedule issues.



Scene Set-up: Jackson has called an immediate meeting.  She wants to discuss the executive bonuses and win more money for the union employees.



Scene Location: Knudsen’s Office; JBL Publishing; Wednesday 3:00pm



The Meeting - Summary:  Alisa described her surprise that executives were receiving large bonuses when Katherine had said there was no money for salary increases.  She immediately threatens to go the newspapers with this perceived inequity.  Katherine explained that the money allocated for bonuses was based on last year’s performance even though the checks were cut this year.  Alisa comes close to accusing Katherine of not disclosing this information that she feels is pertinent to the current contract.  Katherine explains that it is not pertinent to the current contract because that money was budgeted last year and does not pertain to this year’s cash flow situation.  Alisa then pursues the option of reallocating the bonus money so that the workers may receive a share of the money currently allocated only to upper management.  Katherine agrees to draft a bonus distribution plan and present it to Alisa within the week.  The two shake hands and are gracious to each other at the conclusion of the meeting.



Afterthoughts – Summary:  Katherine admits that the executive bonuses were more relevant to the current negotiation than she let on.  She had held out on mentioning the bonus distribution option because she represents the company and didn’t want to have to do that unless it was absolutely necessary because that was the only concession she had to make.  Alisa’s threat to go to the press to disparage the company concerned Katherine but she did not want give up too much.  Now that they will create a bonus-sharing plan, Katherine says she will work very hard to make that solution work because she recognizes the value of hourly workforce to the revenue of the company and strongly believes in working towards a fair solution that will reward the employees if the company does well.



Dossier:  The specific artifacts included in the DVD are:

1.      Email Summary of Knudsen and Jackson’s previous meeting

2.      Email sent to Jackson re:  the ‘sweet deal’ JBL Execs will be receiving

3.      Excerpts from previous contract





IV.  Discussion Questions:



The References and related Discussion Questions may be found in PowerPoint slides 3-1 to 3-12 on the instructor’s side of the text’s Website.



Learning Objective #1: To assess students’ understanding of the negotiation planning process.  



The “Bargaining Zone for Negotiation” (PPT 3-3) shows where the potential area of agreement may be.  However, without planning and determination of one’s initial offer, target point, and resistance point, a negotiator may be at a disadvantage during the negotiation.  What evidence of planning was demonstrated by Alisa and Katherine?  


See answer for #2



For both Alisa and Katherine, describe how planning impacted their ability to successfully negotiate.


Alisa received an email from a friend saying that he heard that executives at JBL Publishing were receiving large bonuses.  She planned to use this information as leverage in the negotiation process as well as threaten to talk to the newspapers about the perceived inequity.  This strategy did work to Alisa’s advantage because it caused Katherine to concede on the distribution of future management bonuses.  She also insinuated that Katherine was engaged in an unfair labor practice because she did not bargain in good faith when she did not disclose the executive bonuses.  This approach was not that effective because Katherine took it personally and could have ceased or significantly hindered future negotiations.



From the “Afterthoughts” it was clear that Katherine had investigated every option that she would have during the negotiation process.  Because she did not have much, she did everything she could to not give-in to the union’s demands.  Katherine’s plan was to focus on the exact issue the union asked for – salary increases – instead of finding alternatives that would satisfy the union.  Katherine’s plan was successful in that the union representative seemed satisfied with the tentative agreement, did not ask for more, and did not intend to go to the newspaper to create bad press for the company.  



Learning Objective #2: To analyze a negotiation and identify effective and ineffective strategies used and behaviors that occurred during the negotiation.  



What aspects of the distributive and integrative bargaining approaches (see PPT 3-5 & 3-6) did Katherine demonstrate during the negotiation?  Justify your answer with examples from the scenario.


While in the end Katherine supports an integrative approach that is fair to all employees, she initially takes a distributive approach because she wants to give in to the union as little as possible.  She does not mention other alternatives and “hides” behind financial logistics by saying the money that is being paid was from a different pool of money.  An integrative approach is characterized by discussing the various options that would create a win-win outcome from the beginning.



´        1.  What strategy should Katherine take?

A.     Continue to clarify

B.     Apologize for omission

C.     Change subject



Katherine should continue to clarify the situation.  Apologizing for the omission may make her appear guilty of an unfair labor practice by not bargaining in good faith.  If she changes the subject she will also appear to be insincere in her bargaining and it will avoid the issue the union is most concerned about.



´        2.  Why does Katherine offer to show her pay stub?

A.     Proof of salary freeze

B.     Exert power

C.     Change subject



She wants to make it clear and irrefutable that salaries of management have not been raised either.  



´        5. Why didn’t Katherine offer this [bonus distributions] earlier?

A.     Just thought of it

B.     Just persuaded

C.     Was stonewalling



Katherine didn’t want to offer her only point of leverage too soon because she had nothing else to offer.  Therefore, she held out on this solution until she absolutely had to – she was stonewalling.



What aspects of the distributive and integrative bargaining approaches (see PPT 3-5 & 3-6)) did Alisa demonstrate during the negotiation?  Justify your answer with examples from the scenario.


Alisa’s goal is to get more money for the workers she represents.  While initially she tried for salary increases, she was open to other alternatives.  Her approach of threatening the management and accusing them of withholding information, however, was not indicative of an integrative approach because she focused on the person, not the problem.  



´        4.  What should Alisa argue?

A.     To split bonus money

B.     Union will strike

C.     Get better benefits plan



The union could strike on a mandatory issue, which salary and benefits are.  This, however, would emphasize the union’s position, rather than the common interests of both parties.  Offering to split the bonus money would present an alternative solution that would meet the needs of the employees (more money) and prevent a disgruntled workforce.  If Katherine had outright rejected the idea to split the bonus money, threatening to strike may be the only alternative.  Getting a better benefits plan was already in the works.



What is the superordinate goal (see PPT 3-5) in this situation?  How would a discussion of this goal aid the negotiation process?


The superordinate goal is the company’s success.  If the company is not successful, neither management nor the union will have jobs.  Establishing a common goal will aid the negotiation process because the ideas and issues discussed should be linked to the company’s ultimate success.  Therefore, data and information presented during the negotiation as to how salary increases, management bonuses, benefits, etc. will benefit the company directly will have the most impact on the negotiation outcome.



Learning Objective #3:  To have students identify key issues related to labor-management negotiations and the legal and managerial implications, therein.



Katherine discussed a potential solution (bonus sharing) that had not previously been discussed before.  Is this an example of “unlawful circumvention” according to labor laws?  Why or why not?


No, because she was discussing this with the union representative.  If Alisa had been an employee who was not explicitly representing the union, this would have been a case of unlawfully circumventing the union.



Assume Katherine refused to make a concession to Alisa’s request.  According to the National Labor Code, could Alisa organize a strike?


Yes, because the two parties were negotiating a new contract and salary and benefits are “mandatory subjects” that have to be bargained in good faith.



Refer to the “Key Negotiator Behaviors” (PPT 3-11).  Assume you are representing management (like Katherine in this scenario).  What exactly would you do in this situation?  Indicate a specific example for each of the key negotiator behaviors.


Answers will vary.  Students should identify opportunities to use an integrative approach.  



´        1.  Katherine is accused of lying.  She should:

A.     Demand apology

B.     End negotiation

C.     Refute accusation



This is an example of a person, not problem oriented behavior.  Katherine should refute the accusation and indicate that personal attacks will not facilitate the negotiation process.



´        6.  In this negotiation….

A.     JBL gave too much

B.     JBL gave too little

C.     Nothing was resolved



Katherine and Alisa seemed pleased with the outcome of the negotiation.  It seems that both parties were bargaining in good faith and would come up with a mutually beneficial solution.  



Exercising Strategy:  Changing Compensation to Support Changes in Corporate Strategy



Questions



1.         What are the pros and cons of Corning’s new pay structure?



            Student answers will vary.  

Con: Divestiture reduced annual revenues.  

Pros:  Changing job structure helps the company develop products quicker and more efficiently.  Decentralization contributes to flexibility in the organization.  Tying pay to performance motivates employees.



2.         How did shifting product market conditions affect Corning’s restructuring and its success?



Student answers may vary.  These shifting environmental conditions meant that Corning had to increase its ability to move quickly in responding to customer needs.  The company was able to accomplish this by encouraging flexibility, learning, and teamwork.



Managing People: From the Pages of BusinessWeek



Revenge of the Overworked Nerds



Questions



1.         Do you think the “cachet” of being a manager makes up for longer hours at the same pay?



Student responses will vary, but one way of looking at the situation may be that the “cachet” is does not make up for the overtime without pay because employees should be compensated for the work that they do.

  

2.         Even though the law permits it, are there any potential drawbacks for an organization that works its managers long hours without paying more?  Are there consequences for perceived equity or motivation?



Students’ answers will vary, but may include lawsuits, more turnover, and decrease in work performance as drawbacks to overworking employees.  



There are consequences for perceived equity and motivation. For example, since some employees are happy working overtime without the extra pay because they feel their exempt status gives them a certain cachet, companies assume all employees fell that way, when in fact, the title of manager is not enough for others.



3.         Do you think the government should allocate more or fewer resources to enforcing the Fair Labor Standards Act?  Why?



Students’ opinions will vary, but they may suggest that the government allocate more resources to enforcing the Fair Labor Standards Act because the act was written in the industrial age, not the information age, and needs to be updated.  Since work has changed, the effects and types of work need to be taken into consideration when enforcing the act.



Additional Activities



Teaching Suggestions



1.         One issue that may generate considerable heat and controversy is discussing in class whether internal job evaluation results or external market‑pay surveys should be used when there is a conflict between internal and external data. It must be recognized that the use of comparable worth is only an issue when there is conflict between the internal job evaluation and external market‑survey data. The choices are then to pay less and follow the market data or use internal information to increase the salary grade or pay of the job in question.



A debate could be organized to discuss this issue. The promarket side would tend to focus on issues of competitiveness, external equity, and paying appropriately for human resources. The prointernal side would discuss issues of internal equity, employee perceptions of equity and motivation, and comparable worth. Students find the concept of comparable worth highly controversial.

A library assignment to do additional readings and report to the class on the topic would likely make the debate more lively and controversial. For example, reporting on the Canadian province of Ontario's comparable‑worth regulations and how they have worked would be informative.



2.         In 1990 a study was done of 313 firms, which followed a study done in 1987 of 323 firms, about the use of skill‑based pay systems. In 1987, 40 percent of the firms used skill‑based pay, while in 1990 the percentage was 51 percent. The percentage of employees covered remained approximately the same less than 20 percent in both 1987 and 1990. In 1990, 60 percent of the firms stated that skill‑based pay was successful in increasing organizational performance, while only 6 percent rated it as unsuccessful and 35 percent were undecided. The study supported the hypothesis that there would be more skill‑based pay associated with companies that used total quality management systems, experienced heavy foreign competition, and removed management layers in recent years. Lastly, the use of skill‑based pay was associated with the use of a variety of other reward‑system practices (Adapted from: E. E. Lawler, G. E. Ledford, Jr., and L. Chang, "Who Uses Skill‑based Pay and Why," Compensation and Benefits Review 25 no. 2, [1993], pp. 22‑26.)



Questions for discussion on skill‑based pay: Why do you believe that skill‑based pay was found more often in organizations that experienced a great deal of foreign competition? In organizations that had removed management levels in recent years? In organizations that also used a variety of other reward strategies?

Do you believe that the use of skill‑based pay will continue to increase? Why or why not?



3.         One interesting assignment, if you teach in a school where salaries are public information, is to have students look at the differences in salaries for different disciplines within the university and discuss the fairness and rationale for these differences. My students are often interested in looking at the differences between professors in the College of Business and professors in the Arts and Sciences. Also, we discuss the differences in the salaries for professors in the college and the market salaries. In my institutions, the professors' salaries are significantly below the market, and we discuss the issue of wage compression, why this is a problem, and what alternatives organizations have to deal with this issue.



4.         Are CEOs paid too much? This is an issue that students could debate. One article that does a good job of covering both sides is "Contrasting Perspectives—The Growing Pay Gap: Are CEOs Paid Too Much Relative to Other Employees" by G. Crystal and F. Cook, American Compensation Journal, Summer 1996, pp. 22‑29. Another very informative article on the subject is "Executive Pay Trends: Where Have We Been . . . Where Are We Going" by P. Meyer, American Compensation Association News, June 1996, pp. 16‑18.

该用户从未签到

 楼主| 发表于 2010-10-3 20:12:49 | 显示全部楼层
Chapter Summary



This chapter focuses on the design and administration of programs to reward individuals for their contribution to organizational success. The design often amounts to combinations of individual, group, and organiza­tional incentives. Rewards must also be designed for the particular organization and the needs and motives of its employees. Benchmarking (examining other organizations' practices) may be useful, but these practices must be implemented while considering the unique needs of the organization. Since pay is a powerful motivator (although certainly not the only one), pay systems' design is critical to organization success. Process— that is, communicating and administering the process fairly ensures that employees will perceive the pay system as equitable.



Learning Objectives



After studying this chapter, the student should be able to:



1.         Describe the fundamental pay programs for recognizing employees' contributions to the organization's success.

2.         List the advantages and disadvantages of the pay programs.

3.         List the major factors to consider in matching the pay strategy to the organization's strategy.

4.         Explain the importance of process issues such as communication in compensation management.

5.         Describe how U.S. pay practices compare with those of other countries.



Extended Chapter Outline



Note: Key words appear in boldface and are listed in the "Chapter Vocabulary" section.



Opening Vignette:  Microsoft Ushers Out Golden Era of Options



In July 2003 Microsoft announced it would stop issuing options and instead begin giving its 50,000 employees restricted stock.  The company’s decision comes amid mounting pressures on stock options, long a pay perk for senior executives.  For Microsoft, the move to abolish options signals that the once-fast-growing tech business is becoming a mature industry.





I.                    Introduction—Organizations have a relatively large degree of discretion in deciding how to pay.  Differences in performance (by an individual, group, or the organization), seniority, or skills determine the pay. Regardless of cost differences , different pay programs can have very different consequences for productivity and return on investment.



II.                 How Does Pay Influence Individual Employees? Besides the equity theory, described in the previous chapter, there are other theories that influence compensation's effects.



A.        Reinforcement Theory—In Thorndike's Law of Effect, a response followed by a reward is more likely to recur in the future. The importance of a person's actual experience in receiving the reward is critical. If high performance is followed by a reward, high performance is likely to be repeated.



B.         Expectancy Theory—This theory says that motivation is a function of valence, instrumentality, and expectancy.



C.        Agency Theory—This theory focuses on divergent interests and goals of the organization's stakeholders and the ways that compensation can be used to align these interests and goals. Today, most stockholders are removed from the day‑to‑day operations of the organization. This separation has many advantages, but it also creates costs—the interests of the princi­pals (owners) and their agents (managers) may not converge. Agency costs are as follows:



1.         Management may be spending money on perquisites (e.g., executive perks) or empire building (enhancing the image or power of the executive without adding value to the business).  



2.         Managers are more averse to risk since they cannot diver­sify their holdings as shareholders can.



3.         Decision‑making horizons may differ, since managers will likely emphasize short‑term gains to ensure promotion and visibility, perhaps at the cost of long‑term success.





D.        Agency costs may be minimized by the principal choosing a contracting scheme that helps align the interests of the agent with the interests of the principals. These approaches can be behavior oriented (e.g., merit pay) or outcome oriented (e.g., stock options, profit sharing, commissions). Outcome­-oriented approaches link the rewards of the organization and individual. However, agents are often risk‑aversive and may demand a compensating wage differential. Behavior­-oriented contracts do not transfer risk and therefore do not require a compensating wage differential. Deciding what to use is based on the following:



1.         Risk aversion among agents makes outcome‑oriented contracts less likely.



2.         Outcome Uncertainty—If risks are high for desired outcomes, such as profits, agents will be less willing to have outcome‑oriented contracts.



3.         Job Programmability—As jobs become less program­mable (i.e., less routine) outcome‑oriented contracts are more likely.



4.         Measurable Job Outcomes—When outcomes are more measurable, outcome‑oriented contracts are more likely.



5.         Ability to Pay—Outcome‑oriented contracts contribute to higher compensation costs because of the risk premium.



6.         Tradition—A tradition of using (or not using) a certain type of contract will make it more likely (or less likely) that it will continue.



III.               How does pay influence Labor Force Composition? — There is increasing recognition that individual pay programs may also affect the nature and composition of an organization’s workforce.  Different pay systems appear to attract people with different personality traits and values.  Organizations that link pay to individual performance may be more likely to attract individualistic employees, whereas organizations relying more heavily on team rewards are more likely to attract team-oriented employees.





IV.              Programs—Table 12.1 in the text provides an overview of some programs and potential consequences. Programs differ by payment method, frequency of payout, and ways of measuring performance. Potential consequences of such programs are performance motivation of employees, attraction of employees, organization culture, and costs.  Contingencies that may influ­ence whether a pay program fits the situation are management style, and type of work.



A.           Merit pay programs link performance‑appraisal ratings to annual pay

increases.



1.         The size and frequency of pay increases are most often deter­mined by performance rating (since better‑performing employees should be rewarded more than low performers) and position in range (compa‑ratio). Compa‑ratio is used to control compensation costs and maintain the integrity of the pay structure. Table 12.2 demonstrates an example of a merit increase grid, which combines an employees performance rating with the employee’s position in a pay range to determine the size and frequency of his or her pay increases. Table 12.3 indicates how compa‑ratio targets and performance ratings might be combined.



2.         Another factor that requires close attention is the distribution of performance ratings.  Some organizations provide guidelines regarding the percentage of employees who should fall into each performance category.



3.         Deming, who is a critic of merit pay, argues that it is unfair to rate individual performance because "apparent differences between people arise almost entirely from the system that they work in, not the people themselves." Examples are co‑workers, the job, materials, customers, management, and supervision. These factors are the responsibility of management.





a.         Deming also argues that the focus on merit pay discourages teamwork.



Example: U.S. Healthcare, Inc., a health maintenance orga­nization (HMO), links doctors' pay to patient satisfaction. The main concern with this program is that doctors may work together less as colleagues and be more individually competitive.



b.         Deming suggests that the link between individual performance and pay should be eliminated. The consequence of this might be that high performers would leave the organization. An appropriate balance between group and individual incentives should be designed.



c.        Another criticism is the way performance is measured. If this is not done fairly and accurately, employees will perceive the whole process to be unfair. Some of the most important aspects of justice that employees assess are distributive (based on how much they receive) and procedural (what process was used to decide how much). Table 12.4 lists questions involved in procedural justice and pay decisions.



d.         A last criticism is that merit pay may not really exist. In the late 1980s and early 1990s, merit budgets were approximately 4 to 5 percent. The difference between a high performer's pay increase (e.g., 6 percent) and a moderate performer's (3 to 4 percent) might mean little dollar difference in the paycheck. In fact, many employees do not believe that there is any payoff to higher levels of performance (Figure 12.1). Communication should be used to indicate that small differences can equate to large differences over time.



B.                 Individual incentives reward individual performance, but payments are not rolled into base pay, and performance is usually measured as physical output rather than by subjective ratings. Monetary incentives increased production by 30 percent in a study by Locke, et al. Individual incentives are, however, relatively rare because:





1.         Most jobs have no physical output measure.

            

2.        There are many potential administrative problems (e.g., setting and maintaining acceptable standards).



3.         Individual incentives may do such a good job of motivating employees that they do whatever they get paid for and nothing else.



4.         Individual incentives typically do not fit in with the team approach.



Example: At Lantech, a small manufacturing company, the incentive pay system ignited "gang warfare" in which individuals and departments tried to make others look bad to make their own numbers look good. They finally abandoned individual and division performance pay, and relied instead on a profit‑sharing plan in which all employees get bonuses based on salary. Source: "Incentive Pay Can Be Crippling" by P. Nulty, Fortune, November 13, 1995.



5.         They may be inconsistent with goals of acquiring multiple skills and proactive problem solving.



6.         Some incentive plans reward output at the expense of quality.



Competing Through Sustainability



BMW’s Labor Practices are Cutting-Edge, Too



At BMW, when demand shifts, the company adjusts thanks to its flexible productive factories.  At times of high demand, the company increases the work hours per week at factories.  Employees bank the extra hours in an account.  When demand is low, instead of laying off employees, the company reduces work hours per week.  Employees can withdraw from the account and take days off.  This process also works well with BMW built to order vehicles.   





C.                 Profit Sharing and Ownership



1.                  Under profit sharing, payments are based on a measure of organization performance (profits), and payments do not become a part of base pay.



a.         An advantage is that profit sharing may encourage employees to think more like owners and take a broad view of what needs to be done, labor costs are reduced in difficult economic times, and organizations may not have to rely on layoffs. There is a downside risk in that there may be no profits in a given period.



b.         A second advantage is that because payments do not become part of base pay, labor costs are automatically reduced during difficult economic times, and wealth is shared during good times.  



c.         The drawback is that workers may perceive their performance has little to do with profit but is more related to top management decisions over which they have little control. Therefore, most employees are unlikely to see a strong connection between what they do and what they earn under profit sharing (instrumentality perceptions are not likely to be reinforced). They may also be disturbed when plans do not pay out when no profit is made.



d.         Another motivational problem is that most plans are deferred (paid to employees in some future time period). Dupont eliminated a profit‑sharing plan when employees in some divisions received less than those in comparable divisions and returned to a system of fixed base salaries with no variable (risk) component. An alternate solution is to have plans that contain upside, but not downside, risk (i.e., base salaries are not reduced when a plan is implemented). When profits are high, employees share, but when profits are low, they are not penalized. This, of course, eliminates the advantage of controlling labor costs in difficult economic times.





e.         In summary, profit sharing may be useful to enhance identification with broad organizational goals, but it may need a supplement of incentives that are individually oriented.



2.         Ownership also encourages employees to focus on the success of the organization as a whole, but, like profit sharing, may not result in motivation for high individual performance. Employees may not realize gain until they sell their stock, often when they are leaving. Reinforcement theory suggests, since rewards may not be directly experienced, that motivation would not be greatly increased.



a.                   One method to achieve employee ownership is through stock options, which give employees the opportunity to buy company stock at a fixed price. For example, if a share is offered to employees at $10 a share and a few years later the value is $30, an employee can "exercise his or her options" and sell the stock at a profit (if stock goes down, there is no financial gain). Stock options are typically reserved for executives, although more companies have offered options to all employees, such as Pepsi‑Cola and Hewlett&#8209ackard. Some studies suggest that higher organi­zation performance occurs when executives are eligible for long‑term incentives, although results are not clear relative to lower‑level employees.



3.         Employee stock ownership plans (ESOPs) are employee ownership plans that give employers certain tax and financial advantages when stock is granted to employees. The number of employees in such plans increased from 4 million in 1980 to over 10 million in 1999.



a.         On the negative side, ESOPs can carry significant risk for employees. ESOPs must, by law, invest at least 51 percent of their assets in the company's stock. This results in less diversification and more risk, so if the company does not do well and pensions are funded by an ESOP, employees risk significant loss.





b.         ESOPs can be attractive to organizations since they have tax and financing advantages and can serve as a takeover defense, assuming that employee owners are "friendly" to management.



c.         Some degree of participation is mandatory on certain decisions, but there appears to be a great deal of range in employee decision making between plans. Some studies suggest that more participation will lead to more perceived ownership in the company.



D.                 Gainsharing, Group Incentives, and Team Awards



1.         Gainsharing programs are based on group or plant performance that does not become part of the employee’s base salary.  They offer a means for sharing productivity gains with employees. They differ from profit sharing in that instead of using an organization‑level performance measure (profits), they measure group or plant performance. This is likely to be seen as more controllable by employees, and it is therefore more motivating. Also more motivating is that payouts are distributed more frequently and are not deferred. Studies indicate positive results on performance.



2.         Table 12.5 provides an example of one kind of gainsharing, the Scanlon plan. This plan provides a monetary bonus to employees (and to the organization) if the ratio of labor costs to the sales value of production is kept below a certain standard. The example indicates that the standard is $240,000 (20 percent of $1.2 million). Because actual labor costs were $210,000, there is a savings of $30,000. The organization receives half and employees receive the other half.



3.         Table 12.6 indicates that there is often a strong emphasis on taking advantage of employee know-how to improve the production process through teams and suggestion systems.  





4.         Conditions that should be in place for gainsharing to be effective include:

a.   management commitment

b.   the need to change or a process of continuous improvement

c.   management's acceptance and encouragement of employee input

d.   high levels of cooperation and interaction

e.   employment security

f.    information sharing on productivity and costs

g.   goal setting

h.   commitment of all involved parties to change and improvement, agreement on a performance standard

i.    agreement on a performance standard and calculation that is understandable, perceived as fair, and closely related to managerial objectives.



Example: Panhandle Eastern Corp.'s gainsharing plan calls for all employees to receive a bonus of 2 percent of their salary at year‑end if the company earns at least $2 a share. If Panhandle earns $2.10 or more a share, the bonus climbs to 3 percent. The company says the plan has turned employees into cost‑cutting vigilantes. Source: "Gas Company's Gain‑Sharing Plan Turns Employees into Cost‑Cutting Vigilantes" by E. Nelson, The Wall Street Journal, September 29, 1995, p. B1.



4.         Group incentives tend to measure performace in terms of physical output, whereas team award plans may use a broader range of performance measures.  Drawbacks are that individual competition may be replaced by competition between teams. In addition, dimensions must be perceived as fair by employees and include all important factors, such as quality.



E.                  Balanced Scorecard—Some companies design plans that combine various elements of the above programs that are appropriate to the situation. If only merit pay/individual incentives are used, there may be high motivation but too much individualistic competition and a lack of focus on organizational goals. If only profit sharing and gainsharing are used, on the other hand, there may be more cooperation and attention to organizational goals, but individual work motivation might be low. (See Table 12.7 in text for an example of a Balanced Scorecard incentive program).





Example: Herman Miller is a furniture manufacturer that has been using a pay plan since 1950 that combines gainsharing and profit sharing. Quarterly bonuses are paid based on company profits, market share, and efficiency in using noncash assets. Quality is heavily emphasized and customer surveys are regularly used. Suggestions are rewarded through the quarterly bonus so that idea hoarding and individual competition will not occur. Monthly, videos are distributed that explain the calculation of the bonus and the rationale and that show executives discussing operational and marketing strategies.



V.                 Managerial and Executive Pay—Because of their significant ability to influence organization performance, top managers and executives are a strategically important group whose compensation warrants special attention.  One concern appears to be that in some companies rewards for executives are high regardless of organizational performance.



A.        Executive pay can be linked to organizational performance (from agency theory) by making some portion of executive pay contingent on company profitability or stock performance. This means less emphasis on non-contingent pay (e.g., base salary) and more emphasis on outcome‑oriented contracts, both short‑term and long-term. Each year, Business Week publishes a list of top executives who did the most for their pay and those who did the least.  



B.         Organizations vary a great deal in the extent to which they use both short‑term and long‑term incentive programs. Greater use among top-and middle‑level managers was associated with higher levels of profitability (Table 12.8).



C.        There has been increased attention to executive pay from regulators. The Securities and Exchange Commission (SEC) recently required companies to more clearly report executive compensation levels and the company's performance relative to that of competi­tors over a five‑year period. The Omnibus Budget Reconciliation Act of 1993 eliminated the deductibility of executive pay that exceeds $1 million. Table 12.9 shows how the balanced scorecard can be used to balance shareholder, customer, and employee satisfaction.





Example: Because of economic problems, several of Japan's leading corporations have announced pay cuts for their executive groups. For example, at Fujitsu, Ltd., some executives' compensation has declined by 35 percent. While some U.S. companies are doing the same, there are still significant differences in executive pay. For example, the highest paid Japanese executive, Hiroshi Yamauchi of Nintendo, received $6.3 million in 1991. Thomas F. Frist of Hospital Corp. of America received $127 million (with stock options). For 1991, Japanese CEOs averaged $872,646, which is about one-fourth of the average pay of U.S. CEOs. Japanese CEOs earn 32 times the pay of the average factory worker, while American CEOs earn 157 times the average factory worker (although bonuses for workers are not included and this can boost salaries by a third). Perks may be significant in Japan, but consultants say these fail to match the millions more that American CEOs are paid, and the perks may actually be fairly similar in the United States. The lower pay of the Japanese may be attributed to the perception that business results are a joint effort and rewards should be shared. On the other hand, American CEOs may carry 90 percent of the responsibility for making decisions attached to the office. In Japan that is rarely the case, since so much of the burden is distributed among subordinates. (Adapted from: "Executive Pay: The Party Ain't over Yet," Business Week, April 26, 1993, pp. 60‑79.)



VI.              Process and Context Issues—These issues represent areas of significant company discretion and pose opportunities to compete effectively.



A.        Employee participation in decision making relative to design and implementation of pay policies has been linked to higher pay satisfaction and job satisfaction. Agency theory suggests that employees may set goals in their own favor; however, monitoring would be less costly and more effective since employees know their workplace best.



Example: C&S Wholesale Groceries, Inc. empowered its employees to design an incentive plan. Employees from various departments have designed incentives for their particular departments. Teams design the plan and set their own standards with input from human resources and finance. About 75 percent of the 1,000 employees are covered by an incentive plan. Standards are perceived as flexible since business conditions and goals are always changing. Product damage has been reduced by 50 percent, and the company's productivity is twice the industry average.





B.         Communication is critical since any change will engender anxiety and rumors. Videotapes from officers are used frequently, as are brochures and focus group sessions where small groups of employees are interviewed about communication (and the programs).



C.         Pay and Process: Intertwined Effects—It is suggested that changing the way workers are treated may boost productivity more than the way they are paid; that is, using participation with a particular pay plan may increase nonpay rewards as well as productivity and profitability.



Competing Through Technology



Technology Alters How Performance Performance is Gauged—and Rewarded



During the boom, in many companies, all performances were considered strong and all cube dwellers were deemed above average.  Now, in the New Economy’s first recession, harsh reviews pummel grades, and batter bonuses.  More and more companies are turning to Web-enabled employee performance software.  However, critics say these technological developments can’t take on an Orwellian cast, as some companies will soon be able to blend in the monitoring information with the new goal data.  





VII.            Organization Strategy and Compensation Strategy: A Question of Fit— One should consider how the pay strategy will enhance organizational strategy. Table 12.11 suggests some matches of strategies; for example, a prospector organization's (see Chapter 2) emphasis on innovation, risk taking, and growth is linked to a pay strategy that shares risk with employees but also provides them with the opportunity for high future earnings by having them share in whatever success the organization has. Fixed compensation is low, but the use of bonuses and stock options can pay extremely well. Defender organizations will use a very different pay plan.





A Look Back



The chapter beginning examines how incentive plans can have both intended and unintended consequences.  The information contained within this chapter has explored the potential advantages and disadvantages associated with the various types of incentive or pay for performance plans.



Questions



1.         Does money motivate?  Use the theories and examples discussed in this chapter to address this question.



According to the Reiforcement Theory, money is a motivator.  It states that “a response followed by a reward is more likely to recur in the future.”  If employees know they will be rewarded for high performance, they will continue to perform well.  If the rewards stop, the performance will drop.



The Expecancy Theory states that monetary rewards increase motivation and performance, but the increased motivation will be extrinsic, and intrinsic motivation will decrease.



Monetary rewards are a motivator according to the Agency Theory, but is focused on the risk-reward trade-off.



2.         Think of a job that you have held.  Design an incentive plan.  What would be the potential advantages and disadvantages of your plan?  If your money was invested in the company, would you adopt the plan?



Students’ answers will vary.





Chapter Vocabulary



These terms are defined in the "Extended Chapter Outline" section.



Expectancy Theory

Principal

Agents

Merit Increase Grid

Profit Sharing

Stock Options

Employee Stock Ownership Plans (ESOPs)

Gainsharing



Discussion Questions



1.            To compete more effectively, your organization is considering a profit‑sharing plan to

increase employee effort and to encourage employees to think like owners. What are the potential advantages and disadvantages of such a plan? Would the profit‑sharing plan have the same impact on all types of employees? Is the size of your organization an important consideration?  Why?  What alternative pay programs should be considered?



The advantages are that employees will be more inclined to think like owners and will probably broaden their view about job duties and what needs to be done. Labor costs will decline in poor economic times, and layoffs may not be necessary. Disadvantages are that employees may not believe that they have much power to control outcomes and will be disenchanted when there is no profit and therefore no profit sharing.



The plan would not have the same impact on all employees. Of particular concern would be high performers with high‑achievement motivation. They may find the plan frustrating, since they are not rewarded for individual effort.



Size of an organization should not be an important consideration because profit-sharing is based more on productivity and profits of the company, not how the profits will be split up (e.g. the more employees there are, the less money each employee gets).



Alternatives might be programs such as stock options or ESOPs. The company might add some component of individual and/or group incentives such as gainsharing.





2.            Gainsharing plans have often been used in manufacturing settings, but can also be applied

in service organizations. Discuss how performance standards could be developed for gainsharing plans in hospitals, banks, insurance companies, and so forth?



Performance standards should be developed with the help and teamwork of managers and employees. These standards must be perceived as fair, reasonable, and equitable. Costs are possible to measure in some departments, but other departments may need to rely on measures that are oriented to process, including components of quantity, quality, and timeliness of service. In service organizations, customer satisfaction with service would be extremely important.



3.            Your organization has two business units. One unit is a long­ established manufacturer of

a product that competes on price and has not been subject to many technological innovations. The other business unit is just being started. It has no products yet, but it is working on developing a new technology for testing the effects of drugs on people via simulation instead of through lengthy clinical trials. Would you recommend that the two business units have the same pay programs for recognizing individual contributions? Why?



No, the plan should not be the same, since the business and risk are very different. Incentives for the new organization might include stock options, since salaries and benefits may need to be low in start‑up. In other words, employees, to be attracted to join, may be willing to take risk for the possibility of a large reward. There may also be a plan in place PTO reward individual research efforts that result in large profits, patents, and so on, by a share of the proceeds (this is labeled  “intrapreneurship”).



4.         The opening vignette describes a change in pay strategy at Microsoft and the Finding and Keeping Employees section also discusses this change as well as similar changes at Dell and Amazon.  Do you believe the changes at these companies make sense?  What are the potential payoffs and pitfalls of their new pay strategies?



            Student answers will vary.







Exercising Strategy: Paying for Good Employee Relations



Questions



1.         Should companies worry about employee attitudes?  Why or why not?



Student answers will vary.  Most would suggest that yes companies should worry about employee attitudes, because this affects worker productivity.  



2.         If positive employee attitudes are an objective, should organizations directly link pay incentives with attitudes?



            Student answers will vary.  Linking pay incentives with attitudes may discourage recruitment of people with problematic dispositions.  However, a positive attitude does not mean that a worker is unproductive.  



Managing People: From the Pages of BusinessWeek



The CEO’s Gravy Train May Be Drying Up



Questions



1.         Have CEO’s been overpaid?  Why or why not?



Student answers will vary.  Some would suggest that CEOs should be paid highly because they control organizational strategies that ultimately lead to success or failure.  Others may argue that high CEO pay undermines the efforts of lower-tiered employees.  



2.         How can boards of directors and their governance procedures be structured so as to better serve shareholders?  Should they serve other stakeholders as well?  



            Student aswers may vary.  Boards of directors exist to serve shareholders.  On many boards, company shareholders are members of the board.  They have the ability to influence decisions that affect shareholder wealth.  The Board of directors should also serve employees and customers.  The goal is to provide fair and equitable pay and benefits for employees, but still keep costs low for customers.  This is difficult to accomplish for companies that want to maximize shareholder wealth.  





3.         What are your predictions for the future regarding the level and appropriateness of CEO pay?  



            Student answers will vary.





Additional Activities

Teaching Suggestions



Students will typically be quite interested in this chapter, since they tend to see significant personal career implications. Beyond the discussion, research, and case suggestions below, an instructor might use a problem solving approach by groups to discover what students would like to learn about and design projects that evolve from such discussions.



1.         Risk aversion was one factor discussed regarding agency theory. You might introduce a discussion with students about individual perceptions regarding risk taking. They are likely aware of the high failure rate of small businesses. What do they personally consider "acceptable risk"? What factors would their personal perception of acceptable risk depend on (e.g., spouse, children, etc.)? What type of employee do they believe a high‑risk possibility for a high‑profit situation would attract, and what are the motivational implications?



2.         One student project would be for students to benchmark human resource strategies through rewards. Student groups may each interview a manager responsible for compensation to describe nontraditional rewards (those different from merit plans or general increases). Groups could report back to the class the "benchmark" strategy they discovered, as well as any evidence that may be available on its success (this will likely provide an opportunity to discuss the difference between anecdotal and empirical evidence).





3.         A resource is the Harvard Business School case on Merck and Co., Inc. (9‑491‑005 and teaching note 5‑491‑008) by K. J. Murphy. Merck and Co., Inc., a major pharmaceutical company, is in the process of reviewing and evaluating its personnel policies and practices. Employee interviews revealed that rewards for excellent performance were not adequate: outstanding performers received salary increases that were, in many cases, only marginally better than those given to average performers. In many cases, outstanding performance was not even clearly identified. The objective is to have students wrestle with a common malady of performance‑appraisal systems: the tendency of managers to assign uniform ratings to employees regardless of performance. Alternative appraisal systems should be suggested and discussed.



Another resource is the case of Merck and Co., Inc. B (9‑491‑006) by K. J. Murphy, teaching note (5‑491‑008) and supplement (9‑491‑007). In late 1986, Merck revised its performance review and pay practices. The most important change was a shift from an absolute rating system to a forced‑distribution system in which managers were forced to adhere to a given distribution of performance ratings. Other major changes included revised rating categories, revised performance categories, and a shift in timing of performance evaluations. A discretionary award program was also introduced. The objective is to have students discuss the costs and benefits of the revised performance plan, paying particular attention to the relative performance‑evaluation aspects of the new plan. Is it better than the plan it replaced? Is pay more closely related to performance under the new plan?



Discussion Questions



1.        Should the performance appraisal and salary administration system have been revised? Why or why not?



2.        Consider your student group to be the Employee Relations Review Committee. What changes in the performance appraisal and salary administration system would you recommend? How should changes be implemented? Carefully consider the consequences of your recommendations.



3.        What did you learn about managing human resources from reading and analyzing this case?





4.         Case: Direct Response Group Restructures. Direct Response Group (DRG) was a direct response insurance company with work structured for the mass market. DRG competed with insurance and financial services companies such as Prudential and Allstate. DRG offered a full line of life and health insurance and property and casualty insurance to individuals via mail, phone, television commercials, newspaper inserts, and other response methods. Between 85 percent and 150 percent of a product's first‑year premium was spent in marketing costs before a single policy was sold. DRG had identified and developed several target markets, including veterans, credit card holders, credit union members, and senior citizens. For the past several years, margins had declined due to underwriting risks and higher costs to acquire new customers. Sales were no longer enough to offset lapsed policies, let alone grow the business. As a result, DRG felt that they had to redefine how they approached their customers from both a sales and service perspective.



Currently, customer interactions suffered from the curse of departmentalization. The business was handed off one stage at a time until someone got back to the customer. Since there was a lack of responsibility for any individual customer, interactions with customers were seldom used to find out more about the customer and to probe for unmet needs. DRG has started to implement a customer management team (CMT) concept. The CMT is viewed as the centerpiece of the customer‑driven strategy—a component that would enable DRG to move from its product‑driven, mass marketing strategy, to a customer driven strategy of "caring, listening, satisfying, one by one."



The first CMT provides sales and service to a group of 40,000 customers from the veterans' business in 16 states where DGR marketed life, health, and property and casualty products. The CMT sells customers new products, provides some services directly, such as policy changes, and acts as an interface and advocate while other departments provide services, such as claims and underwriting. CMT members were selected and trained specifically for this program. Employees are paid market wage rates. They can receive a bonus for "up selling" customers (getting them to take on additional coverage or purchase new products). The team consists of 10 managers who had worked in the telemarketing area and were licensed to sell insurance products. In addition to the 10 agents, the team includes one member from the marketing, operations, and systems areas. Teams are expected to be self‑managing. They control goal setting, allocating work assignments, and scheduling.





You are overseeing the new CMT. You notice that employees tend to resist sharing information with one another that could be helpful in serving customers. Some employees are being too enthusiastic in trying to sell new products to customers who are not interested. From a compensation perspective, what do you think needs to be done to make the CMT work?



Suggested Approach: Students should focus both on individual incentives and group incentives (for example, directed to the level of the self‑managed team). Sales should be rewarded, of course, but there clearly must be a mechanism to trace and reward service to the customer.



5.         Case: Wells Fargo Employee Recognition Program. Wells Fargo Bank had a year of record‑breaking profits in 1988. Throughout the company, bottom‑line oriented managers and their staffs were steeped in the importance of "return to the shareholder." A renewed focus on the customer was evident in ambitious new customer service standards. Extra effort and constant change became the norm. A decision was made to implement an all‑employee reward program for the final quarter of 1988. The objectives of this program included the following:



·        To recognize contributions made by employees as a group.

·        To recognize individuals who had made exceptional contributions.

·        To reinforce the qualities most valued in Wells Fargo employees.

·        To have fun.

·        To retain an awareness and an appreciation for the program over an extended period of time.



Wells Fargo developed a program with several elements phased in over about eight months. The program involved three phases: all ­employees cash awards, peer recognition through cash bonuses, and corporate recognition of those receiving the most peer recognition. The theme of the program, "In Good Company," was chosen because it was upbeat and positive and recognized the importance of the team effort as well as the effort of individuals.





To recognize all employees as a group, the program design included a $550 pretax cash award to be given to every salaried employee (16,000 employees) below the senior vice‑president level with at least one year of service. An award of $50 (pretax) was given to 3,000 hourly employees with a year of service. The cash awards were announced and given to employees by managers in staff meetings. Managers had no prior knowledge of the event. Each employee also received the first issue of the "In Good Company" newsletter with the check. The newsletter explained the $500 and announced the peer recognition phase of the program. Most of the newsletter focused on the qualities most valued in a Wells Fargo employee. To answer employee questions about the awards, an "In Good Company" hotline took calls throughout the duration of the program.



The peer recognition part of the program involved giving a coupon worth $35 to each employee with a year of service. With the coupon were instructions for awarding $35 to a co‑worker. The rules provided were that an employee could not award the coupon to themselves, no coupons could be given to employees who were senior vice‑presidents or above, and coupons could not be awarded to contract workers or temporaries. Any other employee could be given a coupon, even those who did not have a year of service and had not qualified for any other part of the program. On each coupon, employees were asked to mark the valued quality that the awardee had demonstrated (e.g., "this coupon is a way of saying thanks for inspiring me to excel"). Employees were given three weeks from the date of distribution to return the coupons to payroll for payment. Coupons received after that date were paid, but were not counted for the corporate recognition part of the program. Coupon recipients were tracked by Social Security number to calculate the number of coupons each received and to determine qualifications for corporate recognition.



Employees who received the most coupons were singled out for additional recognition. Most of those who qualified were from the administrative/clerical ranks of the company. Employees who received the most coupons qualified to select a gift from "101 Awards." The awards list was constructed to appeal to a wide variety of tastes, lifestyles, and priorities. Examples of the awards included a week off with pay, payment of an individual's mortgage for a month, grooming for a pet for a year, and four movie tickets a month for a year. The "101 Award" winners were recognized at a cocktail party and dinner hosted by the CEO, president, vice‑chairman, and group heads in each winner's reporting structure.





Questions



1.         What are the strengths and weaknesses of the Wells Fargo recognition program? What improvements would you suggest?



2.         What part of the program has the strongest link to employee motivation? Why?



3.         Assume Wells Fargo has now experienced a year of losses. Cash is not available for the recognition program. How could the program be modified or changed to continue to meet the program objectives?

(The example on ARPs may be very helpful for students in addressing this case).



Source: Adapted from J. McNitt, "In Good Company: An Employee Recognition Plan with Staying Power," Compensation and Benefits Management, Spring 1990, pp. 242‑246.





6.         Students may debate the issue of executive compensation with the additional information provided. In addition, the following presents a position against executive compensation as it is now managed:



The argument against the current system of executive compensation consists of two points. First, American CEOs are paid too much and their salaries bear no relation to the performance of their companies. Second, critics suggest that the irrational system of executive incentives saps the competitiveness of U.S. companies and is a major contributor to U.S. economic woes. The average pay of an American CEO is $2.4 million a year. Only a paltry 4 percent of the salary differential among executives can be explained by the performance of their companies. Graef Crystal, author of In Search of Excess: The Overcompensation of American Executives, says that CEOs get paid hugely in good years, then merely wonderfully in bad years. For example, Crystal points to option‑repricing schemes in which the price at which an option can be exercised ("the strike price") is lowered as the stock falls. These schemes reward managers even when the performance of the company slips (Frank Lorenzo of Texas Air received this type of option). In Crystal's view, American CEO compensation is an insider's game; everyone wins except the shareholders. CEOs tend to control their compensation by appointing friends to the board of directors, paying them handsomely, and having the favor returned when it is time to ratify a compensation plan. The Business Week article cited did note that there is now more resistance on boards to this sort of activity; however, this resistance appears minimal when one views the 1993 salary increases.



1.         What position do you hold about executive compensation? Why?



2.         Formulate a "pro" position for current executive compensation.



3.         What type of compensation plan should executives be provided that would motivate them to do the best possible job for the shareholders? Do you believe that this accountability is the only one that executives have?



Source: Adapted from A. R. Brownstein, and M. J. Panner, "Who Should Set CEO Pay? The Press? Congress? Shareholders?" Harvard Business Review, May/June 1992, pp. 28‑32+.





Have students discuss the advantages and disadvantages of Employee Stock Ownership Plans (ESOPs). One good source for reference is "Avis Employees Find Stock Ownership Is Mixed Blessing" by J. Hirsch, The Wall Street Journal, May 2, 1995, p. B1.
您需要登录后才可以回帖 登录 | 注册

本版积分规则

快速回复 返回列表 联系我们

社区首页|美华管理传播网|中国经济管理大学|中国营养治疗网|北京管理培训|天津管理培训|重庆管理培训|哈尔滨管理培训|上海管理培训|深圳管理培训|浙江管理培训|广东管理培训|新疆管理培训|内蒙古管理培训|青海管理培训|广西管理培训|西藏管理培训|吉林管理培训|沈阳管理培训|辽宁管理培训|河北管理培训|山东管理培训|安徽管理培训|福建管理培训|海南管理培训|贵州管理培训|四川管理培训|湖北管理培训|河南管理培训|安徽管理培训|江西管理培训|深圳管理培训|广州管理培训|珠海管理培训|香港管理培训|免费公益MBA培训|台湾管理培训|中国管理传播网|全国管理培训认证网|经理人的网上家园|中国管理人才库|经理圈|Archiver|手机版|美华管理人才学校|学校新浪微博V|管理考证|MBA公益课堂|律师声明:知识产权保护声明|

黑公网安备 23018302010102号

QQ

GMT+8, 2024-3-29 03:58 , Processed in 0.266809 second(s), 20 queries , Gzip On. ICP备13013142号

Powered by Discuz! Templates yeei! © 2001-2010 Comsenz Inc.

快速回复 返回顶部 返回列表